Download MCQs 3.25MB 2017-03

Document related concepts

Photoreceptor cell wikipedia , lookup

Blast-related ocular trauma wikipedia , lookup

Keratoconus wikipedia , lookup

Retinal waves wikipedia , lookup

Retina wikipedia , lookup

Marfan syndrome wikipedia , lookup

Human eye wikipedia , lookup

Dry eye syndrome wikipedia , lookup

Visual impairment due to intracranial pressure wikipedia , lookup

Idiopathic intracranial hypertension wikipedia , lookup

Macular degeneration wikipedia , lookup

Cataract surgery wikipedia , lookup

Cataract wikipedia , lookup

Diabetic retinopathy wikipedia , lookup

Mitochondrial optic neuropathies wikipedia , lookup

Retinitis pigmentosa wikipedia , lookup

Transcript
1. Wilson's disease:
a. autosomal dominant
b. cavitation and neuronal loss in putamen and globus pallidus
c. low ceruloplasmin
d. Kayser-Fleischer ring persists despite treatment
e. Kayser-Fleischer ring is due to deposition of copper in
Descemet membrane
2. Retinoblastoma:
a. is the commonest intraocular tumour of infancy and childhood
b. constitute about 10% of childhood malignancies
c. the diffuse infiltrating type usually affects girls
d. perivascular mantles of surviving cells enclosed by white necrotic tissue is
pathognomonic
e. Flexner-Wintersteiner rosette is a pathological feature.
3. Homocystinuria:
a. is associated with retinal detachment
b. increase blood methionine level
c. X-linked recessive
d. associated with lens subluxation
e. risks of thrombosis under general anaesthesia
4. Neurofibromatosis is associated with:
a. bilateral glaucoma
b. Lisch nodules
c. pulsating exophthalmos
d. S-shaped nodule
e. cafe-au-lait spots
5. Mooren's ulcer:
a. better prognosis if less than 40 years old
b. is painful
c. most rapid growth centrally
d. conjunctival resection occasionally useful
e. 2-5% bilateral
1
6. Superior oblique palsy commonly associated with:
a. a blow on the vertex
b. congenital malformation
c. medial orbital wall fracture
d. myokymia
e. internal hydrocephalus
7. Coloboma of the choroid and retina:
a. is usually autosomal recessive
b. is associated with retinal detachment
c. is usually inferotemporal
d. does not involve the macula
e. causes autofluorescein
8. Ocular cicatricial pemphigoid:
a. causes intra-epithelial bullae
b. has antibodies to basement membrane
c. is autosomal recessive
d. dry eye is an early presentation
e. steroid alone is ineffective
9. Acanthoamoeba keratitis can be confirmed by:
a. Grams stain
b. Giemsa stain
c. non-nutrient agar overlaid by E.coli
d. indirect immunofluorescent antibody stain
e. Saboraoud's agar
10. Blow-out fracture is associated with:
a. decrease elevation
b. decrease depression
c. decrease abduction and adduction
d. enophthalmos
e. lateral canthus displaced downwards.
Past MRCOphth/MRCS questions: 2
2
1. After cataract extraction:
a. plus cylinder at the meridian of right suture
b. plus cylinder at 900 from meridian of a removed tight suture
c. minus cylinder at 600 meridian relieved by suture removal at 1500
d. minus cylinder at 200 aggravated by suture removal at 1100
e. minus cylinder at 1300 relieved by suture removal at 900
2. Optic nerve trauma in head injury:
a. usually involves the intracranial portion
b. commonly produces altitudinal defects
c. often complicated by optic atrophy
d. causes contralateral relative afferent pupillary defect
e. visual evoked potential showed delayed latency
3. Causes of rubeosis iridis include:
a. choroidal melanoma
b. stage 2 retinopathy of prematurity
c. irradiation treatment
d. aortic arch syndrome
e. total retinal detachment
4. Adult type retinoschisis:
a. is bilateral
b. split at outer plexiform and inner nuclear layers
c. usually found in the superotemporal quadrant
d. the fluid in the cavity derives from vitreous
e. 2% incidence in autopsy
5. Cycloplegic refraction in neonates:
a. 0.5% cyclopentolate can cause seizure
b. 2.5% phenylephrine can cause increase blood pressure and central nervous system
haemorrhage
c. 0.5% proparacaine decreases the blinking reflex and potentiate cyclopentolate action
d. physostigmine subcutaneously is used to treat atropine poisoning
e. cyclopentolate poisoning causes decrease gastric acid and volume
6. Apparent divergent squint can occur in:
a. fixation disparity
b. wide inter pupillary distance
c. exophthalmos
d. wide papillary aperture
e. epicanthal folds
3
7. Aqueous tear deficiency is affected by:
a. pilocarpine
b. antihistamine
c. atropine
d. tricyclic antidepressants
e. acetazolamide
8. Cluster headache:
a. is more common in females
b. pain is persistent for 1-2 days
c. associated with Horner's syndrome
d. associated with epiphora
e. relieved by propanolol prophylaxis
9. Optic nerve drusen:
a. associated with autofluorescence
b. early staining in fluorescein angiography
c. causes subretinal neovascularization
d. is associated with retinitis pigmentosa
e. is associated with abnormal branching of the retinal vessels
10. Vernal conjunctivitis is characterized by:
a. cobble stone papillae
b. Trantas' dots
c. mucus secretion
d. superior peripheral ulcer
e. recurrent uveitis
Past MRCOphth/MRCS questions: 3
1. Red-green colour defect occurs in:
a. Stargardt's disease
b. autosomal dominant optic nerve drusen
c. toxic optic neuropathy
d. optic neuritis
4
e. glaucoma
2. Benign intracranial hypertension:
a. is associated with tetracycline use
b. diagnosed by CT scan
c. is associated with frontal headache
d. can be treated with oral steroids
e. causes hard exudate in the macular
3. Lyme's disease:
a. is caused by Borrelia burgdoferi
b. is a louse born disease
c. is treated with doxycycline in stage 1
d. is associated with pathognomonic erythema migrans
e. causes sight threatening panuveitis
4. In cataract extraction:
a. intracapsular cataract extraction is used for subluxated lens
b. cystoid macular oedema is more common in ICCE than ECCE
c. ICCE is associated with a higher incidence of retinal detachment than ECCE
d. in pseudoexfoliation syndrome, ECCE put less stress on the zonule than
phacoemulsification
e. posterior capsule rupture is associated with an increased incidence of cystoid macular
oedema
5. Keratoconus:
a. is associated with Down's syndrome
b. is associated with Leber's congenital amaurosis
c. can be treated by epikeratophakia in early stage
d. is more common in teenage body
e. commonly gives rise to eccentric in inferotemporal quadrant.
6. The following are true about systemic lupus erythematosus:
a. the retina is the most commonly involved ocular structure
b. Sjogren's syndrome is a recognized complication
c. it can give rise to positive VDRL
d. it gives rise to ischaemic choroidopathy
e. it produces lupus anticoagulant which is associated
with central retinal vein occlusion
5
7. The following are true about pupil reaction:
a. Argyll Robertson's pupils are typically small and irregular
b. a small pupil excludes Adie's pupil
c. dilated pupil is a feature of advanced optic atrophy
d. spherical aberration decreases with pupil constriction
e. following radial sphincterectomy, pupil constriction does not occur
8. The following are true about refractive errors:
a. more plus lens should be added if the patient sees the red letters clearer in Duochrome test
b. more plus lens will be needed if the patient is on MAO inhibitors
c. more plus lens is needed in a patient who accommodates excessively
d. more minus lens is required in poorly controlled diabetes
e. hypermetropic shift occurs in nuclear sclerosis.
9. Aneurysms of the following vessels causes third nerve
compression:
a. posterior cerebral artery
b. inferior cerebral artery
c. internal carotid artery
d. posterior communicating artery
e. anterior communicating artery
10. The following are true about contact lens:
a. the power is higher than spectacle prescription in myopia
b. contact lens made off silicone-hydrogel has a higher oxygen transmission rate than
PMMA
c. it is the treatment of choice in paediatric aphakia
d. soft contact lens can not be used to correct astigmatism
e. rigid gas permeable lens has a bigger diameter than soft contact lens
Past MRCOphth/MRCS questions: 4
1. Man with X-linked retinitis pigmentosa:
a. must have an affected mother
b. must have a carrier mother
6
c. 50% of sons affected
d. daughters must be affected
e. tends to have poor visual prognosis than autosomal dominant type
2. Cerebral hemisphere lesion:
a. homonymous hemianopia
b. bitemporal hemianopia
c. dysphasia
d. focal epilepsy
e. intentional tremor
3. The following are true about botulinum toxin:
a. cannot be used under general anaesthesia
b. can only be used for the horizontal recti
c. ptosis is one of the complications when used for blepharospasm
d. will develop antibodies in blood
e. the onset of action takes 24 hours
4. Regarding ultrasound biometry:
a. higher frequency gives better resolution
b. wider beam width gives more accurate result than narrower beam width
c. a central fixation target gives more accurate result
d. modern instruments allows 0.01 mm resolution
e. measurements repeated in a pseudophakic patient gives same result as in pre-operative
state
5. In ophthalmic ultrasound:
a. lower frequency gives better resolution than higher frequency
b. sound wave is refracted in the lens
c. highest signal in ocular tissues is from the sclera
d. high internal reflectivity in melanomas
e. will not show up retinoblastoma calcification
6. SF6:
a. is lipid soluble
b. causes posterior subcapsular cataract
c. absorbs nitrogen from vein rapidly
d. will not expand with a mixture of 40% SF6 and 60% air
e. no electrophysiological evidence of damage to photoreceptors
7
7. With regard to electro-oculogram (EOG):
a. it arises from extraocular muscles
b. its potentials come from retinal ganglion cells
c. its potentials arise from retinal pigment epithelium
d. it is a better test than ERG for Best's disease
e. patients with abnormal ERG usually have abnormal EOG
8. Optic nerve trauma in head injury:
a. usually involves intracranial portion
b. commonly produces altitudinal defects
c. often complicated by optic atrophy
d. causes contralateral relative afferent pupillary defect
e. visual evoked potential (VET) shows delayed latency
9. Moron's ulcer:
a. is a slowly progressive unilateral disease
b. is usually preceded by a viral prodromal phase
c. begins as a grey peripheral infiltrate which breaks down into a ulcer furrow
d. the margin of the ulcer is infiltrated with monocytes and plasma cells
e. it is similar to corneal melting associated with connective tissue diseases such as
rheumatoid arthritis in that the sclera is involved
10. In corneal graft rejection:
a. 90% of immune graft rejection occurs within the first year
b. one ore more pregnancies in the past increases the risk
c. previous grafting in the fellow eye increases the risk
d. one or more blood transfusion in the past increases the risk
e. prior to rejection there is an increase in corneal vascularization.
Past MRCOphth/MRCS questions: 5
1. Exudation into the suprachoroidal space:
a. occurs as a complication of trabeculectomy
b. is a complication of scleritis
c. occurs in nephrotic syndrome
d. may be mistaken as a ring melanoma of the ciliary body
e. the effusion extends forwards in the superficial layers of the ciliary body as far as the
scleral spur
8
2. Posterior subcapsular cataract is a complication of:
a. irradiation
b. prednisolone
c. ethambutol
d. gentamicin
e. chlorpromazine
3. With regard to orbital infection in children:
a. preseptal cellulitis is more common than orbital cellulitis
b. ocular motility defect is a feature of orbital cellulitis
c. CT scan is useful in differentiating preseptal and orbital cellulitis
d. frontal and maxillary sinusitis are common causes
e. the most common pathogens are Pneumococcus, Haemophilus inluenzae, Staphylococcus
4. The following are true about trachoma:
a. mini-pannus is common
b. Tranta's dots occurs in the inferior cornea
c. superior tarsal conjunctiva is more commonly affected than inferior tarsal conjunctiva
d. it is a sexually transmitted disease
e. tetracycline is useful as treatment
5. Goldenhar's syndrome has the following features:
a. hypertelorism
b. iris and choroid coloboma
c. decreased corneal sensation
d. microphthalmos
e. under-development of the mandible
6. Rubella cataract:
a. can be unilateral
b. is usually cortical
c. 50% of congenital cataract in the UK is caused by rubella
d. is the commonest presentation of congenital rubella
e. usually occurs when the mother acquires the disease in the third trimester
7. Sympathetic ophthalmia:
a. about 80% occurs within the first week of ocular trauma
b. causes granulomatous inflammation
c. causes Dalen-Fuch's nodules within the retina
9
d. is curative by removing the inciting eye
e. can occur following cataract surgery
8. Dermolipoma:
a. occurs most common in the supero-nasal quadrant
b. is a harmatoma
c. requires early excision as 1/3 of them turns malignant
d. can infiltrate into orbital fat
e. is associated with Goldenhar's syndrome
9. In a baby with epiphora:
a. conservative treatment until 3 years of age is recommended
b. endotracheal tube should be inserted, if syringing and probing were to be performed under
general anaesthesia
c. recurrent orbital cellulitis is a common complications
d. congenital glaucoma needs to be excluded
e. the obstruction is usually at the common canaliculus
10. PHPV (persistent hyperplastic primary vitreous):
a. is caused by failed reabsorption of the hyaloid vascular system
b. is usually unilateral
c. causes angle closure glaucoma
d. causes cataract
e. is a cause of intraocular calcification in the neonate
Past MRCOphth/MRCS questions: 6
1. In corneal grafts:
a. anterior synechia increases the chances of rejection
b. specific allograft rejection often occurs within 2 weeks of transplantation if the host
cornea is vascularized
c. graft failure during the first 2 weeks may be due to reactivation of herpes simplex
d. topical steroids have a lympholytic effect on killer lymphocytes which are responsible for
endothelial rejection
e. subepithelial infiltrates, epithelial oedema and keratitic precipitates are signs of rejection
10
2. The following are true about anti-glaucoma drugs:
a. pilocarpine reduces production of aqueous by exerting an inhibitory action on the ciliary
epithelium
b. sympathomimetics reduce intraocular pressure by increasing uveal scleral outflow
d. carbachol increases aqueous outflow via the trabecular meshwork thereby lowering
intraocular pressure
d. pilocarpine induced miosis begins 5-10 minutes after instillation and lasts 4-8 hours
e. echothiophate iodide has accommodation spasm, cataractogenicity, iris cysts and retinal
detachment side-effects.
3. With regard to the cranial nerves:
a. trochlear nerve paralysis may be idiopathic in 1/3 of cases
b. trochlear nerve paralysis is commonly caused by diabetes mellitus
c. abducens nerve paralysis is frequently caused by diabetes mellitus
d. the tricolour nerve is more frequently damaged by neoplasms than the abducens nerve
e. trochlear nerve paralysis, can be caused by intracranial aneurysms.
4. In the treatment of retinoblastoma:
a. reduction of the total dose given by avoiding usage of supervoltage equipment reduces the
complications of irradiation
b. Group III B tumours should be enucleated
c. gold spheres should be used after enucleation as adjunctive radiotherapy may be required.
d. CNS spread of tumour treated with intrathecal methotrexate can result subacute
leukoencephalopathy
e. external beam irradiation results in posterior subcapsular cataract in 20% of cases
5. In cataract surgery:
a. phacoemulsification has the advantage of a smaller section resulting in less astigmatism
b. YAG capsulotomy for thickened posterior capsule can result in retinal detachment
c. uveitis-glaucoma-hyphaema syndrome is an early post-operative complication of cataract
extraction
d. with the rule astigmatism is associated with loose sutures
e. the extracapsular technique can cause sensitization to the endothelial cells resulting in
corneal decompensation
6. Temporal lobe tumours:
a. causes homonymous hemianopia
b. are associated with formed visual hallucinations
c. may present with psychomotor epilepsy
d. may cause Foster-Kennedy syndrome
11
e. may cause ataxic nystagmus
7. Optic nerve glioma:
a. is more common in children than in adults
b. originates form the myelin sheath of the optic nerve fibres
c. causes erosion of the optic foramen margin which is demonstratable radiologiacally
d. causes loss of vision early
e. causes axial proptosis
8. Sturge-Weber's syndrome is:
a. associated with adenoma sebaceum
b. associated with unilateral congenital glaucoma
c. associated with retinal capillary haemangioma
d. autosomal; dominant
e. associated with intracranial calcifications
9. Parinaud's syndrome consists of:
a. ataxia
b. vertical gaze paralysis
c. pupillary areflexia to light
d. optic atrophy
e. convergence weakness
10. The following may be causes of unilateral oculomotor
palsy with pupil involvement:
a. migraine
b. herpes zoster
c. myasthenia gravis
d. diabetes mellitus
e. posterior communicating artery aneurysm.
Past MRCOphth/MRCS questions: 7
1. Advantages of cryotherapy over diathermy include:
a. cryotherapy causes little or no scleral damage
b. cryotherapy shrinks the sclera thus facilitating closure of the drainage site
12
c. cryotherapy does not force fluid from the eye thus maintaining a reasonable intraocular
pressure cryotherapy can be safely applied over staphylomatous sclera
e. cryotherapy causes no damage to the vortex vein or ciliary arteries
2. Juvenile retinoschisis:
a. is an autosomal recessive condition
b. there is classical cystoid oedema at the macula
c. the split is in the nerve fibre layer
d. is a bilateral disease
e. it is most commonly seen in the superior nasal quadrant
3. Complications of retinal detachment operations include:
a. strabismus
b. refractive error
c. macular pucker
d. anterior segment necrosis
e. angle closure glaucoma
4. Regarding buckles:
a. encirclage reduces vitreous traction in the eye
b. scleral dissection is required in explants
c. encirclage is indicated when there is evidence of proliferative vitreo-retinopathy
d. an encirclage can be used if no holes are found on examination
e. maximal reduction of vitreous traction can be achieved only if the encircling band
constricts the vitreous base.
5. After retinal detachment:
a. protein synthesis in the retina is increased
b. intraretinal oedema appears
c. there is a disruption of the photoreceptors
d. retinal pigment epithelial cells can proliferate and they appear as subretinal white dots
e. large intraretinal cysts form after a short period of time
6. Syphilitic keratitis is characterized by:
a. early pain and photophobia
b. diffuse vascularization
c. stromal haze heaviest in the centre
d. lack of corneal sensitivity
e. punctate epithelial keratopathy
13
7. Keratopathies in which fat is a prominent feature include:
a. arcus senilis
b. lipid dystrophy
c. keratitis centralis annularis
d. chronic scleritis
e. Stocker's line
8. Macular dystrophy is characterized by:
a. diffuse and rapidly progressing cloudiness
b. superficial lesions in central cornea
c. stromal lesion in peripheral cornea
d. early impairment of visual acuity
e. autosomal dominant inheritance
9. Band keratopathy:
a. appears in children with Still's disease
b. is associated with arthritis and iritis
c. appears in the interpalpebral area
d. slit lamp reveals dark holes in bowman's membrane
e. may be removed with sodium EDTA
10. Recurrent corneal erosions:
a. improve during the waking hours
b. can be treated with topical steroids in acute phase
c. results from epithelium removal by the lids in sleep
d. characteristically causes symptoms in the evening
e. can be treated with a bandage contact lens.
Past MRCOphth/MRCS questions: 8
1. Trachoma follicles:
a. occur in the conjunctival stroma
b. consist of lymphocytes in early stage
c. may show necrosis in the central zone
d. are not accompanied by submucosa inflammation
e. are pathognomonic of the disease
14
2. Pseudomembranes are produced in:
a. gonorrhoeal ophthalmia
b. pneumococcal conjunctivitis
c. meningococcal conjunctivitis
d. Steven-Johnson syndrome
e. herpes simplex conjunctivitis
3. There is an association between the following:
a. iris coloboma and lens dislocations
b. spherophakia and Weill Marchesani syndrome
c. ataxia telangiectasia and rubeosis irides
d. juvenile chronic rheumatoid arthritis and band keratopathy
e. Bardet-Biedl syndrome and night blindness
4. Retinoblastoma:
a. 90% of patients have a germ line mutation
b. is associated with an abnormality of the short arm of chromosome 13
d. rarely shows calcified flecks
e. is classified according tot he Callender system
5. Ocular manifestations and syndromes recognized
associations are:
a. eyelid coloboma and Treacher-Collins
b. keratoconus and Down
c. convergent squint and Crouzon's
d. hypertelorism and Kleinfelter
e. limbal dermoids and von Recklinghausen
6. Retinopathy of prematurity:
a. apnoea is a risk factor
b. often regresses spontaneously
c. is associated with hypovitaminosis E
d. is treated with cryotherapy at stage 2a
e. is associated with myopia
7. Aniridia:
15
a. is inherited as an autosomal recessive disease
b. rarely leads to glaucoma
c. often causes photophobia
d. is associated with nephroblastoma
e. mainly affects oculocutaneous albinos
8. Following penetrating ocular injury:
a. cryotherapy must be applied to sites of scleral rupture greater than 6 mm from the limbus
b. vitreous incarceration invariably leads to retinal detachment
c. posterior vitreous detachment usually occurs within the first 48 hours
d. fibronectin may play a role in wound healing
e. vitrectomy is invariably carried out when removing intraocular foreign bodies
9. Behcet's disease:
a. is commonly associated with HLA B5 and HLA BW51
b. oral ulcers are the most frequently seen clinical feature
c. arthritis is not a feature
d. confusional state is well recognized
e. is associated with retinal infiltrates
10. Infants:
a have average ocular axial lengths of approximately 17 mm
b. ideally require implants of approximately 35 dioptres for vision after cataract extraction
c. with congenital cataracts often have small miosed pupils
d. after unilateral congenital cataract surgery should have at least 3 hours patching a day to
the better day
e. have less post-operative inflammation after cataract surgery adults do.
Past MRCOphth/MRCS questions: 9
1. Recognized side effects of cyclopsorin A include:
a. renal toxicity
b. cerebellar deficit
c. hirsutism
d. gingival hyperplasia
e. hepatotoxicity
16
2. Disc drusen:
a. are familial
b. show pseudofluorescence
c. usually cause central field loss
d. may be associated with disc haemorrhage
e. become less obvious with increasing age
3. The diagnosis of an orbital blow out fracture is suggested by:
a. enophthalmos
b. hypoanaethesia of the gum
c. restricted down-gaze
d. positive forced duction test
e. orbital margin tenderness
4. Multiple sclerosis can cause:
a. diplopia
b. abduction nystagmus
c. internuclear ophthalmoplegia
d. unilateral optic disc oedema
e. lateral rectus saccadic paresis during versions
5. Which of the following are true of lacrimal gland tumours:
a. benign mixed tumours (pleomorphic adenomas) constitute 50% of all lacrimal gland
swellings
b. adenoid cystic carcinoma is the commonest epithelial tumour of the lacrimal gland
c. pleomorphic adenoma may undergo malignant change
d. pain is a predominant feature in malignant lesions
e. suspected benign mixed tumours need to be biopsied for tissue diagnosis prior to a
definitive treatment
6. Treatment of benign intracranial hypertension includes:
a. acetazolamide
b. weight reduction
c. steroids
d. limbo peritoneal shunt
e. anti-prostaglandins
17
7. Drusen of the optic nerve head are associated with:
a. refractile bodes
b. leakage of fluorescein on angiography
c. no filed defects
d. central serious retinopathy
e. peripaillary haemorrhages
8. Six months after resolution of an acute attack of retrobulbar
neuritis the following are often present:
a. defects of the nerve fibre layer
b. impaired colour vision
c. normal visual acuity
d. altitudinal field defects
e. Marcus Gunn pupil
9. Pigmentary retinopathy is a recognized feature of:
a. dystrophic myotonica
b. chloroquine toxicity
c. Duchenne muscular dystrophy
d. thioridazine toxicity
e. mitochonridal myopathies
10. True statements include:
a. the Steel-Richardon syndrome is associated with slow saccades
b. lesions in the posterior commissure cause abnormalities in horizontal eye movements
c. the maximum velocity for pursuit movements is 400 degrees per second
d. frontal eye fields generate ipsilateral saccades
e. phenytoin may cause defective pursuit
Past MRCOphth/MRCS questions: 11
1. Corneal sensation:
a. is always decreased in neurofibromatosis
b. corneal hyperaesthesia is a form of diabetic sensory neuropathy, and is seen in patients
with proliferative diabetic retinopathy
c. neuropathic keratopathy is a contraindication for performing laser panretinal
photocoagulation in eyes with proliferative diabetic retinopathy
18
d. is reduced following herpes zoster keratitis
e. is reduced by prolonged hard contact lens wear
2. Pigmentary glaucoma:
a. typically occurs in patients from 20 to 50 years of age
b. there is often a positive family history of pigmentary glaucoma
c. the onset is usually rapid with corneal oedema and blurring of vision
d. pigmentary glaucoma is commonly caused by diabetes mellitus
e. is often seen in high myopes with shallow anterior chambers
3. In central retinal vein occlusion:
a. there may be a reversible shallowing of the anterior chamber following vein occlusions,
sufficient to cause angle closure glaucoma
b. neovascular glaucoma usually develop within five weeks of the development of central
vein occlusion
c. the neovascular glaucoma that occurs secondary to occlusion of the central retinal vein is
best treated by an iridectomy
d. neovascular glaucoma is best treated with cycloplegic agents and steroid
e. the pupillary light reflex is not useful in the assessment of retinal ischaemia
4. In glaucoma:
a. visual field loss may be aggravated by systemic hypotension
b. an arcuate scotoma can be increased by artificial elevation of the intraocular pressure
c. size and depth of cupping of the disc is always related to the field defects
d. there may be arteriorsclerotic changes in nutrient vessels 1-2 mm behind the optic nerve
head
e. nasal step usually precedes arcuate scotoma
5. Pilocarpine is:
a. a lactone of pilocarpine acid
b. a contractor of ciliary muscle and sphincter
c. more effective as its concentration is increased
d. useful in the treatment of malignant glaucoma
e. the first line of treatment of open angle glaucoma in 30
years old
6. Chronic retinal detachments:
a. are usually mobile
b. may show high water marks
19
c. are associated with retinal cysts
d. may be associated with subretinal fibrosis
e. causes shrinkage of the globe
7. Proliferative vitreoretinopathy:
a. is an uncommon cause of failure in retinal detachment surgery
b. Grade C may involve all 4 quadrants
c. Grade D2 implies that the optic disc is visible
d. is more commonly with larger retinal tear
e. is unlikely to be helped by surgery
8. Pulsation of orbital contents may occur in:
a. orbital varix
b. carotid cavernous fistula
c. neurofibromatosis
d. thyrotoxicosis
e. blow out fracture of the orbital roof
9. Angoid streaks are associated with:
a. sickle cell anaemia
b. pseudoexfoliation
c. Marfan's syndrome
d. myopic degeneration
e. tuberous sclerosis
10. With regard to trauma:
a. in a Le Fort type I fracture, fracture lines are found
through the maxilla, orbital floor and lateral wall of the
orbit
b. following a blunt injury with hyphaema some degree of
angle recession will be evident in approximately 5% of
cases
c. topical steroid drops should be used to suppress
collagenase activity during the second week following
an alkaline burn
d. the second commonest site of blow out fracture of the
orbit is the roof of the orbit
e. iron containing intraocular foreign bodies must be
removed within 12 hours of injury if siderosis bulbi is to
be prevented
20
Past MRCOphth/MRCS questions: 12
1. Paving-stone retinal degeneration:
a. is present in 50% of patients
b. is located superiorly
c. is located posterior to the equator
d. does not predispose to retinal breaks
e. is most prevalent in younger age groups
2. The following are causes of non-rhegmatogenous retinal
detachment:
a. eclampsia of pregnancy
b. Wagner's vitreoretinal degeneration
c. malignant hypertension
d. Coat's disease
e. haemangioma
3. In toxic amblyopia:
a. both eyes are usually involved within 3 months of each
other
b. treatment consists of abstention from tobacco and
alcohol plus cyanocobalamin injections
c. improvement is usually seen in 3 months if patient
adheres to the prescribed regimen
d. visual fields abnormalities may include junctional
scotoma
e. the scotomas are larger for red and green targets than
for blue
4. In retinitis pigmentosa:
a. perifoveal telangiectasia have been reported
b. use of recombinant probe shows a locus on the long
arm of the X chromosome in X linked retinitis
pigmentosa
c. there may be serum lipid abnormalities in some cases
d. fluorescein angiogram may show a subtle bulls eye
pattern
21
e. optic nerve pallor is not a diagnostic criteria
5. In Bird-shot chorioretinopathy:
a. mild anterior chamber reaction is often seen
b. arteriolar narrowing and vascular sheathing can be seen
c. ERG shows decreased amplitude
d. there is an associations with HLA-A 29
e. there is a sensitization to retinal S antigen
6. Acute retinal necrosis:
a. is associated with retinal tears and retinal detachment
b. is caused by cytomegalovirus
c. is caused by herpes zoster virus
d. acyclovir given early is useful in presenting retinal
detachment
e. Bechet's disease is a differential diagnosis
7. Regarding intraocular gases (SF6) used in retinal detachment
surgery:
a. sulfur hexafluoride is highly lipid soluble
b. there is rapid influx of venous nitrogen into the
intravitreal pocket of gas
c. a mixture of 40% SF6 and 60% air does not increase
in volume when left in the eye
d. posterior capsular cataracts are a complications
e. there is no electrophysiologic evidence of toxicity
8. With regard to ultrasound:
a. a choroidal haemangioma shows a highly reflective
A-scan pattern
b. a choroidal melanoma shows a highly reflective A-scan
pattern
c. a metastatic carcinoma has medium high retinal
reflectivity
d. a disciform scar has high reflectivity
e. a fresh subretinal haemorrhage has low reflectivity
9. In retinoblastoma:
a. the ratio of aqueous to plasma lactate dehydrogenase is
22
elevated
b. the ratio of aqueous to plasma phosphoglucose
isomerase is reduced
c. aqueous and vitreous aspirate for cytology may help
differentiate retinoblastoma from other lesions
d. a CT scan is useful to detect intraocular tumours with
calcification
e. there is an association with deletion of specific band in
chromosome 13p14
10. Vitamin B deficiency:
a. may produce a decrease in central vision with relative
sparing of peripheral vision
b. colour vision is not involved
c. in long-standing cases temporal pallor of the optic
nerve is always present
d. retinal haemorrhages may be present
e. is actually the cause of alcohol-tobacco amblyopia.
Past MRCOphth/MRCS questions: 10
1. Retinal laser treatment should be:
a. heavy in pan-retinal photocoagulation
b. gentle for subretinal neovascular membrane
c. heavy for macular grids
d. gentle for retinal breaks
e. gentle for exudative maculopathy
2. The optokinetic nystagmus (OKN) response:
a. is impaired when the OKN drum rotates towards the
side of a deep parietal lesion
b. is impaired when the OKN drum rotates towards the
side of a unilateral lesion of the vestibular nucleus
c. is accompanied by convergence retraction nystagmus
in cases of pineal tumours when the OKN drum is
rotated in an upward direction
d. is reversed in congenital nystagmus
e. is usually normal in malingerers
23
3. Chronic progressive external ophthalmoplegia:
a. diplopia is a common feature
b. is usually seen in the under 20 age group
c. is associated with ocular pharyngeal dystrophy
d. has a good prognosis
e. is a feature of Kerans-Sayre syndrome
4. The following are true:
a. X-linked retinoschisis usually involves the central and
peripheral retina
b. retinoschisis is usually treated surgically
c. sickle cell retinopathy is characterized by "sunburst"
lesions
d. sickle cell retinopathy is a feature of sickle cell trait
e. haemoglobin SC disease is often associated with "sea
fan"
5. Keratoplasty in the infant:
a. a fFieringa ring should be attached to the infant globe
before surgery
b. the infant pupil may not dilate postoperatively because
of hypoplasia of the musculature of the iris
c. it is best to use donor tissue from children or infant in
keratoplasty where the host is child or infant
d. it is best not to use continuous suture technique with
buried knot
e. repeated, frequent examinations under anaesthesia are
important in follow up
6. Keratoconus:
a. this condition is most often a bilateral impairment
b. visual impairment stems from the irregular hyperopic
astigmatism
c. this is a non-inflammatory condition found usually in
adolescents
d. abnormally low ocular rigidity is found to be
characteristic of most patients with keratoconus
e. may be caused by soft contact lens
7. Bilateral corneal crystalline deposits may occur in:
a. cystinosis
b. gout
24
c. monoclonal gammanopathy
d. Salzmann's dystrophy
e. Schnyder's dystrophy
8. Treatment modalities which may be used in the treatment
of herpes simplex keratitis include:
a. oral acyclovir
b. oral steroids
c. topical trifluorothymidine
d. mechanical debridement
e. corneal graft
9. Interstitial keratitis may be present in:
a. syphilis
b. herpes zoster
c. neurofibromatosis
d. Cogan's syndrome
e. Lyme's disease
10. Immunosuppressive therapy may be beneficial in:
a. retinoblastoma
b. Coats' disease
c. Mooren's ulcer
d. Terrien's peripheral ulcer
e. Behcet's disease
Past MRCOphth/MRCS questions: 13
1. With regard to nutrition:
a. zinc deficiency is commonly associated with difficulty of
dark adaptation - unresponsive to vitamin A treatments
b. cystinosis results in a peripheral pigmentary retinopathy
c. gyrate atrophy is a dominantly inherited disease
d. restriction of arginine and administration of pyridoxine
is helpful in gyrate atrophy
e. Menkes' syndrome is associated with copper excess
2. Diabetic retinopathy:
25
a. is more prevalent in young men than young women
b. is primarily a disease of the posterior retina
c. an early histopathologic finding is relative loss of endothelial cells
d. new blood vessels come mainly from the arterial side of the circulation
e. depends mainly on the duration of the disease
3. The ideal contact lens:
a. should have a high DK value for gas permeability
b. should be soft
c. should give clear vision
d. should be durable
e. should not be resistant to deformation
4. Hard contact lenses can be made from:
a. PMMA - polymethylmethacrylate
b. CAB - cellulose-acetyl-butyrate
c. HEMA - hydroxyethelmethacrylate
d. silicone and PMMA copolymer
e. glass
5. Hard contact lenses are better than soft contact lens because:
a. they are more permeable to oxygen
b. they are easily processible
c. they are smaller in diameter
d. they are easier to sterilize
e. they are not hydrophilic
6. Soft contact lenses are better than hard contact lenses
because they:
a. do not cause overwearing syndrome
b. do not cause spectacle blur
c. scratch less easily
d. do not not cause refractive changes
e. dislocate less less easily
7. Residual astigmatism is:
a. astigmatism due to corneal distortion
b. astigmatism which cannot be corrected with glasses
c. astigmatism which cannot be corrected with contact lens
26
d. astigmatism due to lenticular distortion
e. correctable by photorefractive surgery
8. Giant papillary conjunctivitis is related to:
a. contact lens deposits
b. an autoimmune reaction
c. soft contact lenses
d. hard contact lenses
e. cold sterilization of contact lenses
9. Acetzolamide:
a. is know to cause aplastic anaemia
b. may cause a metabolic acidosis
c. may cause a metallic taste in the mouth
d. causes paraesthesia in 30% of patients on treatment
e. may precipitate liver failure in cirrhosis
10. Latanoprost:
a. inhibits cyclo-oxygenases
b. decreases aqueous production
c. causes cystoid macular oedema in aphakic patients
d. causes pigment dispersion syndrome
e. increases aqueous outflow through the trabecular
meshwork
Past MRCOphth/MRCS questions: 14
1. The following substances usually lower the intraocular
pressure:
a. para-amino clonidine
b. salbutamol
c. ethyl alcohol
d. marijuana
e. the prostaglandin PGF2 alpha
27
2. Fuchs' endothelial dystrophy:
a. is the most common reason for penetrating keratoplasty in developed countries
b. is more common in men
c. causes worsening of vision during the course of a day
d. may be helped by using a hair dryer on the eye
e. is often associated with other systemic diseases
3. Acanthamoeba keratitis:
a. is caused by Acanthamoeba castellani
b. is caused by Acanthamoeba polyphaga
c. has not been successfully treated medically
d. is charateristically painless
e. may be successfully cultured on Escerichia coli enriched chocolate agar
4. Intraocular gases used in retinal detachment surgery:
a. pure sulphur hexafluoride is non-expansile
b. a one ml bubble of C3F8 disappears in 10 days
c. pneumatic retinopexy is not recommended in patients with glaucoma
d. are best injected into the most dependant part of the eye
e. should not be used with nitrous oxide anaesthesia because this causes an immediate
decrease in bubble size
5. Light-near dissociation occurs in:
a. myotonic dystrophy
b. chronic progressive external ophthalmoplegia
c. Holmes-Adie pupil
d. Parinaud's syndrome
e. Wernicke's pupil
6. Disc drusen:
a. are similar histologically to retinal drusen
b. become progressively embedded with age
c. may cause arcuate field defects
d. may cause sudden loss of vision
e. may be associated with anomalous retinal vessel patterns
7. In the optic tract:
a. the macular fibres lie centrally
28
b. lesion causes sectorial optic atrophy
c. lesion may be associated with a contralateral /Marcus-Gunn pupil
d. lesion causes congruous homonymous hemianopia
e. carries 60% of its fibres to the lateral geniculate body
8. Persistent hyperplastic primary vitreous:
a. is unilateral in 90% of cases
b. is associated with calcification early
c. is associated with elongated ciliary processes
d. has an abnormal aqueous/serum lactate dehydrogenase level
e. exhibits relative microphakia
9. Mooren's ulcer:
a. are painful
b. run a chronic course of 10-30 years
c. may show scleral involvement
d. are not associated with uveitis
e. only occurs in elderly patients
10. The following are true about corneal sensation:
a. panretinal photocoagulation therapy may cause a reduction in corneal sensation
b. corneal hypoesthesia is a form of diabetic sensory neuropathy, and is seen in patients with
proliferative diabetic retinopathy
c. neuropathic keratopathy is a contraindication for performing laser panretinal
photocoagulation in eyes with proliferative diabetic retinopathy
d. corneal hypoesthesia may be seen in patients with background diabetic retinopathy
e. cataract surgery can lead to corneal hypoesthesia.
Past MRCOphth/MRCS questions: 15
1. Chemical burns:
a. acid solution readily penetrate the corneal epithelium but are repelled by the lipoprotein
cell walls of the lkeratocytes
b. alkaline corneal burns are less destructive than acid burns
c. solutions of alkaline pH readily penetrate and spread deep into the corneal stroma and into
the anterior chamber
d. acid burns of the cornea are usually sharply demarcated because of the buffering action fo
the tissues
29
e. refrigerant and bleach are common causes of alkali burns
2. The Kayser-Fleischer ring:
a. consists of copper deposits in Bowman's membrane
b. gives a green-brown, grey, orange or brown appearance tot he corneal periphery
c. initially appears as a thin crescent at the 3 and 9 o'clock positions of the cornea
d. may regress with D-penicillamine therapy
e. cannot result from copper intraocular foreign bodies
3. Cogan's syndrome is:
a. a form of acquired syphilis
b. characterized by deafness
c. occasionally associated with lupus erythematousus
d. occasionally associated with polyarteritis nodosa
e. characterized by interstitial keratitis
4. Causes of aqueous tear deficiency:
a. practolol
b. tricyclic anti-depressants
c. facial nerve palsy
d. antihistamines
e. atropine
5. Mucopolysaccharidoses associated with corneal clouding:
a. Hurler
b. Hunter
c. Scheie
d. Morquio
e. Sanfilippo
6. Keratoconus:
a. should be suspected in an adolescent with progressive myopic astigmatism
b. does not exhibit corneal epithelial thinning
c. is accompanied by Bowman's membrane
d. may be associated with Vogt's striae which are fine vertical folds in the deep stroma
e. can be optically corrected with hard contact lenses
7. Causes of chronic conjunctivitis:
30
a. chlamydial infection
b. cosmetics
c. molluscum contagiosum
d. chronic meibomitis
e. psoriasis
8. Bilateral corneal crystalline deposits may occur in:
a. cystinosis
b. gout
c. monoclonal gammopathy
d. Salzmann's dystrophy
e. Schnyder's dystrophy
9. Indications for a larger donor button in keratoplasty:
a. phakic patient
b. shallow anterior chamber
c. peripheral corneal thinning
d. prior history of uveitis
e. aphakic patient
10. Exclusion criteria for donor cornea:
a. previous intraocular surgery
b. retinoblastoma
c. death of unknown cause
d. bowel cancer
e. Creutzfeldt-Jakob disease
Past MRCOphth/MRCS questions: 16
1. Preservation of donor corneas:
a. McCarey-Kaufman preserves endothelial viability up to 6 days
b. moist chamber stored corneas should be transplanted within 48 hours
c. TC 199 is a component of McCarey-Kaufman medium
d. organ culture allows preservation up to one month
e. McCarey-Kaufman eyes should be stored at 10C
31
2. Mooren's ulcer:
a. about 2.5% of cases are bilateral
b. is associated with much pain
c. satisfactory results have occasionally been obtained by resection of the limbal
conjunctiva adjacent to the lesion
d. the most rapid movement of the ulcer is centrally with the leading edge de-epithelialized
and undermined
e. is associated with a better prognosis in patients below their fourth decade of life
3. Band keratopathy can be seen in:
a. adult rheumatoid arthritis
b. sarcoidosis
c. iridocyclitis
d. vitamin D toxicity
e. juvenile rheumatoid arthritis
4. Hassal-Henle warts are associated with:
a. appearance in the central cornea
b. thinning of endothelial cytoplasm near wart
c. fissures over and into substance of wart
d. mouths of fissures beneath endothelial cells
e. there are structural differences from guttae located
elsewhere
5. Corneal involvement in leprosy include:
a. superficial punctate keratitis
b. disciform keratitis
c. pannus
d. deep interstitial keratitis
e. neurotrophic keratopathy
6. Stargardt's disease:
a. is characterized by nyctalopia
b. peripheral fields and colour vision are normal
c. patients present in second and third decade of life
d. has an autosomal recessive as well as dominant mode of inheritance
e. is characterized by an abnormal EOG early in the course of the disease.
32
7. The following are associated with congenital cataracts:
a. incontinentia pigmenti
b. intrauterine hypoxia
c. Lowe's syndrome
d. Patau's syndrome
e. Rubenstein-Taybi syndrome
8. The ocular complications of bone marrow transplantation
include:
a. keratoconjunctivitis sicca
b. cataracts
c. herpes simplex infections
d. cicatricial lagophthalmos
e. corneal keratinization
9. Congenital nystagmus:
a. is binocular and uniplanar
b. is dampened with fixation effort
c. is usually horizontal, but may be vertical, circular, pendular or jerky
d. may be alleviated with the use of base-in prism glasses
e. may be treated surgically with he Kestenbaum procedure
10. Regard topical corticosteroids:
a. dexamethasone phosphate does not penetrate the uninflammed cornea whereas
dexamethasone alcohol does
b. topical steroid solutions penetrate the cornea better than steroid suspensions
c. steroid eye cream achieves a higher concentration in the eye than drops due to a longer
contact time
d. topical steroids reduce corneal polymorphonuclear leukocyte invasion better than
subconjunctival injections of steroid
e. sudden cessation of topical steroids in corneal inflammation can cause a rebound
nercotizing inflammation.
Past MRCOphth/MRCS questions: 17
1. Regarding common ophthalmic antibiotics:
a. chloramphenicol inhibits bacterial protein synthesis
33
b. chloramphenicol is bactericidal to Haemophilus influenza
c. gentamicin prevents folate synthesis in bacterial cells but not in human ocular tissue
d. polymyxin B is effective against all Gram negative bacteria
e. polymyxin B exerts its effect by disrupting bacterial cell membranes
2. Lid retraction may be found in:
a. Parinaud's syndrome
b. dysthyroid eye disease
c. neurosyphilis
d. congenital oculomotor nerve palsy
e. Wegener' granulomatosis
3. Optociliary shunt vessels on the disc are found in:
a. patients after central retinal vein occlusion
b. proliferative diabetic retinopathy after laser photocoagulation
c. orbital meningioma
d. congenital anomaly
e. von Hippel-Lindau's retinal angiomatosis
4. In benign intracranial hypertension, the following amy
be present:
a. normal CT scan
b. normal visual fields
c. normal CSF pressure
d. transient obscurations o f vision on lateral gaze.
e. spontaneous resolution
5. Recurrent corneal erosions:
a. are often caused by corneal embedded foreign bodies
b. superficial corneal dystrophy patients are more likely to get epithelial erosions
c. regeneration of corneal basal epithelial cells require up to 2 months to form new
basement membrane complexes
d. 5% hypertonic saline cream and drops are used in
erosions for up to 1 year
e. soft extended wear contact lenses are worn
continuously for up to 7-10 days after an erosion
recurrence.
6. In inflammation of the conjunctiva:
a. phylctenules are localized necrotic infiltrates with
plasma cells which occur on cornea and conjunctiva
b. follicles are subepithelial infiltration which are
34
histologically characterized by central Leber cells and
surrounding lymphocytes
c. follicles occur most commonly on the upper palpebral
conjunctiva and the upper fornix
d. papillae histologically have a central fibrovascular core
surrounded by acute inflammatory cells
e. the follicle correlates to cellular antibody production
7. Regarding the use of culture media in bacterial keratitis:
a. blood agar only grows aerobic bacteria
b. chocolate agar enhances the isolation of Morexella,
Neisseria and Haemophilius species
c. Sabouraud's agar contains yeast extracts and
antibiotics and is for fungal isolation
d. brain heart media is best used for anaerobes
e. thioglycolate broth is for anaerobic and microareophilic
bacteria
8. Prolapse of the lacrimal gland may occur in:
a. trauma
b. increased intraorbital pressure
c. spontaneous hereditary blepharochalasis
d. usually involves the palpebral lobe of the gland
e. may present as a subconjunctival mass
9. Pseudoxanthoma elasticum:
a. is autosomal dominant
b. is more common in men
c. is also called Groenbland-Strandberg syndrome
d. is associated with hypertension
e. when associated with diabetes mellitus often results in
florid proliferative diabetic retinopathy
10. A falciform fold of the retina:
a. can occur in retinopathy of prematurity
b. can occur in toxoplasmosis
c. can occur in trisomy 21
d. is usually bilateral
e. is usually associated with hypermetropia
35
Past MRCOphth/MRCS questions: 18
1. Retinoblastoma:
a. can present with rubeosis iridis
b. Flexner Wintersteiner rosettes surround a hyaluronidase
resistant and mucopolysaccharide
b. Homer Wright rosettes may also be found in
medulloblastoma and neuroblastoma
d. fleurettes represent the least differentiated tumour
e. trilateral tumour has a better diagnosis
2. Cytomegalovirus infection:
a. is diagnosed by intramitochondrial inclusions of infected
cells
b. in utero results in congenital deformities in 8-% of cases
c. in an adult is commonest in AIDS and presents with
retinal necrosis and haemorrhage
d. causes deafness, choroiditis and cerebral calcifications
e. is caused by DNA herpes virus and responds to
acyclovir
3. The CT scan findings in thyroid ophthalmopathy are:
a. enlargement of extraocular muscle bellies
b. enlargement of tendons of extraocular muscles
c. enlargement of lacrimal gland
d. thickening of optic nerve
e. medial bowing of lacrimal papyracea
4. In radiation and thermal burns:
a. radiation therapy to the paranasal sinuses or pituitary
lesions exposes the optic nerves to direct ionising
radiation
b. optic nerve radionecrosis occurs within 3 years of
treatment
c. thermal burns of the body can cause optic neuropathy
d. thermal burn optic neuritis is due to septicaemia or
circulatory failure
e. the optic nerve is more resistant than lens to radiation
36
5. In the pharmacology of anti-glaucoma drugs:
a. pilocarpine reduces production of aqueous by exerting
an inhibitory action on the ciliary epithelium
b. sympathomimetics reduce intraocular pressure by
increasing uveal scleral outflow
c. carbachol increases aqueous outflow via the trabecular
meshwork thereby lowering intraocular pressure
d. pilocarpine induced miosis begins 2-10 minutes after
instillation and lasts 4-8 hours
e. echothiophate iodide causes accommodation spasm,
cataractogenciity, iris cysts and retinal detachment as
side-effects
6. Optic nerve glioma:
a. is more common in children than in adults
b. originates from the myelin sheath of the optic nerve
fibres
c. causes erosion of the optic foramen margin which is
demonstrable radiologically
d. causes loss of vision early
e. causes axial proptosis
7. Advantages of cryotherapy over diathermy include:
a. cryotherapy causes little or no scleral damage
b. cryotherapy shrinks the sclera thus facilitating closure
of the drainage site
c. cryotherapy does not force fluid from the eye thus
maintaining a reasonable intraocular pressure
d. cryotherapy can be safely applied over staphylomatous
sclera
e. cryotherapy causes no damage to the vortex veins or
ciliary arteries
8. Complications of retinal detachment operations include:
a. strabismus
b. refractive error
c. macular pucker
d. anterior segment necrosis
e. angle closure glaucoma
37
9. Regarding buckles:
a. encirclage reduces vitreous traction in the eye
b. scleral dissection is required in explants
c. encirclage is indicated when there is evidence of
proliferative vitreoretinoapthy
d. an encirclage can be used if no holes are found on
examination
e. maximal reduction of vitreous traction can be achieved
only if the encircling band constricts the vitreous base.
10. Pseudomembranes are produced in:
a. gonorrheal ophthalmia
b. pneumococcal conjunctivitis
c. meningococcal conjunctivitis
d. Stevens-Johnson syndrome
e. herpes simplex conjunctivitis
Past MRCOphth/MRCS questions: 19
1. Trachoma follicles:
a. occur in the conjunctival stroma
b. consist of lymphocytes in early stages
c. may show necrosis in the central zone
d. are not accompanied by submucosal inflammation
e. are pathognomonic of the disease
2. Carbonic anhydrase inhibitors:
a. can cause exfoliation dermatitis
b. are hyperosmotic agents
c. encourage renal stone formation
d. can result in metabolic acidosis
e. can cause potassium depletion
3. Lattice degeneration:
a. is usually found at the equator
b. is associated with abnormal vitreous retinal adhesions
c. develops from paving stone degeneration
38
d. sometimes requires prophylactic treatment
e. is associated with horseshoe breaks
4. A tumour of the temporal lobe can:
a. produce a homonymous hemianopia
b. produce visual hallucinations
c. simulate migraine
d. VIth nerve palsy
e. disturbance of the medial longitudinal bundle
5. The herpes simplex virus can:
a. cause disciform keratitis
b. cause corneal hypoaesthesia
c. be a conjunctival commensal
d. be activated by sunlight
e. produce marginal keratitis
6. Regarding episcleritis:
a. episcleritis nearly always accompanies scleritis
b. episcleritis clears without treatment
c. simple episcleritis is sectoral in 70% of cases and
generalized in 30%
d. vascular congestion is mainly in the superficial
conjunctiva plexus
e. nodular episcleritis may progress to anterior nodular
scleritis
7. Regarding scleritis:
a. posterior scleritis is seldom related to a systemic
disease
b. the most benign form of scleritis is the nodular anterior
type
c. most patients with inflammatory necrotizing scleritis die
within a few years of onset of disease
d. scleromalacia perforans occur in most connective
tissue disease
e. scleromalacia perforans will response well to pulse
dose methyl prednisolone and cyclophosphamide
39
8. Regarding orbital trauma:
a. orbital blow-out fractures result when the bony orbit is
injured by an object characteristically smaller than the
orbital rim
b. orbital emphysema is a sign of fracture of a sinus wall
and is frequently following ethmoid fractures
c. naso-orbital fractures often affect the medal canthal
tendon
d. visual loss can be a result of posterior ciliary arterial
damage
e. an afferent pupillary defect and visual loss immediately
following surgery should be treated with systemic
steroids
9. Concerning lesion of the optic disc:
a. 40% of patients with central congenital optic disc pits
develop serous detachment of the macula
b. optic disc drusen are situated in the retrolaminar zone
of the optic nerve
c. optic disc drusen are often associated with anomalous
configuration of the disc vessels
d. in optic disc melanocytomas the visual field defect is
determined by the size of the lesion
e. primary optic atrophy is characterized by marked
proliferation of glial and fibrous connective tissue on
the optic disc
10. The following are true:
a. in unilateral cases of optic nerve hypoplasia there may
be a disparity in the size of the optic foramina
b. optic nerve glioma in adults usually presents like an
acute optic neuritis
c. the temporal field defect in tilted disc can be reduced
by the appropriate refractive correction
d. myelinated nerve fibres can be present at birth
e. congenital disc coloboma can be associated with
absent septum pellucidum
Past MRCOphth/MRCS questions: 20
40
1. In fractures of the orbit:
a. pure blow out fractures of the floor are commonest
b. trauma to the infraorbital nerve causes anaesthesia to
the lips and gum
c. enophthalmos on initial examination indicates a large
fracture fo the floor
d. early surgical intervention is indicated in the presence
of diplopia and infraorbital anaesthesia
e. marked orbital and lid swelling is common following
repair of blow-out fractures
2. In papilloedema:
a. axonal distension occurs as the first sign
b. it takes 24-48 hours for early papilloedema to occur
and one week for it to develop fully
c. is usually asymmetric and greater on the side of a
supratentorial lesion
d. a fully developed papilloedema fully resolves in four
weeks with adequate treatment
e. it can be associated with sudden visual loss after
sudden decrease in systolic perfusion pressure
3. Regarding blow out fractures:
a. forced duction test is useful in differentiating tissue
prolapse from muscle shortening
b. the inferior oblique is more prone to nerve paresis than
the inferior rectus
c. surgery should only be performed after documentation
that there is no further worsening of ocular motility
d. mydriasis is more likely to be seen in medial wall
fractures than floor fractures
e. enophthalmos due to muscle fibrosis and shortening is
difficult to correct
4. Retinoblastoma:
a. is caused by a genetic abnormality on chromosome
14q13
b. is inherited with a visible chromosomal defect in most
cases
c. is inherited in an autosomal dominant fashion
d. is associated with esterase D deficiency in most case
e. is due to somatic mutation in most cases
41
5. Retinoblastoma tends to present with:
a. leukocoria
b. squint
c. microphthalmia
d. cataract
e. a painful red eye
6. Risk factors for metastatic retinoblastoma are:
a. a short optic nerve stump at enucleation ie. less
than 5 mm
b. a large tumour size
c. optic nerve invasion of any degree
d. choroidal invasion of any degree
e. anterior segment involvement
7. Coat's disease:
a. tends to occur in females
b. usually presents before the age of four years
c. is usually bilateral
d. is inherited as an autosomal dominant fashion
e. is indistinguishable from retinoblastoma on
ultrasonography
8. Birdshot choroidopathy:
a. tends to be unilateral
b. is associated with HLA A-29 antigen
c. is associated with cystoid macular oedema
d. can be treated with steroids
e. rarely occurs after the age of 50
9. Acute multifocal placoid pigment epitheliopathy:
a. usually has a good visual prognosis
b. usually occurs in elderly individuals over the age of 50
c. may be unilateral or bilateral
d. can be associated with cerebral vasculitis
e. can cause a serious retinal detachment
42
10. Sympathetic ophthalmia:
a. is best prevented by early enucleation of the
traumatized eye
b. should be treated by enucleation of the exciting eye
c. usually develops between 2 weeks and 3 months in
most cases
d. causes multiple pinpoint hyperfluorescence spots in
angiography
e. can be prevented by prophylactic steroids
Past MRCOphth/MRCS questions: 21
1. Retinitis pigmentosa:
a. results in a n abnormal EOG with a normal ERG
b. can be regional or diffuse
c. is most severe in autosomal dominant disease
d. can result in early visual loss from macular oedema
e. can be associated with deafness
2. Sulphur hexafluoride:
a. is used to tamponade a retinal break
b. has a longer half life than perfluoropropane
c. has a lower lower surface tension than silicone
d. expands to about five times its volume in 24 hours
e. can cause cataract
3. A circumferential buckle:
a. is indicated if there are multiple breaks in several
quadrants
b. may cause fishmouthing of a retinal break
c. is preferably used in combination with a radial buckle
d. tends to cause less volume displacement than a radial
buckle
e. can be supported by lamellar scleral flaps instead of
sutures
4. Drainage of subretinal fluid:
43
a. is indicated if there is significant vitreoretinal traction
b. should be performed as close to the retinal hole as
possible
c. can help to localize the retinal hole
d. is useful for softening the eye
e. should be performed before buckling
5. With iris tumours, risk factors for tumour growth are:
a. tumour vasculairty
b. medial location
c. ectropion
d. extension to angle
e. glaucoma
6. Uveal melanoma:
a. rarely occurs in children
b. is commoner in whites than blacks
c. can cause dilatation of the episcleral vessels
d. tends to metastasize to the liver
e. tends to show optic nerve invasion
7. Subretinal nevoascularization:
a. can occur in choroidal melanomas
b. occurs in retinitis pigmentosa
c. is successfully treated by laser in most cases
d. is stimulated by vasoproliferative factors produced in
response to choroidal ischaemia
e. results in serous detachment, hard exudates,
haemorrhage and degeneration of the photoreceptors
8. After penetrating ocular trauma:
a. all prolapsed uveal tissue should be excised to
prevent sympathetic opthalmia
b. scleral wounds extending posterior to the rectus
muscle insertions have worse prognosis than
corneoscleral laceration ending anterior to the rectus
c. epithelial downgrowth is a complication
d. phacoanaphylactic glaucoma is a complication if the
lens capsule is damaged
e. all intraocular foreign body should be removed.
44
9. Conditions associated with poliosis include:
a. Behcet's disease
b. Vogt-Koyanagi-Harada syndrome
c. sympathetic ophthalmia
d. leprosy
e. ocular albinism
10. Eyelid retraction occurs in:
a. facial nerve palsy
b. thyroid eye disease
c. use of iodipine
d. Parinaud's syndrome
e. tricyclic antidepressants
Past MRCOphth/MRCS questions: 22
1. Duane's syndrome:
a. is an autosomal dominant condition
b. is associated with aplasia of the sixth nerve nucleus
c. causes palpebral narrowing on abduction
d. can be classified according to the age of onset
e. is rarely bilateral
2. Down's syndrome is associated with:
a. posterior subcapsular cataract
b. glaucoma
c. keratoconus
d. peripheral retinal pigmentation
e. myopia
3. Regarding onchocerciasis:
a. the ocular lesions are due to direct invasion of the
cornea by the adult worm
b. the inflammation is reduced when the nematode dies
c. deer and antelope act as animal reservoirs
45
d. diethydcarbamazine kills the adult worm
e. the larvae are transmitted to humans by the bites of the
male black fly (Simulium)
4. Loss of cornea sensation occurs in:
a. leprosy
b. tuberculosis
c. Bell's palsy
d. acoustic neuroma
e. Lyme's disease
5. Staphylococcal infections can cause the following ocular
disease:
a. angular blepharitis
b. toxin epithelial keratitis
c. dacryocystitis
d. haemorrhagic conjunctivitis
e. eczematoid blepharodermatitis
6. Regarding herpes virus infections:
a. conjunctival swapings for lipschutz bodies are typical
findings in herpes simplex virus
b. Giemsa staining of epithelial cells helps diagnosis of
herpes simplex virus
c. fluorescent antibody technique staining demonstrates
intracellular herpetic inclusion bodies
d. varicella zoster infection os the lids can be confirmed
by staining of Henderson Patterson bodies
c. idoxuridine drops are effective treatment for zoster
keratitis
7. In Terrien's marginal degeneration:
a. begin typically at 12 and 6 O'clock on the corneal
periphery
b. causes progressive ulceration spreading around the
cornea
c. the central wall is steep while the peripheral wall slopes
gradually
d. spontaneous perforation occurs rarely
46
e. annular lamellar corneoscleral grafting may be helpful
8. Map dot fingerprint dystrophy:
a. is the most common corneal anterior dystrophy
b. there is thickened corneal epithelial basement
membrane
c. there is fibrillar material between Bowman's layer and
epithelial basement membrane
d. recurrent erosions occur in the majority of these
patients
e. inheritance is autosomal recessive
9. The following pigmentations appear on the cornea:
a. Ferry's line occurs in descemets membrane due to
chalosis
b. Stockers' iron line occurs next to a filtering bleb
c. Hudson Stahli line occurs on lower 1/3 of the cornea in
elderly adults only
d. Kayser Fleishcer ring occurs in progressive
intraherpatic cholestasis of childhood
e. chrysiasis has gold deposition in he deep corneal
stroma
10. Brown's syndrome:
a. is caused by abnormality of the inferior oblique muscle
b. causes hypotropia of the affected eye
c. is associated with spinal abnormality
d. is more common in male than female
e. is bilateral in 10% of cases
Past MRCOphth/MRCS questions: 23
1. Visible enlarged corneal nerves can occur in:
a. keratoconus
b. ichthyosis
c. endocrine neurofibromatosis
d. Hansen's disease
47
e. Refsum's syndrome
2. Regarding corneal grafting:
a. congenital rubella is a contra-indication for taking graft
material
b. infant donor tissue is a good graft material
c. tissue culture medium can store donor corneas viably
for up to 3 months
d. glycerin preserved corneal tissue may be useful for
lamellar keratoplasty
e. cryopreservation allows corneal storage for up to a year
3. Pertaining to cranial nerves:
a. trochlear nerve paralysis may be idiopathic in 1/3
of cases
b. trochlear nerve paralysis is commonly caused by
diabetes mellitus
c. abducens nerve paralysis is frequently caused by
diabetes mellitus
d. the trochlear nerve is more frequently damaged by
neoplasms than the abducens nerve
e. trochlear nerve paralysis can be caused by intracranial
aneurysm
4. Temporal lobe tumours:
a. cause homonymous hemianopia
b. are associated with formed visual hallucinations
c. may present with psychomotor epilepsy
d. may cause Foster-Kennedy Syndrome
e. may cause ataxic nystagmus
5. Parinaud's syndrome consist of:
a. ataxia
b. vertical gaze paralysis
c. pupillary areflexia to light
d. optic atrophy
e. convergence weakness
6. The following may be causes of unilateral oculomotor palsy
48
with pupil involvement:
a. migraine
b. herpes zoster
c. myasthenia gravis
d. diabetes mellitus
e. posterior communicating artery aneurysm
7. Multiple sclerosis can cause:
a. diplopia
b. abduction nystagmus
c. internuclear ophthalmoplegia
d. unilateral optic disc oedema
e. lateral rectus saccadic paresis during versions
8. The following are complications of LASIK:
a. epithelial downgrowth
b. corneal perforation
c. reduced visual acuity
d. stromal opacity
e. reduced contrast sensitivity
9. The following conditions are associated with retinal
artery occlusion:
a. Behcet's disease
b. Wegener granulomatosis
c. pancreatitis
d. syphilis
e. sickle cell trait
10. The following are advantages of Moh's micrographic
excision of skin tumour:
a. tissue conservation
b. differentiating squamous cell carcinoma from basal
cell carcinoma
c. useful in the diagnosis of basal cell carcinoma
d. reduced incidence of tumour recurrence
e. faster tissue healing
49
Past MRCOphth/MRCS questions: 24
1. Chronic closed angle glaucoma:
a. can always be treated by cataract extraction alone
b. tropical medication is the first line of treatment
c. is common in Pekinese dogs
d. the prevalence of the disorder in indigenous people
varies with longitude
e. iridotomy alone is the adequate treatment in advanced
disease
2. Pseudoexfoliation syndrome:
a. is always eventually complicated by glaucoma
b. is associated with pseudoexfoliation in the orbit
c. the exfoliative material contains laminin
d. Sampaolesi's line is a characteristic feature
e. is common in France
3. Photodynamic therapy of subretinal neovascular membrane:
a. causes cataract
b. involves the injection of fluorescein at the time of
treatment
c. is not suitable for pigment epithelium detachment
d. is not suitable for foveal subretinal neovascular
membrane
e. is contraindicated in the presence of diabetic
retinopathy
4. Age related macular degeneration:
a. may be dominantly inherited
b. is commoner in dark skinned race
c. is usually a unilateral condition
d. may undergo spontaneous functional improvement
e. soft drusen have a better prognosis than that of hard
drusen
5. The following statements are true:
a. fundus flavimaculatus masks choroidal fluorescence
b. type B personalities are common in patients with
50
central serious retinopathy
c. different pattern dystrophies occur in the same family
d. Best's disease causes an enhanced EOG
e. central retinal artery occlusion abolishes the a wave of
the ERG
6. Cataract:
a. is a complication of lactose intolerance
b. may be a late sequel of severe dehydration
c. is reversible in diabetes
d. complicates blunt eye injuries
e. may be prevented by aspirin
7. The following complications are well-recognised:
a. retinopathy due to tamoxifen
b. cataract due to ibuprofen
c. cornea verticillata due to chloroquine
d. palinopsia due to chlorpromazine
e. optic atrophy due to ethambutol
8. In patients with visual field loss:
a. wedge scotomata result from lateral geniculate body
pathology
b. junctional scotoma and Foster-Kennedy syndrome
may occur in the same patient
c. macular sparing is the hallmark of occipital tumour
formation
d. arcuate scotomata can occur with chiasmal
compression
e. central scotoma complciate retrochiasmal dysfunction
9. Accommodation:
a. is impaired in Adie's syndrome
b. produces tension in the lens capsule
c. is impaired following traumatic hyphaema
d. is enhanced by beta blockers
e. is reduced by alpha agonists
10. The following drugs or techniques can be used to reduce
51
intraocular pressure:
a. hypoventilation
b. ketamine
c. intravenous sucrose
d. halothane
e. nitrous oxide
Past MRCOphth/MRCS questions: 25
1. Ocular manifestations and syndromes with recognized
associations are:
a. eyelid coloboma and Treacher-Collins
b. keratoconus and Down's syndrome
c. Convergent squint and Crouzon's syndrome
d. hypertelorism and Kleinfelter's
e. limbal dermoids and von Recklinghausen's disease
2. Keratoconus:
a. may become stationary at any time
b. onset is usually in the early twenties
c. the Fleischer ring consists of melanin pigments
d. hydrops is caused by endothelial decompensation
e. may be associated with retinitis pigmentosa
3. Aniridia:
a. is inherited as an autosomal recessive disease
b. rarely leads to glaucoma
c. often causes photophobia
d. is associated with nephroblastoma
e. mainly affects oculocutaneous albinos
4. Macular dystrophy is characterized by:
a. diffuse and rapidly progressive cloudiness
b. superficial lesions in central cornea
52
c. stromal lesion in peripheral cornea
d. early impairment of visual acuity
e. autosomal dominant inheritance
5. The following may be causes of unilateral oculomotor palsy
with pupil involvement:
a. migraine
b. herpes zoster
c. myasthenia gravis
d. diabetes mellitus
e. posterior communicating artery aneurysm
6. Juvenile retinoschisis:
a. is an autosomal recessive condition
b. there is classical cystoid oedema at the macula
c. the split is in the nerve fibre layer
d. is a bilateral disease
e. it is most commonly seen in the superior nasal quadrant
7. A circumferential buckle:
a. is indicated if there are multiple brakes in several quadrants
b. may cause fishmouthing of a retinal break
c. is preferably used in combination with a radial buckle
d. tends to cause less volume displacement than a radial buckle
e. can be supported by lamellar scleral flaps instead of sutures
8. Sulphur hexafluoride:
a. is used to tamponade a retinal break
b. has a longer half life than perfluoropropane
c. has a lower surface tension than silicone
d. expands to about five times its volume in 24 hours
e. can cause cataract
9. Retinitis pigmentosa:
a. results in abnormal EOG with a normal ERG
b. can be regional or diffuse
c. in most severe in autosomal dominant disease
53
d. can result in early visual loss from macular oedema
e. can be associated with deafness
10. The following topical medications are contraindicated in
post-cataract patients due to the risk of cystoid macular
oedema:
a. latanoprost
b. beta blocker
c. alpha blocker
d. acetazolamide
e. adrenaline
Past MCQs for Paper one
1. Genetic questions on the mode of inheritance of the following condition:
a. choroidaemia
b. congenital retinoschisis
c. xeroderma pigmentosa
d. pigmentosa incontentia
e. protanopia
2. Giant cell arteritis:
a. histology is based on fragmentation of the internal intimal
b. anterior cerebral artery is often involved
c. C-reactive protein is always raised
d. giant cell is needed for diagnosis
3. Staining techniques useful for diagnosis:
a. Gram negative diplococcus in neonatal conjunctivitis
b. ostium tetraoxide for electron microscopy
c. Haematoxylin and eosin for collagen and haemtoxylin stain collagen and eosin stain
mucopolysaccaride
4. Choroidaemia:
54
a. female carrier has abnormal retinal features
b. present in the second decade of life
c. ERG shows abnromality early in the disease
5. In congenital retinoschisis:
a. maculopathy is 50% of patient
b. peripheral retinopathy in 50%
c. involved retinal shows retinal elevation
d. female carrier develop fundal changes
6. Leber's optic neuropathy:
a. female carrier never affected
b. more than 75% occurs in male
c. optic disc is pale in the early stage
7. The following conditons are pre-malignant:
a. actinic keratosis
b. oncocytoma of the caruncle
c. squamous papilloma
8. Glaucoma due to steroid:
a. commonest cause of secondary glaucoma in the Middle East
b. commoner in type 1 diabetic mellitus
c. is seen in 15% of the population
d. progressive damage is prevented on stopping steroid
e. manifest as low tension glaucoma
9. Pigment dispersion syndrome:
a. raised intraocular pressure with mydriasis
b. complete escape from beta blocker after a few months
c. peripheral transillumination
d. commoner in diabetic melliuts
10. Iris melanoma:
a. infiltrate corneal stroma
b. infiltrate trabecular meshwork
c. invade lens cortex
d. heterochromia is a feature
11. Follwoing biopsy report may be found with rheumatoid arthritis:
a. posterior scleritis
b. loss of goblet cells in the conjunctiva
55
c. episcleral necrotic tissue
12. Panphotocoagulation in dibaetic mellitus is indicated for:
a. preprolifertive
b. ischameic maculopathy
c. new vessels at the disc
d. retinal cottol wool spots
13. Hypoxic damage in diabetic mellitus is suggested by:
a. nerve fibre layers haemorrhage
b. iris neovascularization
c. anterior uveitis
d. decrease jard exidate
e. vascular loop
14. Angioid streaks are seen in:
a. sickle cell anaemia (HbSS)
b. myeloma
c. leukaemia
d. Ehler-Danlos syndrome
15. Retinal detachment:
a. SF4 lasts longer than C3F8
16. Inantile glaucoma is associated with:
a. axial iris stromal abnormality
b. aniridia
c. Sturge-Weber syndrome
17. Albinism:
a. hair bulb for tyrosin positivity test is useful when the child is 5 year old
b. is associated with platelets abnormality especially with aspirin
c. complete decussation is invariable in all albinism
d. is associated with thrombosis under general anaesthesia
18. Major hyphaema:
a. raised intraocular pressure in the first 24 hours and then normalize
b. 20% rebleed
56
c. less than 50% have angle recession
d. corneal staining is irreversible
19. Optic nerve avulsion:
a. seen in 1% of baby with high forcep delivery
b. 20% of patients with frontal lacrimal bone fracture
c. in dense vitreous haemorrhage ERG is useful for diagnosis
20. Optic nerve glioma:
a. is assoicated with neurfibromatosis type I
b. increased incidence after 30 year old
c. majorty is of pilocytic astrocytoma
d. rarely infiltrate perineural tissue
21. Nystamus:
a. manifest latent nystagmus in unilateral congenital cataract
b. see-saw nystamus associated with homonymous hemianopia
c. down-beat with foramen magnum lesion
d. gaze-evoked is a specific sign
22. Vernal keratoconjunctivitis:
a. usually unilateral
b. eosinophil is common in the paillae
c. can be of limbal type alone
23. Audit:
a. collect data against activity
b. penalised health profession
c. improve quality of care
24. Mutton-fat keratic precipitate are seen in:
a. uveal effusion syndrome
b. tuberculosis
c. Fuch's heterochromia cyclitis
d. sarcoidosis
57
25. Medications for glalucoma:
a. beta blocker is contraindicated in primary heart block
b. beta-blocker is contraindicated in those on nifedipine
c. acetazolamide is contraindicated in allergy to sulphonamide
d. acetazolamdie needs potassium supplement
26. Gout:
a. purine metabolism disorder
b. high incidence of scleritis
c. tophi found in extraocular muscle tendon
d. related to alcohol consumption directly
d. related to alcohol consumption directly
e. crystal found in deep stroma
27. Corneal dystrophy may contain the following substance:
a. mucopolysaccharides
b. cholesterol
c. amylodiosis
d. uroprohyrin
e. urate
28. Seen in vitreous:
a. amyloid
b. lipofusin
c. haemosiderin
d. clacium
29. Melanocytoma:
a. associated with clear cell carcinoma of the kidney
b. contain plump black naevus cells
c. metastasize to the lungs
30. Lacrimal gland tumour:
ametastasize along the orbital nerve
b. commonly metastasize throught eh lymphatic drainage
c. early metasis to the lung
d. Merkel cell carcinoma can occur
31. Retinoblastoma:
58
a. 50% inherited
b. due to 11q14 deletion
c. rosettes suggest good prognosis
Past MCQs for Paper two
1. Superior oblique palsy:
a. 25% congenital
b. bilateral causes has V esotropia
c. bilateral has more than 10 degrees of excyclotorsion
d. hypertropia increases on gaze to opposite side
2. The following are true about inferior oblique muscle operion:
a. ipsilateral myectomy is performed for fourth nerve palsy
b. myectomy is more effective than recession
c. haematoma is common
3. Ciliary body melanoma:
a. has a better prognosis than choroidal melanoma
b. may be amelanotic
c. more radiosensitive than choridal melanoma
4. Cataraacts are asscoaited with:
a. Lowe's syndrome
b. Usher's syndrome
c. Wilson's syndrome
d. Klipper-Field's syndrome
e. pseudoxanthoma elasticum
5. Syphilis:
a. teritary syphilis causes inflammation of zonules, lens and retina
b. commonest cause of interstitial keratitis
59
6. High AC/A ratio:
a. phosphholine iodide decreases it
b. increase iwth age
c. rrequires fixed working distance for measurement
d. can be corrected by executivd bifocal
e. congenital esotropia is associated with high ratio
7. Laser trabeculoplasty:
a. is successful in 80% of cases
b. may induce cataract
c. can reduce the pressure in the other eye
d. can increase field loss
e. casues fall in pressure in 2 hours
8. Acute mullfifocal placoid punctaate epitheliopathy:
a. is usually unilateral
b. on fluorrescein angiography, masking is seen in the initial stages
c. is associated iwth increased calcium concentration in the serum
d. improves spontaneouslly
9. Malignant melanoma of chorid have a worse prognosis with:
a. patients older than 70 years old
b. icnreaseing size
c. scleral extension
d. anergy to BCG
e. loss of humoural antiobodies with tumour
10.In optic nerve trauma:
a. intracranial part is most commonly affected
b. optic atrophy is common
11. Dissociated vertical deviation:
a. commonly bilateral and asymmetrical
b. the covered eye goes up and intorts and rapidly comes down on removal of cover
c. associated with latent nystagmus
d. assocaited with heterotropia
60
12. Optic disc ppit:
a. associated iwth retinal serous dtachment in 10% of cases
b. inferonasal typical but can be anywhere
13. SF6:
a. is highly lipid soluble
b. does not affect ERG
c. causes posterior subcapsular cataract
14. Idiopathic macular hole:
a. commoner in females
b. may close spontaneously
c. 90% biolateral
d. can occur with posterior vitreous detachment
15. Advantages of MRI over CT scan:
a. can be used in pacemaker
b.can be used to identify bony lesion
c. less radiatioon
d. all axis without moving patient
16. Retinal breeeeaks due to blunt trauma:
a. usually forms 7 days later
b. pigment epithleial proliferaiton 10 days post injury
. frequently associated with large oval breaks in quadrant opposite to impact
d. anterior dialysis is commonlly superonasal quadrant
e. arise due to compression of vitreous face
17. In rod monochromatism:
a. cones are totally absent
b. ERG at 30Hz is flt
c. dark adapted eye will have a cone response
18. Retinoschsis:
a. is commonly bilateral
b. is nearly always present in nasl quadrnat
61
c. juvenile type is frequently associated with macular schisis
d. split present in outer plexiform layer
e. inner layer breaks require treatment
19. Wernicke's encephalopathy:
a. large doses of vitamin B12 is useful
b. patients develop long-standing epilepsy
c. lesion is in the wall of the fourth ventricle
d. only occur in the alcoholic
e. puillary reflexes and extraocular muscles are affected
20. Primary hyperplastic persistent vitreous:
a. unilateral in otherwise healthy child
b. presents in neonatal period
c. causes cataract
d. causes angle closure glaucoma
e. duje to persistence of secondary and tertiary vitreous
21. Posterior inferior cerebellary artery thrombosis:
a. causes infarction of lateral medulla
b. painful trigeminal area on the same side
c. Horner's syndorme
22. Spasmus nutans:
a. fine jerky nystagmus usually monocular
b. rarely resolves spontaneously
c. present between 2 and 12 months of age
23. Aniridia may be associated with:
a. short arm of chromosome 11 deletion
b. naterior polar cataract
c. pannus
c. hypernephroma
c. optic disc hypoplasia
62
24. Damage to the optic nerve from head injury:
a. is usually in the intracranial part
b. result in optic atrophy nasally
c. usually causes an afferent pupillary defect in the other eye
d. cuases an altitudinal field defect
25. V phenomenon occurs in:
a. superior rectus weakness
b. superior oblqieu wekaness
c. too high insertion of medial wall
d. may be treated by sagitalisation of inferior oblique in appropriate cases
e. is always pathological
26. In severe diabetic retinopathy:
a. absence of cotton wool spots
b. increased foveal avascular zone
Past MCQs from FRCS (Ed) 2000
1. Corneal epithelium
a. Is ten cells thick
b. Has microplicae
c. Has microvilli
d. Has zonulae occludens
e.Hemidesmosomes attach to basal layer
2. Regarding anatomy of sac region
a.MPL is attached to posterior lacrimal crest
b.Angular vein is superficial to MPL
c.MPL is the tendon of orbicularis oculi
3. Myasthenia gravis
a.20% present before 20 years of age
63
b.90% has anti acetyl choline antibody
4. Regarding SLE
a.Commonest manifestation is SLE retinopathy
5. Retinoblastoma
a.More common in boys
b.Has deletion of 13q14
c.Bilateral is always hereditary
d.Endophytic has better prognosis
6. Diabetic retinopathy
a.Endothelial proliferation is characteristic
b.Basement membrane thickening is a late feature
c.Pericyte is reduced
7. Behcet's disease
8. Angiod streak associations
a.Acromegaly
b.Sickle cell anemia
c.Pseudo xantoma elasticum
d.Erhler Danlos
9. Migraine
a.Aura is due to vasospasm and cortical ischemia
b.Classical migraine can occur with aura and no headache
10. Adeno viruses
a.Incubation period of Adeno virus is two weeks
11.Common organisms causing ophthalmia neonatorum are
a.Chlamydiae
64
b.Hemophilus
c.Pneumococcus
d.Herpes virus type 2
e.Streptococcus
12. Acanthamoeba keratitis
a.Is common in immunocompromised
b.Disinfection doesnot clear amoeba from contact lens
c.Perineural infiltrates characteristic
13. Thyroid ophthalmopathy
14. Sympathetic ophthalmia
15.Ankylosing Spondylitis
a.90% of patients are positive for HLA B 27
b.15% have acute iritis
16.Contra-indication for eye donations
a.Acute conjunctivitis
b.Creutzfeld Jacob disease
17.PRK
a.Is an established technique for myopia
18.Lyme disease
a.Caused by Borrelia burgdoferi
b.Tetracycline is used for treatment
c.Erythema chronican migrans can occur
18.Toxoplasmosis
20.In ultra sound B-scan RD vs PVD
a.Attached to optic nerve in most sections indicate RD
65
b.Mobility is more with PVD
c.After movement is suggestive of PVD
21.Ocular Albinism
a.X-linked inheritance
b.90% crossing in chiasma
c.Bruising is exaggerated with aspirin
d.Fovea may not be formed
22.Treatment of Superior oblique palsy
a.Ipsilateral IR recession
23.MRI
a.Doesnot image bone
b.Calcification is better seen with MRI
24.Cataract
a.Christmas tree cataract is seen with myotonic dystroph
25. Homocystinuria
a.Lens dislocation can occur in any direction
26. Wegener's granulomatosis
27. Progressive supranuclear palsy
a.Can be caused by degenerative condition
b.Can be caused by pinealoma
28. Pituitary adenoma
a.Bromocriptine can shrink prolactinoma in 24 hrs.
b.Acromegaly field defect can be true/false
66
Neuroophthalmology
1. The following are true about isolated fourth (trochlear nerve)
palsy:
a. head trauma is the most common cause in children
b. aneurysm is a common cause in adult
c. the head is usually tilted away from the palsied side
d. congenital form tends to have a large vertical fusional range
e. vertical prism is useful in correcting the torsional diplopia
1.
a.F
b.F
c.T
d.T
e.F
2. The following signs are useful in distinguishing between the
congenital and acquired form of Horner's syndrome:
a. absent light reflex of the affected eye
b. lighter iris colour of the affected eye
c. decreased facial sweating of the affected side
d. dilatation of the affected pupil with 10% cocaine
e. dilatation of the affected pupil in response to 1% adrenaline
2.
a.F
b.T
c.F
d.F
e.F
3. Palinopsia occurs in:
a. use of hallucinogens
b. parietal lobe lesion
c. occipital lobe lesion
67
d. frontal lobe lesion
e. migraine
3.
a.T
b.T
c.T
d.F
e.T
4. In spasmus nutans:
a. the age of onset is usually between 4 and 18 months of age
b. the nystagmus is classically horizontal
c. the condition does not requires further investigation
d. the eye movement is symmetrical
e. spontaneous resolution commonly occurs
4.
a.T
b.F
c.F
d.F
e.T
5. Downbeat nystagmus occurs in:
a. lithium toxicity
b. alcohol intake
c. Arnold-Chiari's syndrome
d. middle ear diseases
e. spinocerebellar degeneration
5.
a.T
b.T
c.T
d.F
e.T
6. Bitemporal hemianopia may be seen in:
a. tilted disc
b. bilateral ischaemic optic neuropathy
c. dermatochalasia
68
d. sectorial retinitis pigmentosa
e. retinoschisis
6.
a.T
b.F
c.T
d.T
e.T
7. Vertical gaze is impaired in the following conditions:
a. internuclear ophthalmoplegia
b. walled-eye syndrome
c. juvenile Gaucher disease
d. progressive supranuclear palsy
e. Sylvian aqueduct syndrome
7.
a.F
b.F
c.T
d.T
e.T
8. The following findings make benign intracranial hypertension
unlikely:
a. bilateral sixth nerve palsy
b. morning headache and nausea
c. increased protein in the cerebrospinal fluid
d. enlarged ventricle on brain scan
e. hard exudate in the macula
8.
a.F
b.F
c.T
d.T
e.F
9. Optic nerve hypoplasia is associated with:
a. maternal diabetes mellitus
b. maternal alcoholism
c. hypopituitarism
69
d. maternal use of phenytoin
e. maternal folate deficiency
9.
a.T
b.T
c.T
d.T
e.F
10. In a patient with bilateral centrocecal scotoma, the following
tests are useful in diagnosis:
a. full blood count
b. serum folate level
c. liver function tests
d. serum copper level
e. ESR
10.
a.T
b.F
c.T
d.F
e.F
11. In non-arteritic ischaemic optic neuropathy:
a. optic disc oedema is common
b. embolism is the cause in the majority of cases
c. visual loss is usually more severe than arteritic ischaemic optic
neuropathy
d. a small cup-to-disc ratio is a risk factor
e. optic nerve fenestration is useful in improving final visual outcome
11.
a.T
b.F
c.F
d.T
e.F
12. In contrast to surgical third nerve palsy, medical third nerve
palsy:
a. does not affect pupillary response to light
b. does not affect accommodation
70
c. does not cause upper lid elevation on attempted downgaze
d. does not cause complete ptosis
e. causes a decreased in intraocular pressure
12.
a.T
b.T
c.T
d.F
e.F
13. In optic neuritis:
a. there is usually progression over the first three weeks from onset
b. pain on ocular movement is usually worse on adduction
c. the central scotoma may worsen with raised in body temperature
d. systemic steroid has been shown to affect the final visual outcome
e. a swollen disc make the diagnosis unlikely
13.
a.F
b.T
c.T
d.F
e.F
14. In myasthenia gravis:
a. the horizontal recti are more commonly affected than the vertical
recti
b. there is a delayed tendon reflex
c. the pupillary response to light is not affected
d. steroid may worsen the muscular problems
e. neonates born to mother with the disease may be affected due to
the anticholinesterase receptor antibodies closing the placenta
14.
a.F
b.F
c.T
d.T
e.T
15. The following are true:
a. opticokinetic drum can be used to differentiate a lesion in the parietal
lobe from that of a occipital lobe.
71
b. cerebral dyschromatopsia is associated with bilateral superior
homonymous hemianopia.
c. postfixation visual loss in bitemporal hemianopia
d. afferent pupillary defect and homonymous hemianopia in lesion of
the optic tract
e. sensory inattention is associated with left inferior homonymous
quadrinopia in a right handed person
15.
a.T
b.T
c.T
d.T
e.T
16. In Parinaud's syndrome:
a. the lesion is in the dorsal mid-brain
b. light-near dissociation is a feature
c. accommodative spasm occurs on attempted upgaze
d. convergence-retraction nystagmus occurs with vertical saccades
e. demyelination is a known cause
16.
a.T
b.T
c.T
d.T
e.T
17. The following are true:
a. Uhtoff's symptom refers to a decrease in vision with an increase in
body temperature
b. prosopagnosia occurs with a bilateral medial occipitotemporal lesion
c. in Riddoch phenomenon, the patient can perceive objects in motion
but not stationary objects.
d. skew deviation occurs in lesion of the supraneuclear vertical gaze
pathway
e. Anton's syndrome is associated with denial of blindness
72
17.
a.T
b.T
c.T
T
d.T
e.T
18. In Benedikt's syndrome:
a. third nerve palsy is present
b. there is a contralateral weakness
c. there is a contralateral tremor
d. there is contralateral sensory loss
e. the facial nerve is involved.
18.
a.T
b.F
c.T
d.T
e.F
19. In Leber's optic neuropathy:
a. DNA analysis is useful
b. the offsprings of an affected man have a 50% chance of
acquiring the disease
c. in the acute phase a macular star is typically seen
d. the optic disc is hyperaemic with telangiectatic capillaries
e. fluorescein angiography in the acute phase shows leakage in
the optic disc
19.
a.T
b.F
c.F
d.T
e.F
20. In thyroid eye disease:
a. the medial and inferior recti are the most commonly affected
extraocular muscles.
b. smoker is more likely to be affected than non-smoker
c. optic nerve fenestration is useful in patients with optic nerve
compression
73
d. upgaze is most commonly affected in patients with ocular
motility problems
e. the thyroid function test always shows hyperthyroidism
20.
a.T
b.T
c.F
d.T
e.F
21. In Argyll-Robertson's pupils:
a. the condition occurs in acquired but not congenital syphilis
b. the pupils dilate poorly with atropine
c. the irregularity of the pupils are caused by iritis
d. the pupils dilate with topical cocaine
e. the lesion is in the rostral area of the Edinger-Westphal nucleus.
21.
a.F
b.T
c.T
d.F
e.T
22. In Adie's (tonic) pupil:
a. the pupil may not respond to light or accommodation in the acute
stage
b. Guillain-Barré syndrome is a known cause
c. the pupil constricts with 2.5% mecholyl solution
d. there is usually a permanent loss of accommodation
e. the condition may be confused with Argyll-Robertson's pupil.
22.
a.T
b.T
c.T
d.F
e.T
23. In a dilated pupil:
a. a light-near dissociation is seen with third nerve palsy
b. failure of constriction to 1% pilocarpine is seen in third nerve palsy
74
c. failure of constriction to 1% pilocarpine occurs with mydriasis
induced by atropine
d. problem with accommodation is a useful sign in distinguishing
mydriasis induced by atropine from that caused by third nerve
palsy
e. consensual light reflex of the normal eye is absent in third nerve
palsy
23.
a.F
b.F
c.T
d.F
e.F
24. The following are true:
a. Cogan's rule: a patient with homonymous hemianopia but otherwise
neurologically intact is caused by a lesion in the occipital lobe
b. Wernicke's hemianopic pupillary response: occurs in optic tract field
defect
c. Gertman's syndrome: lesion in the non-dominant parietal lobe
d. Marcus Gunn's pupil: diagnostic of optic neuritis
e. Foster-Kennedy's syndrome: lesion in the optic tract
24.
a.T
b.T
c.F
d.F
e.F
25. The following are true about visual field defects:
a. macular sparing is commoner with an anterior rather than a
posterior lesion of the calcarine cortex
b. horizontal sectoranopia is a feature of lateral geniculate body
lesion
c. an unilateral field loss rules out a lesion in the post-chiasmal region
d. a homonymous hemianopia with temporal sparing is seen in lesion
of the posterior calcarine cortex
75
e. altitudinal defect is a feature of glaucoma
25.
a.T
b.T
c.F
d.T
e.F
26. In a left optic tract lesion:
a. a right afferent pupillary defect may be present
b. the visual field shows congruous homonymous hemianopia
c. the left optic disc may show temporal pallor
d. the right optic disc may show nasal pallor
e. the lesion may result from a pituitary adenoma
26.
a.T
b.F
c.T
d.T
e.T
27. Optic nerve gliomas:
a. usually occur in the third decade of life
b. proptosis is the most common presentation
c. are usually bilateral
d. cause death through cerebral invasion
e. require early surgical excision
27.
a.F
b.F
c.F
d.F
e.F
28. Tuberous sclerosis has the following features:
a. mental retardation
b. ashleaf lesion best seen with Wood's light
c. mulberry tumours which contain calcification
d. presence of café au lait patches
76
e. over 90% of the cases have a positive family history
28.
a.T
b.T
c.T
d.T
e.F
29. In visual evoked potential (VEP):
a. the latency is prolonged in recovered optic neuritis
b. the amplitude is normal in recovered optic neuritis
c. it is essentially a test of the macula
d. the result is abnormal in patients with ocular albinism
e. it can be used to detect malingering
29.
a.T
b.T
c.T
d.T
e.T
30. In type 1 neurofibromatosis, the following signs may be seen:
a. pulsatile proptosis
b. bilateral acoustic neuroma
c. Lisch's nodules
d. cerebral meningioma
e. cavernous haemangioma of the retina
30.
a.T
b.F
c.T
d.T
e.F
Glaucoma
1. The following visual field defects are typically seen in primary open angle
glaucoma:
a. centrocecal scotoma
77
b. nasal step
c. central scotoma
d. paracentral scotoma
e. altitudinal field loss
1.
a.F
b.T
c.F
d.T
e.F
2. Early signs of glaucomatous damage include:
a. splinter haemorrhage of the optic disc
b. glaucoflecken
c. vertical elongation of the cup
d. loss nerve fibre layer
e. peripapillary atrophy
2.
a.T
b.F
c.T
d.T
e.F
3. Static perimetry include:
a. Lister perimeter
b. Goldmann perimeter
c. tangent screen
d. Octopus
e. Humphrey
3.
a.T
b.T
c.T
d.F
e.F
4. The following are direct gonio lenses:
a. Koeppe
b. Goldmann
c. Thorpe
d. Zeiss
78
e. Barkan
4. a.T
b.F
c.T
d.F
e.T
5. In pigmentary dispersion syndrome:
a. the pigments is caused by the release of melanin from
the anterior surface of the iris
b. the incidence of affected male to female is 5:1
c. peripupillary retroillumination is a feature
d. heterochromia can occur
e. latanoprost should be avoided as it increases the release
of pigments
5.
a.F
b.T
c.F
d.T
e.F
6. Blood in Schlemm's canal is seen in:
a. Sturge-Weber's syndrome
b. carotid-cavernous fistula
c. angle recession glaucoma
d. hypotony
e. neurofibromatosis
6.
a.T
b.T c.F
d.T
e.F
7. The following are true about lens induced glaucoma:
a. laser iridotomy is ineffective against pharcomorphic
glaucoma
b. phacolytic glaucoma is characterized by blockage of the
trabecular meshwork by protein laden marcophages
c. phacolytic glaucoma typically occurs following cataract
extraction
79
d. phacolytic glaucoma is associated with Morgagnian
cataract
e. phacoanaphylactic glaucoma is associated with
granulomatous inflammation
7.
a.F
b.T
c.F
d.T
e.T
8. In a patient with angle recession glaucoma, gonioscopy may reveal:
a. separation of the ciliary body from its attachment to the
scleral spur
b. increased visibility of the scleral spur
c. torn iris process
d. marked variation in the ciliary body width in different
quadrant of the same eye
e. new vessels at the angle
8.
a.F
b.T
c.T
d.T
e.F
9. In primary congenital glaucoma:
a. the incidence is 1:12,500 births
b. the corneal diameter is usually greater than 8 mm
c. gonioscopy reveals iris hypoplasia
d. the cup: disc ratio is greater than 0.3
e. males are more commonly affected than females
9.
a.T
b.F
c.F
d.T
e.T
10. Sampaolesi's line:
a. is a pigmented line anterior to Schwalbe's line
b. occurs in pseudoexfoliation syndrome
c. is seen in pigment dispersion syndrome
80
d. occurs in iris melanoma
e. is seen in siderosis bulbi
10.
a.T
b.T
c.T
d.T
e.F
11. In uveitic glaucoma:
a. pilocarpine is the treatment of choice in patient who can
not take beta blockers
b. girls with pauciarticular juvenile rheumatoid arthritis are
more likely to develop glaucoma than their male
counterpart
c. simple trabeculectomy is associated with high failure
rate
d. iridocyclitis is often associated with low rather than high
intraocular pressure
e. iris bombe is a common complication of
Posner-Schlossman syndrome
11.
a.F
b.T
c.T
d.T
e.F
12. In Humphrey automated perimeter, the following definition are true:
a. a false-negative response: failure to respond to a
suprathreshold stimulus at a location that the patient
would be expected to see
b. fixation loss: occurs when the patient responds to a
target shown at his blind spot
c. high positive rate is seen in trigger happy patient
d. short term variation is greater in glaucomatous than
normal population
e. short term variation refers to change in sensitivity when
the same area is retested.
12.
a.T
b.T
c.T
d.T
e.T
81
13. The following glaucoma are associated with bilateral primary abnormalities
of the corneal endothelium:
a. Axenfeld's anomaly
b. Peter's anomaly
c. Cogan-Reese syndrome
d. posterior polymorphous membraneous dystrophy
e. essential iris atrophy
13.
a.F
b.F
c.F
d.T
e.F
14. True statements about topical beta blockers include:
a. 10% of patients do not show therapeutic response
b. therapeutic response is reduced in patients on systemic
beta-blockers use
c. the additive effect on glaucoma is increased if it is
combined with topical adrenaline than with topical
acetazolamide
d. betaxolol is more effective than timolol in lowering
intraocular pressure
e. bradycardia is less with carteolol than timolol
14.
a.T
b.T
c.F
d.F
e.T
15. The risk of developing steroid-induced glaucoma is increased in the following
conditions:
a. myopes
b. patients with a history of primary open angle glaucoma
c. history of angle closure glaucoma
d. diabetes mellitus
e. posterior synechiae
82
15.
a.T
b.T
c.F
d.T
e.F
16. The following are true about laser peripheral iridotomies:
a. Nd:YAG laser requires fewer application compared
with argon laser
b. bleeding is less common with argon than Nd:YAG laser
c. late closure of the iridotomies is commoner with argon
than Nd:YAG laser
d. iris oedema is commoner with argon than Nd:YAG
laser
e. pupillary distortion is commoner with argon than
Nd:YAG laser
16.
a.T
b.T
c.T
d.T
e.T
17. True statements about systemic acetazolamide include:
a. it should be avoided in patients allergic to sulphonamides
b. it is associated with metabolic acidosis
c. it is associated with renal calculi
d. it causes transient myopia
e. it should be avoided in patient with hepatic
encephalopathy
17.
a.T
b.T
c.T
d.T
e.T
18. The following are true about aqueous drainage devices:
a. all contain a silicone tube attached to a synthetic plate
b. corneal decompensation is a known complication
c. Molteno's tube contains pressure sensitive device
d. post-operative hypotony is commoner with Ahmed's
valve than Molteno's tube
83
e. Krupin's valve contains pressure sensitive valve
18.
a.T
b.T
c.F
d.F e.T
19. Regarding anti-metabolites:
a. they are applied to the scleral flap during trabeculectomy
b. they increase the incidence of post-operative
endophthalmitis
c. they inhibit DNA synthesis and therefore cellular
proliferation
d. 5-fluorouracil is more likely to cause hypotony than
mitomycin C
e. mitomycin C is associated with a higher incidence of
wound leak than 5-fluorouracil
19.
a.F
b.T
c.T
d.F
e.T
20. Apraclonidine:
a. is an alpha 2 agonist
b. causes conjunctival hyperaemia
c. causes increased uveoscleral outflow
d. produces miosis
e. causes lid retraction
20.
a.T
b.F
c.F
d.F
e.T
Test 11 (paediatric ophthalmology)
1. Causes of poor vision in aniridia include:
a. macular hypoplasia
84
b. glaucoma
c. cataract
d. retinitis pigmentosa
e. optic nerve hypoplasia
a.T b.T c.T d.F e.T
2. True statements about aniridia include:
a. it is sporadic in one third of cases
b. familial cases are autosomal dominant
c. vision can be improved with tinted glasses
d. Wilm's tumour occurs in one third of sporadic aniridia
e. Wilm's tumour does not occur in familial aniridia
a.F b.F c.T d.T e.F
3. The following features are seen in Leber's congenital amaurosis:
a. nystagmus
b. normal fundus at the early stage
c. myopic refraction
d. absent pupil responses to light
e. arteriolar narrowing in later stage
a.T b.T c.F d.T e.T
4. Chickenpox in expectant mother can produce the following
ocular abnormalities in the newborn:
a. Horner's syndrome
85
b. optic nerve hypoplasia
c. aniridia
d. chorioretinitis
e. cataract
a.T b.T c.F d.T e.T
5. In de Morsier's syndrome:
a. the condition is hereditary in the majority of cases
b. optic atrophy is a feature
c. agenesis of the septum pellucidum is a feature
d. the corpus callosum may be malformed.
e. hypothalamic dysfunction is common
a.F b.F c.T d.T e.T
Lens and cataract
1. The following are true about cataract:
a. it is the most common cause of blindness in the world
b. nuclear sclerosis reduces the distant vision more than
near
c. posterior subcapsular cataract is more likely to cause
problem with distant vision than near
d. more females than males have cataract surgery
e. outdoor workers are more likely to develop cataract
than indoor workers
1.
a.T
b.T c.F d.T e.F
2. True association include:
86
a. Wilson's disease - sun-flower cataract
b. myotonic dystrophy - Christmas tree cataract
c. amiodarone - anterior capsular cataract
d. Alport's syndrome - posterior lenticonus
e. Down's syndrome - cerulean (blue-dot) cataract
2.
a.T b.T c.T
d.F e.T
3. The following are true about microspherophakia:
a. it is associated with myopia
b. it can cause pupillary block glaucoma which should be
treated with mydriatic
c. it is associated with short stature and stiff joints
d. anterior dislocation is a known complication
e. it can be an autosomal recessive disorder.
3.
a.T b.T
c.T d.T e.T
4. The following metabolic disorders are associated with
cataract:
a. hyperlysinemia
b. hypercalcemia
c. hypocalcemia
d. hypoglycaemia
e. uraemia
4. a.F b.F c.T d.T
e.F
5. Drug-induced cataract include:
a. phenothiazine
87
b. echothiopate
c. aspirin
d. lovastatin
e. systemic beta-blockers
5.
a.T
b.T
c.F
d.T
e.F
6. True statements about the ocular effects of congenital rubella
include:
a. eye defect typically occurs with infection during the
first trimester of pregnancy
b. lamellar cataract is the most common type of cataract
seen in congenital rubella
c. viral particles usually remain in the lens particle long
after birth
d. myopia is common
e. congenital rubella causes progressive chorioretinitis.
6.
a.T b.F c.T
d.F e.F
7. The following are true about blunt trauma on the lens:
a. Vossius ring is caused by the imprint of the pupillary
pigment on the lens
b. Vossius ring causes decreased vision in the majority
of patients
c. Vossius ring is a permanent sign
d. contusion cataract is usually rosette-shaped
e. contusion cataract typically involves the anterior cortex
7.
a.T
b.F c.F d.T e.F
8. In congenital cataract:
a. patients with severe bilateral cataract should have the
cataracts removed before three months of age
88
b. intraocular lens should be used
c. the posterior capsule is usually left intact
d. post-operative visual acuity is usually better in patients
with unilateral cataract than bilateral cataract
e. visual prognosis is poor in patients who develop
nystagmus
8.
a.T
b.F c.F d.F e.T
9. With regard to the materials used in intraocular lens:
a. PMMA is the most common material used in making
foldable lens
b. PMMA transmits a broader spectrum of light than
natural lens
c. silicone is associated with increased capsular fibrosis
d. silicone lens tends to be thinner than PMMA for the
same dioptre power
e. hydrogel lens has a lower tensile strength than PMMA
9.
a.F
b.T c.T d.F e.T
10. Regarding eye drops used in cataract surgery:
a. flurbiprofen is used to dilate pupil
b. flurbiprofen prevents post-operative inflammation
c. phenylephrine stimulates the alpha receptors
d. phenylephrine should be avoided in patients on
monoamine oxidase inhibitors
e. cyclopentolate acts on the constrictor muscle of the iris
10.
a.F
b.F c.T d.T
e.T
11. Homocystinuria:
a. is an autosomal recessive condition
b. causes inferonasal subluxation of the lens
89
c. is associated with raised serum methionine
d. is associated with mental retardation
e. causes osteoporosis
11.
a.T
b.T
c.T
d.T
e.T
12. The histological features of the following are true:
a. Elschnig pearls - proliferation of lens epithelium
onto the posterior capsule
b. posterior subcapsular cataract - bladder cells
c. cortical cataract - morgagnian globules
d. glaucoflecken - Wedl cells
e. Soemmerring ring - sequestration of proliferating lens
fibres in the equatorial region of the capsule
12.
a.T b.T c.T
d.F e.T
13. In a 6 month old baby with bilateral cataract, urine test is
useful in:
a. homocystinuria
b. galactosemia
c. Lowe's syndrome
d. abetalipoproteinemia
e. cystinosis
13.
a.F b.T c.T d.F e.F
14. In galactosemia:
a. the condition is autosomal dominant
b. the most common form is caused by
galatose-a-phosphate uridyl transferase deficiency
c. mental retardation is a feature in untreated cases
d. the cataracts usually assume oil droplet appearance
90
e. early cataracts are reversible with dietary restriction
14.
a.F
b.T
c.T
d.T
e.T
15. Factors that can increase intravitreal pressure during
phacoemulsification include:
a. excessive peribulbar anaesthesia
b. patient taking deep breadth
c. pressure from the speculum
d. choroidal haemorrhage
e. a high bottle height
15.
a.T b.F c.T d.T e.F
16. With regard to viscoelastics:
a. they are cleared in an unmetabolised state by filtration
through the trabecular meshwork
b. sodium hyaluronate has a higher pseudoplastic
behaviour than chondroitin sulphate
c. they are toxic to the endothelium if not removed at the
end of the surgery
d. methylcellulose is cheaper than sodium hyaluronate
e. Viscoat is made up sodium hyaluronate and chondroitin
sulphate
16.
a.T b.T c.F d.T e.T
17. Pseudoexfoliation syndrome:
a. is commonly seen in glass blowers
b. is most commonly in the fifth decade of life
c. causes mid-periphery transillumination
d. causes iridonosis
91
e. is associated with poor pupillary dilatation
17.
a.F b.F c.F d.T e.T
18. The advantages of capsulorrhexis over capsulotomy
include:
a. easier delivery of the nucleus during extracapsular
cataract operation
b. holds the nucleus in the capsular bag during
phacoemulsification
c. better centration of the intraocular lens
d. less likelihood of capsular contraction
e. less likelihood of anterior capsular tear extending into
the posterior capsule
18.
a.F
b.T
c.T
d.F
e.T
19. Cataract extraction in an eye that had previous par plana
vitrectomy is associated with an increased risk of :
a. zonular dehiscence
b. suprachoroidal haemorrhage
c. posterior capsule rupture
d. globe collapse
e. cystoid macular oedema
19.
a.T b.F c.T d.F e.T
20. True statements about the aspiration systems used in
phacoemulsification include:
a. diaphragm pump produces the slowest build up of
vacuum
b. peristaltic pump allows linear control of vacuum
c. Venturi pump is associated with the highest risk of
inadvertent engagement of unwanted tissue
92
d. Venturi pump gives the most rapid rise in vacuum
e. high aspiration is needed during the early stage of
phacoemulsification
20.
a.T b.T c.T d.T e.F
Test I (for final FRCOphth /
MRCOphth)
1. Gorlin's syndrome:
a. Is an inherited condition
b. Is associated with multiple basal cell
carcinoma
c. Increases the incidence of cataract
d. Is associated with liver carcinoma
e. Increases the risk of retinal detachment
2. Posterior polymorphous dystrophy :
a. Is present at birth
b. Causes corectopia
c. Is inherited in an autosomal recessive fashion
d. Causes blindness in over 90% of sufferers
e. Can be treated with lamellar corneal grafts
3. Crystalline deposits in the conjunctiva may be
found in:
a. Amyloidosis
b. Cystinosis
c. Gold treatment
d. Contact lens wear
e. Uraemia
4. The histology of pterygium includes:
a. Myxoid degeneration
b. Epithelial inclusion bodies
93
c. Precancerous changes
d. Squamous metaplasia of the epithleium
e. Elastotic degeneration
5. The following may be found in conjunctiva in
keratoconjunctivitis sicca
a. Dysplasia of the epithelium
b. Keratinisation
c. Eosinophils
d. Numerous goblet cells
e. Lymphocytic infiltrates
6. Merkel cell carcinoma:
a. Is most common in the bulbar conjunctiva
b. Is associated with good prognosis
c. Is an APUD cell tumour
d. Is associated with ultraviolet exposure
e. Produces corneal pigmentation
7. Histological changes in lens induced uveitis
include:
a. Ghost cells
b. Giant cell reaction
c. Amyloid in the cornea
d. Vasculitis
e. Non-caseating granuloma
8. Xeroderma pigmentosa is associated with:
a. Intraouclar tumours
b. Metabolic disorders
c. Photosensitivity
d. Autosomal recessive inheritance
e. Squamous cell carcinoma
94
9. Pathological changes in thyroid ophthalmopathy
includes:
a. Lymphocytic infiltrates
b. Giant cell reaction
c. Muccopolysaccharides
d. Fat cells
e. Fibroblast proliferation
10. In histological stains:
a. Chlamydia inclusion bodies stain with Giemsa
b. Osmium tetroxide is the stain used in electron
microscopy
c. Fungi stain with Fuelgin's stain
d. Eosin stains the nuclei blue
e. Haematoxylin stains the nuclei pink
Click for answers
11. Sympathetic ophthalmitis
a. Is characterised by Dellen-Fuchs' nodule
b. Prevented by removal of the exciting eye
within 6 weeks of injury
c. Pigment laden macrophages are diagnostic
d. Is associated with poliosis and vitiligo
e. Is a common cause of visual loss in traumatic
eye injury
12. Abnormal material which may be present in the
vitreous include:
a. Exfoliation
b. Lipofuscin
c. Haemosiderin
d. Amyloidosis
e. Calcium
13. In giant cell arteritis:
95
a. Histological diagnosis is based on
fragmentation of the internal intimal
b. C-reactive protein is always raised
c. Giant cell is needed for diagnosis
d. Anterior cerebral artery is often involved
e. Anaemia is a feature
14. The following conditions are pre-malignant:
a. Oncocytoma of the caruncle
b. Actinic keratosis
c. Bowen's disease
d. Squamous papilloma
e. Syringoma
15. Hypoxic damage in diabetic mellitus is suggested
by:
a. Nerve fibre layers haemorrhage
b. Iris neovascularisation
c. Anterior uveitis
d. Vascular loop
e. Decreased hard exudate
16. The following biopsy report may be found with
rheumatoid arthritis:
a. Posterior scleritis
b. Loss of goblet cells in the conjunctiva
c. Episcleral necrotic tissue
d. Giant cell reaction
e. Vasculitis
17. The following report is associated with increased
mortality in an enucleated eye with retinoblastoma:
a. Involvement of the optic nerve
b. Presence of calcification within the tumours
c. Necrosis within the tumours
d. Exophytic retinoblastoma
e. Presence of retinal detachment
96
18. Diabetic microangiopathy include:
a. Endothelial vascular proliferation
b. Proliferation of pericytes associated with
microaneurysms
c. Deposition of calcium in the intima
d. Thickening of the basement membrane
e. Necrosis of the endothelium
19. Optic nerve glioma:
a. Is associated with neurofibromatosis type I
b. Increased incidence after 30 years old
c. Majority is of pilocytic astrocytoma
d. Rarely infiltrate the perineural tissue
e. Mortality of 40%
20. Mutton fat keratic precipitates are seen in:
a. Uveal effusion syndrome
b. Tuberculosis
c. Fuch's heterochromia cyclitis
d. Sarcoidosis
e. Fungal infection
Click for answers
21. Albinism:
a. Hair bulb for tyrosine positivity test is useful
only when the child is 5 years old
b. Is associated with platelets abnormality
especially with aspirin
c. Complete decussation is invariable in all
albinism
d. Is associated with thrombosis under general
anaesthesia
e. Causes macular hypoplasia
22. Infanitle glaucoma is associated with:
97
a. Aniridia
b. Sturge-Weber syndrome
c. Albinism
d. Axial iris stromal abnormality
e. Von Hippel-Lindau's syndrome
23. The following are true about nystagmus:
a. See-saw nystagmus is associated with
homonymous hemianopia
b. Down-beat nystagmus is associated with
lesion in the foramen magnum
c. Manifest latent nystagmus occur in unilateral
congenital cataract
d. Gaze-evoked nystagmus is specific for
cerebellar lesion
e. Convergence retraction nystagmus is
associated with mid-brain tumours
24. Melanocytoma:
a. Contains plump black naevus cells
b. Metastasises to the brain
c. Is commoner in temperate than tropical
countries
d. Does not cause optic nerve compression
e. Is associated with renal tumour
25. Retinoblastoma:
a. Is inherited in 50% of cases
b. Is caused by deletion of 11q14
c. Rosettes suggest good prognosis
d. There is an increased incidence of
osteosarcoma
e. Causes death in 60% of sufferers in the UK
26. Corneal dystrophy may contain the following
substances:
a. Muccopolysaccharides
b. Cholesterol
98
c. Amyloidosis
d. Uroprophyrin
e. Urate
27. Lacrimal gland tumour of the adenocytic type:
a. Has a peak incidence of around 60 years
b. Has well-defined capsule
c. Spread through perineural tissue
d. Metastasise to the brain early
e. Causes mortality in over 90% of cases in the
UK
28. Dissociated vertical deviation:
a. Hyperdeviation on abduction and adduction
b. Began at 2 to 3 years old
c. Poor binocular single vision
d. Inferior recession is the treatment of choice
e. Highest incidence with congenital esotropia
29. Duane type A:
a. Causes limited abduction and adduction
b. Widening of the palpebral fissure on
abduction
c. Sixth nerve palsy is an associated problem
d. Causes head tilt more commonly than other
types
e. Is associated with six cranial nerve nucleus
aplasia
30. Retinoschisis:
a. Should be treated with laser to prevent
progression
b. Causes absolute scotoma
c. Is associated with water mark
d. Is bilateral in the majority of cases
e. Does not cause retinal detachment
99
Click for answers
31. Medications for glaucoma:
a. Beta-blocker is contraindicated in primary
heart block
b. Beta-blocker is contraindicated in those on
nifedipine
c. Acetazolamide is contraindicated in allergy to
sulphonamide
d. Acetazolamide needs potassium supplement
e. Latanoprost is contraindicated in patient with
uveitis
32. Dendritic ulcers may be treated by:
a. Argon laser
b. Iodination
c. Topical acyclovir
d. Systemic acyclovir
e. Tarsorrhaphy
33. In choroidaemia:
a. Pigmentary changes occurs in carriers
b. Carriers may have abnormal EOG
c. The mode of inheritance is autosomal
dominant
d. Arginine free diet is beneficial
e. Central vision is lost early
34. Deafness is associated with:
a. Cockayne's syndrome
b. Alstrom's syndrome
c. Alport's syndrome
d. Usher's syndrome
e. Abetalipoproteinaemia
35. Prominent corneal nerves are seen in :
100
a. Ectodermal dysplasia
b. Ichythyosis
c. Refsum's syndrome
d. Macular dystrophy
e. Acanthoamoeba keratitis
36. The following are recognised association:
a. Abetalipoproteinaemia and acanthocytosis
b. Refsum's disease is associated with cerebellar
ataxia
c. Spinocerebellar ataxia in Friedrich's ataxia
d. Homocystinuria and recurrent deep vein
thrombosis
e. Paget's disease and deafness
37. Keratoconus is associated with:
a. Down's syndrome
b. Ehlers-Danlos' syndrome
c. Marfan's syndrome
d. Usher's syndrome
e. Atopic keratoconjunctivitis
38. In acquired syphilis:
a. Interstitial keratitis is a recognised feature
b. The infection is most infectious in the
secondary stage
c. Causes secondary uveitis
d. Treatment is with procaine penicillin
e. May present with a rash which is highly
infectious
39. Reduced blink reflex occurs in:
a. Alcohol intoxication
b. Parkinsonism
c. Progressive supranuclear palsy
d. Myasthenia gravis
e. Hypothyroidism
101
40. In heterochromic cyclitis:
a. 60% of patients develop glaucoma
b. Show a good response when treated with
steroids
c. Lens implantation following cataract surgery
is contraindicated
d. Hyphaema during cataract surgery is due to
iris neovascularization
e. Is associated with iris transillumination
Click for answers
41. Retinal dialysis:
a. Is associated with posterior vitreous
detachment
b. Is associated with myopia
c. Has a mobile posterior flap
d. Can be treated by cryotherapy, encirclement
and a local scleral buckle
e. Is commonest in the superior temporal
quadrant
42. Ocular bobbing:
a. Is associated with pontine haemorrhages
b. Is only seen in comatose patients
c. Is associated with loss of doll's head
movement
d. Is associated with glioblastoma
e. Occurs in bilateral cerebral infarction
43. The following tests may be useful in the
assessment of a patient with sarcoidosis:
a. Chest X-ray
b. ACE
c. Conjunctiva biopsy
d. Serum calcium
e. Gallium scan
102
44. In Holmes-Adie pupil:
a. There is absent or retarded pupil reactions to
light and near
b. Most cases are bilateral
c. Commoner in male than female
d. Causes reduced or absent accommodation
e. Constricts with 2.5% methacholine
45. A fixed dilated pupil which does not react to
direct or consensual light stimulus could be caused
by:
a. Optic nerve avulsion
b. Due to blunt trauma
c. Orbital apex syndrome with third nerve palsy
d. Topical application of atropine
e. Siderosis bulbi
46. Leber's optic neuropathy:
a. Typically presents in the fourth decade of life
b. Males do not transmit the disease
c. Usually affect the males more than the
females
d. Is inherited in autosomal X-link fashion
e. The optic disc is pale early in the disease
47. In acute AMPPE:
a. Bilateral in the majority of cases
b. Hypofluorescence in early stage of
fluorescein angiography
c. Spontaneous recovery is common
d. 80% of patients have exudative vasculitis
e. Anterior uveitis is a prominent feature
48. In central serous retinopathy:
103
a. Abnormalities recorded on the Amsler's chart
are uncommon
b. Often shows focal leakage on fluorescein
fundus photography
c. A macular hole is a common end result
d. The image perceived by the patient on the
affected side in unilateral cases is bigger than on
the normal side
e. A dense central scotoma is the most common
presentation
49. Kayser-Fleishcher's rings:
a. Caused by copper in the Bowman's membrane
b. Has brown-green, grey or orange colour
c. Decreases with D-penicllamine treatment
d. Can occur with copper intraocular foreign
body
e. Starts at 3 and 9 O'clock
50. Corneal grafts have:
a. Host epithelium
b. Host endothelium
c. Host keratocytes
d. Host basement and descemet's membrane
e. Host nerves
51. Vernal keratoconjunctivitis:
a. Is usually unilateral
b. Eosinophil is common in the papillae
c. Can be of limbal type alone
d. Is commoner in adult than children
e. Causes corneal melt
52.The following conditions are inherited in an
autosomal recessive patterns:
a. Protanopia
b. Congenital retinoschisis
104
c. Sturge-Weber's syndrome
d. Neurofibromatosis type II
e. Blue cone achromatopsia
53. Coat's disease:
a. Has equal sex incidence
b. Caused by peripheral neovascularization
c. Is inherited in an autosomal dominant pattern
d. Causes rhegmatogenous detachment
e. Can be treated with laser
54. With regard to laser:
a. Xanthophll absorb green light
b. Diode laser penetrate vitreous haemorrhage
c. Argon blue-green is commonly used for grid
laser
d. YAG laser produces red colour
e. Ophthalmic laser is classified as type I laser
55. Roth's spots contain:
a. Positive staphylococcus bacteria
b. Purulent exudates
c. Vascular occlusion
d. Lymphoblasts
e. Eosinophils
56. Breaks in descemet's membrane occur in:
a. Keratoconus
b. Fuch's dystrophy
c. Acute glaucoma
d. Lattice dystrophy
e. Congenital glaucoma
57. Acute glaucoma is suggested by:
105
a. Sphincter paralysis
b. Cataract
c. Ectopia uvea
d. Posterior subcapsular cataract
e. Recurrent corneal erosion
58. In blow-out fracture:
a. A step on the orbital rim is palpable
b. Adduction problem occur
c. Surgical emphysema is a recognised feature
d. Immediate treatment is important
e. Decreased sensation at the tip of the nose is
common
59. Persistent hyperplastic primary vitreous:
a. Caused elongated ciliary muscles
b. Angle closure glaucoma is a recognised
feature
c. Optic disc abnormality is seen
d. Causes microphthalmic eye
e. Visual prognosis is good if treated early
60. Pars planitis:
a. Is frequently associated with retinal
detachment
b. Causes vitreous haemorrhage
c. Has poor visual prognosis
d. Causes cyclitic membrane
e. Has a peak incidence at the fifth decade
Click here for Answers
Multiple Choice Questions on Medical Retina
(click the number for the answers)
106
True statements about exudative vitreoretinopathy include:1.
a. it is an autosomal recessive disorder
b. it has a fundal appearance similar to patients with
retinopathy of prematurity
c. the main cause of visual loss is retinal detachment
d. peripheral retinal neovascularization is a common
features
e. the condition has a relentless progression leading
to severe visual loss.
In AZOOR (acute zonular occult outer retinopathy):2.
a. photopsia is a recognized feature
b. visual loss is caused by retinal necrosis
c. fluorescein angiogram is useful for diagnosis
d. most affected patients are over the age of 40
e. oral prednisolone is the treatment of choice
In PIC (primary idiopathic choriodopathy):3.
a. myopia is an association
b. more male than female are affected
c. vitritis is common
d. viral prodrome is common
e. both eyes are affected in 25% of cases
The following are true about IPCV (idiopathic polypoidal4.
choriodal vasculopathy):
a. it is seen exclusively in patients of African origin.
b. it causes recurrent vitreous haemorrhage
107
c. it has a better visual prognosis than age-related
macular degeneration
d. the condition is characterized by dilated choroidal
is best seen with indocyanine green angiography
e. the lesions respond well to laser treatment.
Stickler's syndrome:5.
a. is an autosomal dominant condition
b. is associated with retinal detachment in 20% of the
patients
c. shows abnormal uvula
d. is associated with pigmentation along the peripheral
retinal vessels
e. has abnormal type II collagen
The differences between ARN (acute retinal necrosis) and PORN6.
(progressive outer retinal necrosis) include:
a. ARN is associated with herpes simplex virus but not
PORN
b. ARN has a worse visual prognosis than PORN
c. ARN is associated with AIDS but not PORN
d. retinal detachment is common in ARN but not in
PORN
e. vitritis is more severe in ARN than in PORN
True statements about vigabatrin include:7.
a. it is the first line of treatment in patient with petit mal
b. it is a GABA transaminase inhibitors
c. it causes vortex keratopathy
108
d. it causes visual field defect in 30% of users
e. stopping the vigabatrin can reverse the visual field
loss
Oguchi's disease:8.
a. is an autosomal dominant condition
b. is characterized by stationary night blindness
c. is associated with colour blindness
d. is characterized by Mizuo-Nakamura phenomenon
e. causes significant visual loss before the age of 50
Retinal crystals are seen in the use of:9.
a. tamoxifen
b. hydroxychloroquine
c. canthaxanthin
d. desferrioxamine
e. magnesium silicate
In retinopathy caused by shaken baby syndrome:10.
a. the patient is usually between the age of 3 and 5
years of age
b. the retinopathy may resemble central retinal vein
occlusion
c. neurological damage is common
d. skull fracture is always present
e. the visual prognosis is good
109
The following signs favour the diagnosis of central retinal 11.
artery occlusion
rather than ophthalmic artery occlusion:
a. the presence of cherry red spot
b. the presence of relative afferent pupillary defect
c. reduction of both a and b wave on the electroretinogram (ERG)
d. delayed choroidal circulation in fluorescein angiography
e. absence of pigmentary changes in the posterior segment
The following conditions can give rise to the clinical appearance of12.
cystoid macular oedema but without fluorescein angiographic
evidence of
late fluorescein leakage:
a. epiretinal membrane
b. Irvine-Gass syndrome
c. juvenile retinoschisis
d. nicotinic acid maculopathy
e. Goldman-Favre syndrome
The following retinal findings are associated with rhegmatogenous 13.
retinal
detachment:
a. paving stone degeneration
b. cystic retinal tuft
c. pars plana cyst
d. congenital hypertrophy of retinal pigment retinal epithelium
e. meridonal folds
110
Features of acute macular neuroretinopathy include:14.
a. paracentral scotoma
b. mild vitritis
c. presence of a triangular shape dark lesion in the macula
d. abnormal electroretinogram
e. profound visual loss with only 5% recover 6/36 vision
In von Hippel-Lindau's disease:15.
a. retinal angioma is seen in 50% of the patients
b. angioma of the disc may mimic choroidal neovascularization.
c. any retinal angioma should be treated early as spontaneous
regression is rare
d. treatment of the retinal angioma can give rise to subretinal
exudation
e. the main cause of visual loss is exudative maculopathy
True statements about Stargardt's disease include:16.
a. the majority of the cases are inherited in an autosomal
dominant pattern
b. the condition is symptomatic in the first or second decade of life
c. abnormal red-green colour vision is common
d. histopathology reveals the presence of subretinal lipofuscin
substance
e. macular oedema is a common cause of visual loss
In Wyburn-Mason's syndrome:17.
a. is an X-linked inheritance
111
b. the fundal appearance is caused by arteriovenous
communication
c. exudative maculopathy is the main cause of visual loss
d. skull X-ray reveals tram-line calcification
e. visual field defect occurs in one third of the patient
The following statements are true:18.
a. electroretinogram is diagnostic of Stargardt's disease
b. amplitude of electroretinogram is reduced in carriers of
choroideremia
c. electro-oculogram light peak to dark trough ratio is normal
in adult onset foveomacular dystrophy
d. electroretinogram is diagnostic of Leber's congenital amaurosis
e. electroretinogram is useful in detecting carrier of X-linked
retinitis pigmentosa
Bietti's crystalline dystrophy:19.
a. is commoner in females than males
b. causes choroidal atrophy
c. has abnormal crystalline deposits in the cornea and lymphocytes
d. produces crystalline deposits in all layers of the retina
e. is associated with abnormal electroretinogram
The following mucopolysaccharidosis are associated with 20.
pigmentary
retinopathy:
a. Hunter's syndrome
112
b. Sanfilippo's syndrome
c. Scheie's syndrome
d. Morateaux-Lamy's syndrome
e. Sly's syndrome
The following conditions are associated with an optically empty 21.
vitreous
and peripheral pigmentary changes:
a. Jansen's disease
b. Wagner's disease
c. X-linked retinoschisis
d. Favre-Goldmann's syndrome
e. Kearn-Sayre's syndrome
The following statements are true about Jansen's disease and Wagner's 22.
disease:
a. both conditions have autosomal dominant inheritance
b. both conditions are associated with increased retinal detachment
rate
c. the ERG are reduced in both conditions
d. the EOG are normal in both conditions
e. high myopia are common in both conditions
Features of macroaneurysms include:23.
a. more common in women than men
b. more common in the temporal arcade than the nasal arcade
113
c. retinal artery occlusion
d. occurs in IRVAN syndrome
e. progressive enlargement and therefore early treatment with
treatment with laser photocoagulation is recommended
In carcinoma associated retinopathy (CAR):24.
a. the condition is a paraneoplastic syndrome
b. colour vision and night blindness are typical presentation
c. melanoma is the most common type of carcinoma seen
d. autoantibodies are seen in the blood
e. plasmaphoresis is useful in restoring the visual function
Deafness and pigmentary retinopathy are seen in the following 25.
conditions:
a. Usher's syndrome
b. congenital rubella
c. congenital syphilis
d. Hunter's syndrome
e. Refsum's syndrome
Norrie's disease has the following features:26.
a. mental retardation
b. histology of the retina reveals rosette formation
c. X-linked inheritance
d. deafness
e. leukocoria which may be mistaken for retinoblastoma
114
In X-linked retinoschisis:27.
a.the splitting of the retina occurs at the level of inner plexiform
layer
b. electrophysiology reveals negative ERG
c. the main cause of visual loss is recurrent retinal detachment
d. DNA analysis is useful in detecting carrier of the condition
e. myopia is common
In ETDRS (Early Treatment Diabetic Retinopathy Study) significant28.
macular oedema is defined as:
a. hard exudates within 500um of the fovea
b. retinal thickening within 500um of the fovea
c. macular staining in fluorescein angiography
d. retinal thickening greater than one disc area in size and within
one disc diameter of the centre of the fovea
e. macular haemorrhage within 500um of the fovea
The following are true regarding the recommendations of the Branch29.
Retinal Vein Occlusion Study for patient with branch retinal vein
occlusion:
a. fluorescein angiography should be performed within four weeks
of the onset of branch retinal vein occlusion
b. presence of more than 5 disc diameter of ischaemia should be
treated with pan-photocoagulation
c. macular oedema should be treated within 2 months of the onset
of branch retinal vein occlusion
d. macular laser is useful in patient who has macular ischaemia
e. 30% of branch retinal vein occlusion develops
neovascularization
115
The following are the findings from the Diabetes Control and 30.
Complications
Trial:
a. tight glycaemia control reduces the progression of diabetic
retinopathy in both insulin and non-insulin dependent diabetic
b. the need for macular laser treatment is decreased in wellcontrolled group
c. worsening of the retinopathy can occur during the initial phase
of tight control
d. attack of hypoglycaemia is increased in tightly controlled group
e. the incidence of cataract is reduced.
Click here for the answers
Oculoplatic and lacrimal
1. With regard to ptosis:
a. the most common abnormality in congenital ptosis is in the levator
muscle
b. lid lag on downgaze is a feature of senile ptosis
c. in Horner's syndrome, ptosis is due to paralysis of the Muller's
muscle
d. the levator function is good in aponeurotic defect
e. raised skin crease is a feature of congenital ptosis.
1.
a.T
b.F
c.T
d.T
e.F
2. True statements about eyelid lesions include:
116
a. basal cell carcinoma affects the upper lid more commonly than
the lower lid
b. recurrent follicular conjunctivitis can be the result of molluscum
contagiosum
c. keratoacanthoma can resolve spontaneously without treatment
d. sebaceous cell carcinoma has a worse prognosis than squamous
cell carcinoma
e. basal cell carcinoma is common in patients with xeroderma
pigmentosa
2.
a.F
b.T
c.T
d.T
.eT
3. The following are true about tumours arising from hair follicles:
a. trichoepithelioma is commoner in women than men
b. women with multiple trichilemmomas have an increased risk of
breast cancer
c. calcification is a feature of pilomatrixoma
d. pilomatrixoma is commoner in children than adult
e. malignant transformation is common in pilomatrixoma
3.
a.T
b.T
c.T
d.T
e.F
4. In blepharochalasis:
a. most affected patients are in their 40s
b. there are recurrent lid oedema
c. there are levator disinsertion
d. the condition is associated with Ascher's syndrome
e. systemic steroid is useful
4.
a.F
b.T
c.F
d.T
e.F
117
5. The nasolacrimal canal is made up of the following bones:
a. maxillary
b. lacrimal
c. inferior turbinate
d. palatine
e. frontal
5.
a.T
b.T
c.T
d.F
e.F
6. The Müller's muscle:
a. is innervated by the parasympathetic nerve
b. has the peripheral arcade of the upper lid running
on its anterior surface
c. is about 5 mm long
d. arises from the superior rectus
e. elevates the upper lid by about 2 mm
6.
a.F
b.T
c.F
d.F
e.T
7. The following are true about the treatment of basal cell carcinoma:
a. Mohs' micrographic excision should be performed in all cases of
lower lid basal cell carcinoma
b. Mohs' micrographic excision is useful for the diagnosis of basal
cell carcinoma
c. Mohs' micrographic excision allows three dimensional examination
of tumour margins
d. aggressive tissue removal is avoided in Mohs' micrographic
excision
e. before Mohs' tissue excision the skin is spread with zinc sulphate
paste
118
7.
a.F b.F c.T d.F e.F
8. The following are true:
a. in epicanthus tarsalis, the medial canthal skin fold arises from the
lower lid
b. epicanthus inversus is a cause of in-growing eye lashes
c. euryblepharon is a feature of Treacher-Collin's syndrome
d. in distichiasis, there is an extra row of eyelashes in the orifices of
the meibomian gland
e. ankyloblepharon refers to the fusion of the upper and lower eyelids
8.
a.F
b.F
c.T
d.T
e.T
9. Blepharophimosis:
a. is an autosomal dominant condition
b. epicanthus inversus is a feature
c. has nasal bridge hypoplasia
d. has hypertelorism
e. is associated with mental retardation
9.
a.T
b.T
c.T
d.T
e.F
10. In congenital obstruction of the nasolacrimal duct:
a. the success rate of probing at 12 months of age is about 60%
b. 85% of the cases are bilateral
c. the blockage is found most commonly at the common canaliculus
d. in congenital dacryocystocele, the skin is usually inflamed and hot
e. in congenital dacryocystocele, the sac contains amniotic fluid
119
10.
a.F
b.F
c.F
d.F
e.T
11. Upper lid retraction may be seen in:
a. Grave's eye disease
b. an eye with contralateral ptosis
c. hepatic failure
d. hydrocephalus in children
e. Parinaud's syndrome
11.
a.T
b.T
c.T
d.T
e.T
12. During dacryorhinostomy, the following are usually removed:
a. maxillary bone
b. frontal bone
c. lacrimal bone
d. lacrimal fossa
e. ethmoid bone
12.
a.T
b.F
c.T
d.T
e.F
13. The following are true about chronic canaliculitis caused by
Actinomyces:
a. it is the most common cause of chronic canaliculitis
b. this infection is caused by a fungus
c. bloody tear is a feature
d. syringing is usually normal despite epiphora
e. dacryorhinostomy is the treatment of choice
13.
a.T
b.F
c.T
d.T
e.F
120
14. Features of eyelid keratoacanthoma include:
a. malignant transformation in 50% of cases
b. loss of eyelashes
c. rapid growth
d. ulcerated surface filled with keratin
e. spontaneous resolution
14.
a.F
b.T
c.T
d.T
e.T
15. Features of Goldenharg's syndrome:
a. eyelid coloboma
b. Duane's syndrome
c. optic nerve hypoplasia
d. ptosis
e. lipodermoid
15.
a.T
b.T
c.T
d.F
e.F
16. Hypertelorism is seen in:
a. Curzon's syndrome
b. frontal encephalocele
c. blepharophimosis
d. capillary haemangioma
e. Turner's syndrome
16.
a.T
b.T
c.F
d.F
e.T
17. The following are contributory factors in senile (involution)
entropion:
121
a. atrophy of the orbital fat
b. migration of the preseptal orbicularis over the pretarsal muscle
c. dehiscence of the lower lid retractor
d. horizontal lower lid laxity
e. fat herniation through the orbital septum
17.
a.T
b.T
c.T
d.T
e.F
18. The following are true about Jones dye test in epiphora:
a. it is used to diagnose complete obstruction of the lacrimal drainage
system
b. if the Jones 1 (primary) test is normal, the cause may be due to
hypersecretion of tear
c. if the Jones 1 (primary) test is negative, the obstruction is in the
common canaliculus
d. if the Jones II (secondary) test is negative, the abnormality may be
due to pump failure
e. if the Jones II (secondary) test is positive, there is a partial
nasolacrimal duct obstruction
18.
a.F
b.T
c.F
d.T
e.T
19. True statements about cavernous haemangioma include:
a. it is commoner is female than male
b. spontaneous resolution is common
c. it is well-encapsulated
d. retina striae is a recognized sign
e. the proptosis increases with Valsalva's manoeuvre
19.
a.T
b.F
c.F
d.T
e.F
122
20. In a patient with laceration to the upper lid:
a. the presence of fat herniation indicates the orbital septum is
penetrated
b. the presence of lacrimal gland indicates the orbital septum is
penetrated
c. the orbital septum should be resutured if penetrated
d. in the presence of ptosis, exploration should be delayed for at least
72 hours to avoid retrobulbar haemorrhage
e. ptosis is usually caused by damage to the oculomotor nerve
20.
a.T
b.T
c.F
d.F
e.F
MCQs on Ocular Motility
(click the question number for the answers)
A 8 year-old girl was referred with a history of sore eyes when watching TV and
reading the blackboard. Her parents complained that her eye appeared to
wonder out at times. The visual acuity was 6/6 in both eyes. Alternate cover test
revealed exophoria with slow recovery with the angle of deviation measuring 35
prism dioptre at distance and 10 prism dioptre at near. Following a patch test,
she measured 35 prism dioptre at distance and 15 dioptre at near. (Questions 1 3)
1. The following are true regarding this patient:
a. the patient has intermittent exotropia of the divergence
excess type
b. binocular single vision may be suppressed for distant
fixation but normal with near fixation
c. patching of the eye suspend the tonic fusional
convergence
d. a V-pattern deviation is common on upgaze
123
e. spontaneous resolution is common as the patient ages
2. The following treatments are appropriate:
a. tinted glasses
b. over minus glasses if refraction revealed myopia
c. orthoptic exercise
d. miotic
e. base-in prisms
3. Operation was carried out with bilateral lateral rectus
recession. One day post-operative, the patient measured 10
prism dioptres of consecutive esotropia. The following are true:
a. patching of the good eye should be carried out
b. prescribe prism to maintain binocular fusion
c. the eye should be explored for slipped lateral rectus
d. overcorrection of exotropia is desirable and the patient
should be reassured
e. the esotropia will lessen with time
A 6 year-old gird was referred having moved into your area. She was previously under the
care of another hospital and had squint operation for a convergent squint which was
present since infancy. Her visual acuity was 6/9 in the right eye and 6/6 in the left. Cover
test revealed a small right exotropia of 5 prism dioptres. The covered eye showed
elevation and nystagmus was observed when either eye was covered.The nystagmus was
absent when both eyes were uncovered. (Questions 4 - 6)
4. The following are true:
a. the esotropia prior to the surgery was likely to be more
than 30 prism dioptres
b. the patient is likely to have high hypermetropia in the right
eye
124
c. binocular single vision is usually better than 60 degrees of
arc
d. asymmetrical optokinetic nystagmus is common
e. further surgery is likely to be needed.
5. True statements about elevation of the covered eye include:
a. it is caused by inferior oblique muscle overaction
b. the eye demonstrates Bielchowsky phenomenon
c. the elevation increases on adduction
d. the elevated eye usually demonstrate extortion as it
elevates
e. the condition is usually bilateral but may be asymmetrical
6. With regard to the nystagmus:
a. it is termed manifest latent nystagmus
b. the fast phase is toward the side of the uncovered eye
c. the intensity of the nystagmus increases on abduction
d. the nystagmus has a similar waveform to congenital
nystagmus
e. it is caused by cerebellar dysfunction
A 5 year-old child is referred the orthoptic because of strabismus. The findings include an
esotropia of 20 prism dioptre in the primary position with limited abduction of the right
eye. On adduction, there is narrowing of the lid fissure and upshooting ot he right. The
right eye was normal. (Questions 7 - 9)
7. The following is true:
a. the child is likely to be male than female
b. the patient is likely to have a right face turn
c. the narrowing of the lid is caused by aberrant
regeneration of the third nerve
125
d. amblyopia is found in 90% of patients with this condition
e. the diagnosis is type II Duane's retraction syndrome
8. The following may be associated with this condition:
a. Marcus Gunn Jaw winking
b. crocodile tears
c. glaucoma
d. cataract
e. microphthalmos
9. True statements of this condition include:
a. the strabismus is concomitant
b. surgery is useful to restore normal ocular motility
c. surgery is indicated in patient with abnormal head posture
d. resection of the muscles may worsen narrowing of the lid
e. Faden procedure can reduce the upshoot
A 24 year-old man was referred to the eye casualty because of intermittent vertical
diplopia. Alternating cover test revealed a vertical phoria and when looking at a white dot
through Maddox rods through both eyes he described the lines as follow:
(Questions 10 - 13)
10. The following are true if the patient had a fourth nerve palsy:
a. the patient has a right fourth nerve palsy
b. when the vertical diplopia is present, the left eye may be
hyperdeviated
c. with the first step of the three step tests, the right eye will
show hyperphoria
126
d. with the second step of the three step test, the left eye will
show hyperphoria in left gaze
e. with the third step of the three step test, the right eye will
show hyperphoria with right head tilt
11. The following features favours a congenital to an acquired fourth nerve palsy:
a. absence of cyclotorsion
b. abnormal head posture
c. V pattern on upgaze
d. vertical fusional amplitude greater than 4 prism dioptre
e. absence of binocular single vision
12. The following favours a bilateral to an unilateral fourth nerve
palsy:
a. extorsion of more than 10 degrees
b. an abnormal head posture consists mainly of chin
depression
c. positive Bielchowsky head tilt test to either side
d. significant hyperdeviation in the primary position
e. large V pattern
13. Surgical treatment in this patient may include:
a. right inferior oblique recession
b. right super rectus recession
c. left inferior rectus recession
d. right Harada-Ito procedure
127
e. Faden's procedure of the right superior oblique muscle
A 64 year-old man complains of horizontal diplopia which is worse on right gaze.
(Questions 14 - 17)
14. The following are true if he had a sixth nerve palsy:
a. a right esotropia which is worse for distance than near
b. a face turn to the left
c. improved right eye movement when the left eye is closed
d. V pattern on upgaze
e. upshooting of the right eye on left gaze
15. The following muscle sequelae may occur if he had a sixth nerve palsy:
a. contraction of the right lateral rectus
b. contraction of the right medial rectus
c. inhibitional palsy of the left lateral rectus
d. overaction of the left medial rectus
e. contraction of the left lateral rectus
16. The following additional signs and the location of the lesion are
true:
a. bilateral swollen disc - posterior fossa tumour
b. right miosis and ptosis - cavernous sinus lesion
c. fourth nerve palsy - orbital lesion
d. right facial nerve palsy and analgesia and loss of taste to
anterior two third of the tongue - dorsal pon
128
e. left hemiplegia - ventral pon
17. Treatment of this patient may include:
a. injection of botulinum toxin into the right medial rectus
b. base out Fresnel prism over the right eye
c. Faden operation of the left medial rectus
d. recession of the left medial rectus and resection of the
right lateral rectus
e. recession of the right medial rectus and resection of the left
lateral rectus
18. True statements about microtropia include:
a. it may result from operation for congenital esotropia
b. the angle of deviation is typically between 15 and 20
prism dioptres
c. stereopsis is usually absent
d. anisometropia is a common feature
e. a base out 4 dioptre prism can be used to detect central
scotoma
19. Spasmus nutans:
a. usually begins at 3 years of age
b. is associated with head nodding
c. has jerky and large amplitude nystagmus
d. is rarely associated with neurological disorders
e. usually resolves spontaneously.
20. The following are true about accommodative esotropia:
129
a. usually begins around the age of 2 years
b. may be caused by a high AC/A ratio
c. there is usually hypermetropia of more than + 3.00 D
d. diplopia is rare
e. amblyopia is uncommon.
Test two (for final FRCOphth/MRCOphth)
1. Optic neuropathy have been reported with:
a. ethambutol
b. penicillin
c.chloramphenicol
d. isoniazid
e. tobultamide
2. The following are known features of ocular
hypotony:
a. optic oedema
b. cataract
c. retinal detachment
d. corneal oedema
e. choroidal ischaemia
3. True statements about ankylosing spondylitis
include:
a. HLA-B27 is found in 90% of sufferers
b. uveitis is found in 15 to 20% of
sufferers
c. the condition is commoner in female
130
d. female sufferers have a more severe
course than male
e. the radiological changes can occur in
the spine before symptoms
4. In scleritis:
a. scleromalacia perforans are more
commonly associated with systemic
diseases than posterior scleritis
b. pain is not a prominent feature
c. retinal detachment is a known
complication
d. systemic steroid is indicated in all
scleritis
e. glaucoma is a known complication
5. The following parasites can cause ocular
problems:
a. Trichinella spiralis
b. Schistosoma haemolytica
c. Ascaris lumbricoides
d. Loa loa
e. Leishmania donovani
6. In uveal melanoma:
a. the condition is more common in
Caucasian than other races
b. the incidence is higher in female than
males
c. the optic nerve is commonly infiltrated
by the tumour
d. epithelioid cell type has poorer
prognosis than other cell type
e. abdominal CT scanning is more
sensitive than blood test in detecting
hepatic metastasis
7. Concerning Marfan's syndrome:
131
a. the condition is inherited in autosomal
dominant manner
b. it is caused by defect in the formation
of type I collagen
c. the lens typically shows upper nasal
subluxation
d. the patients have a higher incidence of
hypermetropic refractive error
e. the sufferers tend to be mentally
subnormal
8. The following are true about measles:
a. it is an RNA virus
b. causes Koplik's spots on the
conjunctiva
c. acute catarrhal conjunctivitis is a
known features
d. causes blinding keratitis in the
presence of vitamin A deficiency
e. causes subacute sclerosing
panencephalitis
9. Cyclosporin A:
a. is a fungal metabolite
b. has poor tissue penetration when
applied topically
c. is the drug of choice in Behcet's
disease
d. causes hepatotoxicity as the main side
effects
e. hirsutism and gingival hyperplasia are
known side effects
10. Phlyctenular conjunctivitis:
a. the most common cause is tuberculosis
b. the lesions are typically found near the
limbus
c. should not be treated with steroid
d. predominantly affects children
e. is a type IV hypersensitivity reaction
132
11. The following are true about thiamine
deficiency:
a. external ophthalmoplegia is a typical
feature
b. failure of the pupils to respond to light
c. the condition is found in patients with
gastric carcinoma
d. post-mortem reveals perivascular
haemorrhages in the region of the fourth
ventricle and aqueduct
e. the condition responds to high dose
vitamin B12
12. Features of tentorial herniation include:
a. vomiting
b. deterioration of consciousness
c. dilated pupil
d. impaired respiration
e. hemiparesis
13. Features of Lawrence-Moon-Biedle's syndrome
include:
a. hypogonadism
b. obesity
c. polydactyly
d. mental retardation
e. pigmentary retinopathy
14. Sarcoidosis:
a. is associated with HLA B1
b. is commoner in elderly people
c. causes caseating granuloma
d. produces bilateral hilar
lymphadenopathies in the absence of
pulmonary symptoms
e. causes lymphocytosis
133
15. With regard to sickle cell disease:
a. 25% of the black population have
sickle cell disease
b. the most severe form of sickle cell
retinopathy is associated with SS
disease
c. optic disc is the first site of
neovascularization in patient with severe
retinal ischamia
d. salmon patches are the result of
ischaemia
e. choroidal ischaemia is the main cause
of visual loss
16. Alport's syndrome is associated with:
a. neuro-sensory deafness from birth
b. posterior lenticonus
c. retinal detachment
d. peripheral retinal fleck
e. haemorrhage nephritis
17. Corneal deposits are seen in:
a. Hurler's syndrome
b. Morquio's syndrome
c. myeloma
d. cystinosis
e. Sanfilippo's syndrome
18. The following are true about central retinal
artery occlusion:
a. it is a common cause of rubeosis iridis
b. can cause neovascularisation of the
retina
c. is a recognized complication of atrial
fibrillation
d. fluorescence angiography typically
shows delayed hyperfluorescence of the
choroidal circulation
e. causes optic atrophy within the first
week
134
19. In accommodative esotropia:
a. there is an increased incidence of
hypermetropia
b. the patients typically presents after the
age of 18 months
c. bifocal glasses are commonly
prescribed for these patients in the
United Kingdom
d. the binocular fusion is usually poor
e. inferior oblique overaction is a
common feature
20. With regard to the use of phopholine iodide in
the treatment of esotropia:
a. it can be used to correct hypermetropic
accommodative esotropia
b. it is used in patients with esotropia due
to high AC/A ratio
c. it can cause iris cysts the incidence of
which can be reduced with concurrent
treatment with phenylephrine
d. it causes retinal detachments
e. it should be stopped in patients about
to undergo strabismus surgery
21. In infantile esotropia:
a. amblyopia is uncommon
b. the majority of patients have
associated neurological deficit
c. a high refractive error is common
d. dissociated vertical deviation is found
in 75% of cases
e. monocular opticokinetic asymmetry is
a feature
22. The following can occur with congenital
nystagmus:
a. there is inverse opticokinetic
nystagmus
b. the nystagmus is worse with
convergence
135
c. the nystagmus is worse when one eye
is covered
d. oscillopsia is a common complaint
e. paradoxical pupillary constriction to
darkness
23. In Brown's syndrome:
a. there is downshooting of the ipsilateral
eye on adduction
b. there is associated superior rectus
overaction in the contralateral eye
c. may resolve spontaneously
d. can be treated by inferior rectus
recession in the contralateral eye
e. can be treated by inferior oblique
recession in the ipsilateral eye
24. Unilateral internuclear ophthalmoplegia is
associated with:
a. reduced adduction on the affected side
b. reduced convergence
c. nystagmus on abduction in the
affected side
d. diplopia can occur
e. there is abnormal opticokinetic
nystagmus
25. The following are features of aberrant third
nerve regeneration:
a. reduced abduction of the ipsilateral
eye
b. pupillary constriction on abduction
c. cyclotorsion on elevation
d. elevation of the contralateral lid on
lateral gaze
e. elevation of the lid on ipsilateral
136
adduction
26. The following are true about Holme-Adie's
pupil:
a. the diagnosis can be confirmed with
edrophonium
b. females are more commonly affected
than males
c. causes problem with reading that can
be corrected with plus lenses
d. constriction of pupil can occur
e. partial relative afferent pupillary
defect is seen in 50% of cases
27. Angioid streaks occur in:
a. Sturge-Weber's syndrome
b. Sickle cell disease SS
c. pseudoexfoliation syndrome
d. septo-optic dysplasia
e. Ehlers-Danlos syndrome
28. The following are true about latanaprost:
a. it is a prostaglandin inhibitors
b. it reduces the intraocular pressure by
increasing the uveal scleral outflow
c. it has a greater effect than beta
blockers in reducing the intraocular
pressure
d. hypertrichosis is a known side effects
e. it is contra-indicated in patients with
corneal graft rejection
29. Features of pigmentary glaucoma include:
a. it is commoner in females than males
b. it is a features of Chandler's syndrome
c. the pressure can be controlled with argon
137
laser trabeculoplasty
d. there is sectorial iris transillumination
e. the intraocular pressure may rise sharply
following exercise
30. In benign intracranial hypertension:
a. there is a restriction of upgaze
b. normal ventricles is found in 50% of
cases
c. brain scan is important in young
woman to exclude saggital sinus
thrombosis
d. optic nerve fenetration should be
performed early before the vision is
affected
e. ventricular-peritoneal shunt is
indicated in the majority of patients
31. The following medications may interfere with
contact lens wear:
a. rifampicin
b. contraceptive pill
c. oral penicillin
d. erythromycin
e. digoxin
32. Gout:
a. is caused by purine metabolism
disorder
b. causes scleritis
c. causes tophi in the extraocular muscle
tendon
d. is directly related to alcohol
consumption
e. uric crystals may be found in the deep
stroma
33. The following are true about iron deposition in
the cornea:
138
a. Fleischer's ring is found at the apex of
keratoconus
b. Hudson-Stahli line is found in
recurrent erosion syndrome
c. Stocker's line is associated with
pterygium
d. Ferry's line is found at the front of a
trabeculectomy bleb
e. iron deposition occurs in radial
keratotomy
34. The following are true about corneal
grafts:
a. zenografts refers to transplant between
different species
b. isografts refers to transplant between
the same species
c. lamellar keratoplasty is useful in
patient with Fuch's dystrophy
d. previous blood transfusion increases
the incidence of rejection
e. broken corneal suture can elicit
rejection
35. Features of Goldenhar's syndrome include:
a. unilateral epibulbar dermoids
b. presence of preauricular skin tag
c. presence of syndactyly
d. increased incidence of angle closure
glaucoma
e. loss of pigments on eyelashes
36. Red-green colour defect is found in:
a. optic neuritis
b. Stargardt's disease
c. tobacco amblyopia
d. macular degeneration
e. primary open angle glaucoma
139
37. The following clinical features are
suggestive of a cavernous sinus lesion in a
patient with unilateral third nerve palsy:
a. dilatation of the pupil
b. impaired ipsilateral corneal reflex
c. constriction of pupil
d. failure of the eye to abduct
e. hearing loss
38. Features of carotid-cavernous fistula include:
a. dilated superior ophthalmic vein on
CT scan
b. blood in the Schlemmn's canal
c. proptosis of contralateral eye suggests
bilateral carotid-cavernous fistula
d. dural fistula is the commonest type
seen following head injury
e. traumatic fistula rarely close
spontaneously
39. The following conditions are X-linked:
a. Fabry's disease
b. Refsum's disease
c. Norries's disease
d. choroideremia
e. rod monochromatism
40. In mesodermal dysgenesis:
a. Axenfeld's syndrome has posterior
embryotoxon and iris hypoplasia
b. Axenfeld's syndrome is inherited as an
autosomal recessive disorder
c. Rieger's anomaly is associated with
dental and facial hypoplasia
d. Peter's anomaly is associated with
bilateral corneal opacities
e. Peter's anomaly is associated with
glaucoma in 50% of cases
41. In lattice degeneration:
140
a. 8% of the population have the
condition
b. photocoagulation is recommended for
extensive cases
c. holes should be treated
d. it has a higher incidence amongst high
myopes
e. 25% of retinal detachment is due to
lattice degeneration
42. The indication for subretinal fluid drainage
include:
a. immobilized retina
b. bullous lesion where tear cannot be
identified
c. hypotony
d. superior retinal detachment
e. presence of blood in the vitreous
43. The following are true about intravitreal
injection:
a. it can cause secondary retinal tear
b. it can cause cataract
c. C3F8 has a longer half life than SF6
d. SF6 is more expansile than C3F8
e. SF6 causes less increase in the
intraocular pressure than C3F8
44. In ptosis operation:
a. brow suspension is the treatment of
choice in severe congenital ptosis
b. Fasavella-Servant is the treatment of
choice in patients without levator
function
c. posterior approach is the
recommended procedure in patient with
previous ptosis surgery
d. maximal resection is recommended in
patients with mitochondrial myopathy
e. Bell's phenomenon and corneal
141
sensation should be carefully assessed
before surgery
45. Fourth nerve palsy can be treated with:
a. ipsilateral superior recession
b. ipsilateral inferior oblique recession
c. contralateral inferior rectus resection
d. ipsilateral superior oblique tucking
e. ipsilateral inferior rectus recession
46. The following are true about corneal
blood staining:
a. it occurs in about 5% of hyphema
b. it is related to the severity of the
hyphaema
c. it is commoner in decompensated
cornea
d. it is caused by red blood cells
migrating into the stroma
e. spontaneous clearing of the hyphema
typically begins centrally
47. In acute bacterial endophthalmitis
following cataract operation:
a. Staphylococcus aureus is the most
common pathogen
b. presents within 48 hours of surgery in
the majority of cases
c. topical and systemic antibiotic are
usually adequate to control the infection
d. steroid is contraindicated
e. it is often difficult to differentiate
bacterial from mycotic causes
48. In peripheral iridotomy with laser:
a. dark colour iris responds poorly to
argon laser iridotomy
b. more energy is needed for argon laser
142
than YAG laser for a given size
iridotomy
c. steroid pre-treatment is effective in
decreasing the intraocular pressure
d. presence of red reflex during the
procedure indicate the iridotomy is
patent
e. significant cataract develops in 50% of
cases following treatment
49. Unsuitable donor for corneal graft
include:
a. a history of conjunctivitis
b. history of hepatitis A
c. less than 10 years old
d. endothelial cell count of less than
2000 per square mm
e. senile dementia
50. In argon laser trabeculoplasty:
a. the success rate is proportional to the
energy used
b. the results are better in the elderly than
the young
c. the intraocular pressure is reduced due
to decreased aqueous production
d. typically produce a drop in the
intraocular pressure of between 10 and
15 mmHg
e. it is used as an adjunctive treatment to
maximal medical therapy
51. In refractive surgery;
a. radial keratotomy may be used in
myopes of 10D or more
b. radial keratotomy is a well-established
safe techniques with predictable result
c. post-operative astigmatism may be
reduced by removing the suture along the
axis of the largest plus cylinders
d. post-operative astigmatism may be
reduced by removing sutures in the
143
flatter axis
e. keratomileusis involves using a donor
cornea
52. The following techniques are useful in reducing
post-operative astigmatism:
a. phacoemulsification rather than
extracapsular technique
b. the use of foldable lens
c. a limbal incision rather than a corneal
incision
d. superior corneal approach rather than
temporal approach
e. divide and conquer technique during
phacoemulsification rather than phacochop
53. The following are true about anterior
capsule removal in cataract surgery:
a. capsulorrhexis produces a stronger
edge than can-opener capsulotomy
b. capsulorrhexis requires less
viscoelastic than can-opener
capsulotomy
c. displacement of the intraocular lens is
less common with capsulorrhexis than
can-opener capsulotomy
d. hydrodissection of the lens is not
required with capsulorrhexis
e. capsulorrhexis is an easier technique
to master than can-opener capsulotomy
54. Inferior oblique recession:
a. need careful prism measurement
b. is performed in ipsilateral fourth nerve
palsy to remove cyclotorsion
c. is performed in V-pattern esotropia
d. is performed in contralateral superior
rectus palsy
144
e. is easier to perform than inferior
oblique myectomy
55. In unrecovered sixth nerve palsy:
a. surgery can speed up recovery
b. botulinum toxin injection into the
ipsilateral medial rectus speeds up
recovery
c. resection/recession is in appropriate
d. no more than two muscles should be
operated
e. can be treated with Faden's procedure
on the contralateral medial rectus
56. The following are true about cystoid macular
oedema:
a. it causes visual impairment in 10% of
uncomplicated extracapsular cataract
operation
b. the incidence of cystoid macular
oedema is higher in intracapsular
cataract extraction than extracapsular
cataract operation
c. the risk of cystoid macular oedema is
increased in patients who have postoperative corneal oedema
d. vitreous prolapsed is a known
precipitating factor for cystoid macular
oedema
e. malpositioned of intraocular lens is a
recognized factor for cystoid macular
oedema
57. The following are true about the classification
of retinopathy of prematurity:
a. Stage 1: normal retina
b. Stage 2: presence of demarcation lines
that protrude into the vitreous
c. Stage 3: presence of fibrovascular
145
proliferation in the peripheral retina
d. Stage 4: total retinal detachment
e. Stage 5: proliferative vitreous
retinopathy
58. The following are true about pseudoexfoliation
syndrome:
a. the age when glaucoma develops is
older than primary open angle glaucoma
b. the intraocular pressure always
responds better to medical treatment than
primary open angle glaucoma
c. asymmetrical glaucoma is more
common than primary open angle
glaucoma
d. there is an increased pigment
deposition in the trabecular meshwork
e. trabeculectomy is less successful in
controlling the pressure than in primary
open angle glaucoma.
59. In radiation retinopathy:
a. the main pathology is occlusive
microangiopathy
b. the lowest dose of radiation required
to cause radiation retinopathy is 11 Gy
c. the photoreceptors are more sensitive
to radiation than the retinal vascular cells
d. patients on chemotherapy are more
vulnerable to radiation retinopathy
e. hyperbaric oxygen is useful in
preventing the progression of radiation
retinopathy
60. The following are true about local anaesthesia:
a. peribulbar anaesthesia requires a
longer time than retrobulbar anaesthesia
to achieve akinesia
b. lignocaine requires a shorter time than
marcaine to achieve anaesthesia
c. lignocaine has a longer duration of
146
action than marcaine
d. topical amethocaine is inadequate for
iris anaesthesia
e. retrobulbar haemorrhage increases the
risk of expulsive haemorrhage
Test 12 (neuro-ophthalmology)
1. The following are true about neuroretinitis caused by cat-scratch
fever:
a. the pathogen is Bartonella henselae
b. the disease is usually acquired from cat under one year
of age
c. skin test is commonly used to diagnose the condition
d. lymphadenopathy is common
e. low-grade fever is usually present
a.T b.T c.F d.T e.T
2. Neuroretinitis from cat-scratch fever:
a. shows optic nerve swelling
b. macular star is typical
c. the vision usually returns to normal even without
treatment
d. antibiotics shorten the course of the disease
e. is usually bilateral
a.T b.T c.T d.T e.F
147
3. The following are true about cat-scratch fever:
a. it is caused by a Gram positive bacillus
b. caseating granulomas are seen in lymph node
c. in patients who develop neuroretinitis, visual loss
is typically preceded by systemic illness
d. it is a cause of Parinaud's oculoglandular fever
e. an aching neck with cough or movement is common
a.F b.T c.F d.T e.T
4. Cecocentral scotoma occurs in:
a. vitamin A deficiency
b. vitamin B12 deficiency
c. folate deficiency
d. thiamine deficiency
e. vitamin E deficiency
a.F b.T c.T d.T e.F
5. The following are features of nutritional optic neuropathy:
a. temporal pallor of the optic discs
b. relative afferent pupillary defect
c. constricted visual fields
d. loss of the fibres in the papillomacular bundle.
e. colour vision loss
a.T b.F c.F d.T e.T
148
Test 13 (uveitis)
1. The following are true about TINU (tubulointerstitial nephritis
uveitis) syndrome:
a. the median age is 15 years of age.
b. females are more frequently affected than males.
c. sarcoidosis is the most common cause
d. the uveitis typically occurs before the renal problems
e. the most common type of uveitis in TINU syndrome
is bilateral anterior uveitis
a.T b.T c.F d.F e.T
2. The following are true about TINU syndrome:
a. ibuprofen is associated with TINUS
b. the uveitis must occur within 12 months of the acute
interstitial nephritis.
c. permanent visual loss occurs in 25% of patients
d. the renal lesion is characterized by T-cell infiltration.
e. chronic renal disease is uncommon
a.T b.T c.F d.T e.T
3. The following are commonly seen in TINU syndrome:
a. anaemia
b. raised ESR
c. elevated immunoglobulins
149
d. elevated creatinine
e. increased urinary beta-2-microglobulin.
a.T b.T c.T d.T e.T
4. The following HLA types are associated with increased risk
of uveitis:
a. HLA B51
b. HLA B7
c. HLA A29
d. HLA DR3
e. HLA DR2
a.T b.T c.T d.T e.T
5. The following conditions can give rise to both uveitis and
erythema nodosum :
a. Crohn's disease
b. Behcet's disease
c. sarcoidosis
d. rheumatoid arthritis
e. systemic lupus erythematosus
a.T b.T c.T d.F e.T
More MCQs
Test 14 (neuro-ophthalmology)
1. The following signs and symptoms are useful for differentiating
150
between papilloedema and optic neuritis:
a. central scotoma.
b. decreased visual acuity
c. optic disc swelling
d. history of recurrent transient arm weakness
e. pain behind eye
a.T b.T c.F d.T e.T
2. In a patient with optic neuritis, the following signs are suggestive of
multiple sclerosis:
a. muscle weakness after taking a hot bath
b. difficulty in walking heel to toe
c. urine incontinence
d. wasting of small muscles of the head
e. nystagmus
a.T b.T c.T d.F e.T
3. Syringobulbia causes:
a. nystagmus
b. dysphagia
c. tongue wasting
d. bilateral Babinski's signs
e. impaired perception of pain and temperature with
preserved light touch and propioception in the upper
extremities
a.T b.T c.T d.T e.T
4. In a patient with optic atrophy, the following signs are useful in
151
differentiating subacute combined degeneration of the cord from
multiple sclerosis:
a. absent ankle jerk
b. bilateral Babinski signs
c. sensory loss in a stocking distribution
d. macrocytic anaemia
e. ataxia
a.T b.F c.T d.T e.F
5. In a patient with ptosis and diplopia, the following findings
are suggestive of myasthenia gravis :
a. symmetrical involvement
b. thymoma on CT chest scan
c. increased serum creatinine phosphokinase
d. improved lid movement after applying ice cube
to the lid
e. absent tendon reflexes
a.F b.T c.F d.T e.F
Test 15 (optic discs)
152
Refer to this picture for question 1, 2 & 3
1. The following conditions are associated with the picture above:
a. pseudoxanthoma elasticum
b. retinitis pigmentosa
c. age-related macular degeneration
d. retinal detachment
e. retinoschisis
a.T b.T c.F d.F e.F
2. The following are true:
a. the lesions are situated within the myelinated part of
the optic disc
b. they occur more commonly in patients with small discs
c. they can be inherited as a autosomal recessive trait
d. ultrasound is useful for diagnosis
e. MRI is useful for diagnosis.
a.F b.T c.F d.T e.F
3. Visual loss can result from
a. optic atrophy
b. retinal detachment
153
c. papilloedema
d. venous occlusion
e. peripapillary choroidal neovascularization.
a.T b.F c.F d.T e.T
Refer to the picture above for question 4.
4. Ocular association include:
a. lens coloboma
b. foveal hypoplasia
c. serous macular detachment
d. aniridia
e. persistent hyperplastic primary vitreous
a.T b.T c.T d.T e.T
Refer to the picture above for question 5
154
5. Conditions associated with the picture include :
a. trisomy 22
b. trisomy 21
c. trisomy 18
d. trisomy 13
e. Turner's syndrome
a.T b.F c.T d.T e.F
Test 16 (ocular parasitology)
Refer to the picture above for question 1, 2 & 3
1. This parasite is found in the following regions of the world:
a. Far East
b. South America
c. Southern Europe
d. South Africa
e. West Africa
a.F b.T c.F d.F e.T
2. The vector for the parasite above is:
155
a
b
c
d
e.
a.F b.F c.T d.F e.F
3. Causes of blindness include:
a. trichiasis
b. keratitis
c. optic neuritis
d. chorioretinitis
e. chronic non-granulomatous iridocyclitis
a.F b.T c.T d.T e.T
156
Refer to the above picture for question 4, 5 & 6
4. The above patient has been doing some country walking recently,
which pest(s) are likely to be the cause of this skin lesion:
a
b
c
d
e
157
a.F b.T c.F d.T e.F
5. The patient develops arthralgia and conjunctivitis a few weeks
later. Which organism is the most likely cause? :
a.
b.
c.
158
d.
e.
a.T b.F c.F d.F e.F
6. The following are true about the ocular manifestations:
a. it is commoner in Europe than in North America
b. conjunctivitis is commoner than uveitis
c. the uveitis is typically non-granulatomatous
d. uveitis usually occurs in the first stage of the
disease
e. systemic antibiotic has no effect on the course of the
ocular disease
a.T b.T c.F d.F e.F
159
Test 17 (paediatric ophthalmology)
Refer to the picture above for question 1.
1. The organisms responsible for the above appearance include:
a.
160
b.
c.
d.
e.
a.T b.T c.F d.T e.F
161
Refer to this picture for question 2.
2. The following conditions are associated with the picture above:
a.
b.
162
c.
d.
e.
a.T b.T c.F d.T e.T
163
Refer to the picture above for question 3.
3. Related features in this boy include::
a.
b.
c.
164
d.
e.
a.F b.T c.F d.T e.F
Refer to this picture for question 4.
4. Associated features include:
a.
165
b.
c.
d.
e.
a.T b.T c.F d.F e.T
166
Refer to the picture above for question 5.
5. The following investigation(s) is/are useful for confirming
the diagnosis:
a.
b.
c.
d.
167
e.
a.F b.F c.T d.F e.F
An albino retina vs a normal retina.
Note the absence of pigments in the RPE.
Test 17 (paediatric ophthalmology)
168
Refer to the picture above for question 1, 2 & 3.
1. You are asked to examine the fundi of a baby with suspected
non-accidental injury and find the above appearance.
The following are possible causes:
a. coagulopathy
b. normal vaginal delivery
c. proliferative diabetic retinopathy
d. glutaric aciduria
e. optic nerve glioma
a.T b.T c.F d.T e.F
2. The paediatrician suspects shaken baby syndrome and order some
radiological investigations. The following pictures are consistent
with the suspicion:
a.
169
b.
c.
d.
170
e.
a.T b.T c.T d.F e.F
3. The final diagnosis was shaken baby syndrome. The following are
true about the ocular features:
a. retinoschisis is a cause of ocular scarring
b. the retinal haemorrhages are confined to
the superficial layer
c. retinal haemorrhages are necessary for the diagnosis
of shaken baby syndrome
d. the main cause of visual loss in this condition is retinal
haemorrhages.
e. ERG is useful in distinguishing between ocular and
brain causes of visual loss.
a.T b.T c.T d.T e.T
171
This is the fundal appearance of a child with
mental retardation. Refer to this picture
for question 4 & 5.
4. The mother of this child may have the following skin lesion
in her first trimester:
a.
b.
c.
d.
172
e.
a.F b.T c.F d.F e.F
5. The following signs may be seen:
a.
b.
c.
173
d.
e.
a.T b.F c.T d.T e.T
'
Test 17 (phakomatoses)
Refer to the pictures above for question 1 & 2
1. Other ocular manifestations of the above condition
include:
174
a.
b.
.
c.
d.
175
e.
a.F b.T c.T d.T e.F
2. Other manifestations of this condition include:
a.
b.
176
c.
d.
e.
a.F b.T c.T d.T e.F
177
Refer to the picture above for question 3
3. Systemic associations include:
a.
b.
178
c.
d.
e.
a.F b.T c.T d.T e.F
Refer to the above picture for question 4
4. The following are associated features:
179
a.
b.
c.
d.
180
e.
a.T b.T c.F d.F e.T
Refer to the picture above for question 5
5. Associated features for the above condition include :
a.
b.
181
c.
d.
e.
a
a.T b.F c.T d.T e.F
More MCQs
Test 20 (systemic diseases and ophthalmology)
182
Refer to this picture for question 1.
1. The following can precipitate or result from the condition shown in
the picture above:
a.
b.
c.
183
d.
e.
a.T b.T c.T d.F e.T
Refer to these picture for question 2.
The figures show the same patient
before and after an intravenous injection.
2. The following can be associated with the pictures above:
184
a.
b.
c.
185
d.
e.
a.T b.T c.T d.T e.T
Refer to this picture for question 3.
a
3. The following are associated with the sign shown above:
186
a.
b.
c.
d.
187
e.
a.T b.T c.T d.T e.F
e
d
Refer to this picture for question 4.
4. Systemic association include:
a.
188
b.
c.
d.
e.
a.T b.T c.T d.T e.T
189
Refer to the picture above for question 5.
5. The child may have the following additional signs:
a.
b.
c.
190
d.
e.
a.T b.T c.T d.F e.T
More MCQs
Test 21 (systemic diseases)
Refer to this picture for question 1.
191
1. Related features include:
a.
b.
c.
d.
192
e.
a.T b.T c.T d.T e.T
Refer to this picture for question 2.
2. Blood films related to the above appearance include:
a.
193
b.
c.
d.
e.
a.T b.F c.T d.T e.T
194
Refer to this picture for question 3.
3. The above condition can be caused by:
a.
b.
c.
195
d.
e.
a.T b.T c.T d.T e.T
Refer to the picture above for question 4.
4. Recognized causes for the above condition include:
a.
196
b.
c.
d.
197
e.
a.T b.T c.T d.T e.T
Refer to the picture above for question 5.
5. Causes of the above appearance include:
a.
198
b.
c.
d.
199
e.
a.F b.F c.F d.F e.F
More MCQs
Test 22 (neuro-ophthalmology)
Refer to this picture for question 1.
1. The following signs may occur in this patient:
a.
200
b.
c.
d.
e.
a.T b.F c.T d.F e.F
201
Refer to this picture for question 2
2. Condition that can give rise to this picture include:
a.
b.
202
c.
d.
e.
a.T b.T c.F d.F e.F
203
...
The two pictures (before and after treatment) were taken
about 20 years apart.
3. The following are true:
a. craniotomy scar is likely to be present
b. abdominal scar is likely to be present
c. if he fails to take his medication,
hypotension and hyperkalaemia will occur.
d. bitemporal hemianopia may be present
e. junctional scotoma may be seen
a.F b.T c.T d.T e.T
Refer to the picture above for question 4.
4. The right pupil is dilated and the result of the swinging
light test is as shown above. The following may
account for the findings:
a. the examination is normal
b. the patient may have a right optic atrophy
c. the patient may have a left optic atrophy
204
d. the patient may have advanced left
glaucoma
e. the patient may have a right macular hole
a.F b.T c.F d.F e.F
Refer to this picture for question 5.
5. Drugs which can precipitate myasthenia gravis include:
a.
b.
205
c.
d.
e.
a.T b.T c.T d.F e.T
More MCQs
206
Test 23 (anterior segment)
a.
1.
b.
c.
d.
e.
f.
2.
3.
4.
5.
6.
Match the clinical pictures (in alphabets) with the appropriate histological changes (in numbers).
a.
1.
b.
2.
c.
3.
207
d.
4.
e.
5.
f.
6.
a=4; b=3; c=6; d=1; e=2; f=5.
More MCQs
Test 24 (eyelids)
a.
b.
c.
d.
208
e.
f.
1.
2.
3.
4.
5.
6.
Match the eyelid skin lesions (in alphabets) with their histological appearance (in numbers)
a.
1.
b.
2.
c.
3.
d.
4.
209
e.
5.
f.
6.
a=4; b=5; c=6; d=2; e=1; f=3
More MCQs questions
Test 25 (conjunctiva)
a.
b.
c.
d.
e.
1.
2.
3.
4.
5.
Match the conjunctival disorders (in alphabets) with their histological
appearance (in numbers).
210
a.
1.
b.
2.
c.
3.
d.
4.
211
e.
5.
a=5; b=1; c=2; d=3; e=4.
Test 26 (neuro-ophthalmology)
1. The following are features of melanoma-associated retinopathy:
a. central scotoma
b. night vision impairment
c. abnormal electroretinogram
d. antibody to rods
e. presence of distant skin melanoma rather than
choroidal melanoma
a.F b.T c.T d.T e.T
2. Features of cancer-associated retinopathy include:
a. narrowing of the retinal artery
b. abnormal electroretinogram
c. stationary bilateral visual loss
212
d. optic atrophy
e. the most common underlying cause is breast carcinoma
a.T b.T c.F d.T e.F
3. In Leber's hereditary optic neuropathy:
a. visual recovery is usually poor
b. peripapillary telangiectasia is a late feature of
the disease
c. 14484 mutation at the mitochondria is associated
with a better visual prognosis
d. 17788 mutation is the most common mitochondrial
mutation seen in this condition
e. only male patients are affected.
a.T b.F c.T d.T e.F.
4. The following are true about ocular myasthenia gravis:
a. thymomectomy is usually performed to improve
the ocular signs
b. application of ice to a ptotic eye may worsen the
symptom
c. the majority of myasthenic patients have ocular
involvement
d. squint operation should never be carried out on
myasthenic patients
e. single fibre electromyography of the orbicularis
oculi may show increased jitter
a.F b.F c.T d.F e.T
5. True statements about non-arteritic ischaemic optic neuropathy
213
include:
a. more common in males than females
b. headache is not a feature
c. it is associated with an increased incidence of
hypertension and diabetes mellitus
d. optic nerve fenestration is beneficial
e. aspirin can improve the visual prognosis of the affected
eye
a.T b.T c.T d.F e.F
More MCQs
Test 27 (Medical retina)
1. Differentiating classic and occult neovascular membranes is
possible using:
a. Amsler grid
b. indocyanine green
c. fundus fluorescein angiography
d. optical coherence tomography
e. B-scan ultrasound
a.F b.T c.T d.F e.F
2. The following are true about choroidal neovascular membranes
(CNV):
a. subfoveal CNV is best treated with direct focal argon
laser
214
b. juxtafoveal CNV is 0-200um from the foveal avascular
zone
c. extrafoveal CNV is more than 200um from foveal
avascular zone
d. optic nerve pit is a cause of CNV
e. optic disc drusen is a cause of CNV
a.F b.T c.T d.F e.T
3. The following statements are true regarding dry age-related macular
degeneration (AMD):
a. it will typically have a visual acuity of 6/60 or worse
within 2 years
b. the optimal management is with low visual aids
c. the contralateral eye will be simultaneously affected in
all cases
d. the patient should be promptly referred for fluorescein
angiography
e. dry AMD can be treated with photodynamic therapy
without verteporfin
a.F b.T c.F d.F e.F
4. Photodynamic therapy (PDT) is useful for the following conditions
resulting from age-related macular degeneration :
a. retinal pigment epithelium detachment
b. disciform macular degeneration
c. geographic atrophy
d. occult subfoveal neovascular membrane
215
e. classic subfoveal neovascular membrane
a.F b.F c.F g d.T e.T
5. The following are true about verteporfin used in photodynamic therapy
(PDT):
a. verteporfin is activated by green laser light
b. verteporfin is a derivative of prophyrin
c. it generates free radicals which causes blood clotting
local to the lesion
d. it destroys the new vessels through heat
e. it is contraindicated in pseudophakia
a.F b.T c.T d.F e.F
More MCQs
Test 28 (Uveitis)
For more information on the topics below, click here to download
an update on uveitis by the American Academy of Ophthalmology.
1. The following medications are effective against toxoplasma tissue
cysts:
a. clindamycin
b. pyrimethamine
c. sulphonamides
d. azithromycin
e. atovaquone
a.F b.F c.F d.T e.T
216
2. In an immunocompromised patient with retinitis, a diagnosis of
toxoplasma retinitis is favoured over cytomegalovirus (CMV)
retinitis by the presence of:
a. ocular pain
b. dense vitreous reaction
c. retinal haemorrhages
d. granulomatous anterior chamber reaction
e. scattered small lesion throughout the posterior segment
a.T b.T c.F d.T e.T
3. The following are true about active ocular toxoplasmosis:
a. the antibodies are always present
b. the central nervous system should be evaluated for
toxoplasmosis in all immunocompromised patients
c. indocyanine green can be used to detect clinically
invisible active satellite lesions
d. the border of the lesion is sharply defined
e. it is the commonest cause of retinitis in AIDS patients
a.F b.T c.F d.F e.F
4. The following drugs are known to cause uveitis :
a. rifabutin
b. cidofovir
c. methotrexate
d. latanoprost
217
e. chloramphenicol
a.T b.T c.F g d.T e.F
5. HAART (Highly Active Anti-virus Therapy)
a. uses a combination of antiretroviral therapy
b. decreases the incidence of CMV retinitis
c. reduces the need for long-term maintenance therapy
in patients with established CMV retinitis
d. increases the number of CD4+ lymphocytes
e. causes uveitis during treatment
a.T b.T c.T d.T e.T
More MCQs
Test 29 (Herpetic eye disease)
1. The following are true about herpes simplex:
a. herpes keratitis in infants is caused mainly by type II
herpes simplex
b. ocular disease is the most common manifestation of
primary type I herpes simplex
c. herpes simplex can be contracted from pests
d. ocular recurrence is usually caused by virus
reactivation within the trigeminal ganglia
e. ocular recurrence is more common than either genital
or oral recurrence
a.T b.F c.F d.T e.F
218
2. With regard to herpes epithelial ulcer:
a. the sensation is reduced
b. the disease usually presents initially as punctate keratitis
c. geographical ulcer occurs as a result of inappropriate
use of topical steroid
d. the ulcer has typical tapering endings
e. Rose Bengal stains infected epithelial cells
a.T b.T c.T d.F e.T
3. The following are true about acyclovir
a. topical acyclovir is more effective than oral acyclovir in
the treatment of herpes simplex epithelial keratitis
b. valacyclovir is the prodrug of acyclovir
c. the dosing frequency is less with valacyclovir than
acyclovir
d. oral acyclovir reduces the incidence and severity of
postherpetic neuralgia
e. oral acyclovir given daily can reduce the recurrent of
herpes simplex epithelial keratitis
a.F b.T c.T d.F e.T
4. In the treatment of herpes simplex ocular disease :
a. oral acyclovir has no effect on the development of
stromal keratitis in the subsequent year when given
during active herpes simplex keratitis
b. oral acyclovir reduces the development of iritis in the
subsequent year when given during active herpes
219
simplex keratitis
c. in patients with previous herpes simplex stromal
keratitis, oral acyclovir reduces the likelihood of
recurrent stromal disease
d. stopping oral acyclovir in a patient with previous
herpes simplex stromal keratitis causes rebound of
herpes simplex disease
e. oral acyclovir has no effect on herpes simplex iritis
a.T b.F c.T d.F e.F
5. In the treatment of herpes simplex stromal disease, HEDS (Herpetic
Eye Disease Studies) show that:
a. topical steroid reduces the duration of the disease
b. patients on topical steroid has better visual outcome at
six months review than the non-steroid treated group
c. topical steroid increases the likelihood of epithelial
disease
d. systemic steroid is useful in the management of
necrotizing stromal keratitis
e. oral acyclovir has added benefit in patients treated with
topical steroid and trifluridine
a.T b.F c.F d.F e.F
More MCQs
Test 30 (Neuro-ophthalmology)
1. Vertical nystagmus:
a. occurs in phenytoin toxicity
b. in an unconscious patient suggests thalamic lesion
220
c. can present with oscillopsia
d. if upbeating indicates cerebellar tonsillar ectopia
e. if downbeating indicates a lesion in the pons
a.T b.F c.T d.F e.F
2. The following are true about blepharospasm:
a. it is commoner in males than females
b. a brain scan should always be performed
c. it is often caused by compression of the facial
nerve by intracranial artery
d. orbicularis oculi stripping is the treatment of choice
e. neuroleptic drug is a common cause
a.F b.F c.F d.F e.F
3. In Creutzfeldt-Jakob disease (CJD):
a. the protein content of the cerebrospinal fluid is raised
b. EEG shows characteristic waveform
c. death usually occurs within 12 months of onset
d. cortical blindness is common
e. the causative agent is a retrovirus
a.F b.T c.T d.T e.F
4. In multiple sclerosis :
a. onset with optic neuritis or sensory symptoms tend to
have better prognosis
221
b. low amplitude of visual evoked potential strongly
support optic nerve involvement
c. CSF lymphocytosis is a feature
d. oligoclonal band in the CSF supports the diagnosis
e. the prognosis is worse for males than females
a.T b.F c.F d.T e.T
5. In Bell's palsy:
a. mild pain in the mastoid area is common at
the onset
b. corneal sensation is reduced
c. the blink reflex is impaired
d. complete facial palsy is associated with poor prognosis
e. electrophysiological testing is useful in predicting
prognosis
a.T b.F c.T d.T e.T
More MCQs
Test 31 (glaucoma)
222
Refer to question 1
1. The above picture is taken from a patient in his mid-20s.
The following are true:
a. the iris is likely to be convex
b. there is posterior insertion of the iris root
c. lattice degeneration is common
d. the sex distribution of this condition is the same
e. the risk of glaucoma development is commoner in
females than males
a.F b.T c.T d.T e.F
Refer to question 2
223
2. The above is an appearance obtained through gonioscopy:
a. the angle is at risk of closure
b. there may be peripupillary transillumination
c. the patient may be pseudophakic
d. the pressure is likely to be above 21mmHg
e. the patient may be a poorly controlled diabetic
a. F b.T c.T d.F e.T
Refer to question 3
3. The following are true with regard to the picture above:
a. the patient may be of tall stature and myopic
b. deafness may be present
c. there may be arachnodactyly
d. there is a risk of lens dislocation
e. if the pressure is raised, the treatment of choice is
intensive pilocarpine
224
a.F b.T c.F d.T e.F
Question 4
4. Regarding the above picture:
a. it is obtained with OCT (optical coherence
topography)
b. it is obtained with the use of laser
c. it is used to analyse nerve fibre layer thickness
d. the blue area has more nerve fibres than the red area
e. it can be used to diagnose glaucoma on its own
a.F b.T c.T d.F e.F
225
Refer to question 5
5. With regard to the above visual field results:
a. it is obtained with octopus perimeter
b. the test uses yellow light
c. the M cells are tested
d. it tests neurones that are sensitive to low-contrast
stimulus
e. it can theoretically detect glaucoma at an early stage
a.F b.F c.T d.T e.T
More MCQs
Test 32 (Thyroid eye disease)
1. Thyroid eye disease is exacerbated by:
a. radio-iodine treatment of hyperthyroidism
b. development of hypothyroidism after treatment of
hyperthyroidism
226
c. smoking
d. male sex
e. presence of diabetes mellitus
a.T b.T c.T d.T e.F
2. Upper eyelid retraction in thyroid eye disease is caused by:
a. adrenergic stimulation of the Muller's muscle
b. superior oblique muscle fibrosis
c. inferior rectus fibrosis
d. fibrosis of the levator
e. fatty infiltration of the levator
a.T b.F c.T d.T e.F
3. Thyroid eye disease:
a. is seen in 40% of patients with Graves' disease
b. does not occur in Hashimoto's thyroiditis
c. is not a feature of thyroid cancer
d. tends to peak 6 months from onset
e. usually cease to be active 18 months after onset
a.T b.F c.T d.T e.T
4. The following are true about the radiological features of
thyroid eye disease:
a. inferior and lateral recti are the two extraocular muscles
most often involved
227
b. the tendon of the affected muscle is not involved
c. medial rectus enlargement is more likely than inferior
rectus enlargement in causing compressive optic
neuropathy
d. oedema of the extraocular muscle is best diagnosis with
B-scan
e. STIR-sequence MRI is useful in assessing the activity of
thyroid eye disease
a.F b.T c.T d.F e.T
5. The following are true about thyroid eye disease:
a. the orbital tissue is infiltrated by B cells during
the active phase
b. when performing orbital decompression for
optic neuropathy, the medial wall should be
removed
c. lacrimal gland enlargement is common
d. bicoronal flap is the best approach for orbital
decompression
e. diplopia is a common complication of orbital
decompression
a.F b.T c.F d.F e.T
More MCQs
Test 33 (External eye disease)
228
1. This patient has this appearance in both eyes, the following are true:
a. the condition is autosomal dominant
b. vision is rarely affected until middle age
c. recurrent erosion affects more than 50% of the patients
d. the endothelium is normal
e. the lesion is best stained with alcian blue
a.T
b.T c.F d.T e.F
2. This patient underwent a corneal procedure recently and complains
of photophobia and pain. The opacities do not stain with
fluorescence. The likely procedure is:
229
a. excimer laser
b. EDTA removal of band keratopathy
c. radial keratotomy
d. LASIK
e. micropuncture of the cornea
a.F b.F c.F d.T e.F
3. The above picture is taken of a patient who has had a corneal
graft. The following are true:
a. the picture shows an early sign of graft rejection
b. the graft is at risk of dysfunction unlike treated with
intensive steroid
c. the line seen above is located in the posterior stroma
of the cornea
d. untreated, the line will move across the cornea
e. the line is made up of aggregates of macrophages
a.F b.T c.F d.T e.F
230
4. This patient presents with bilateral photophobia without
conjunctivitis. No history of infective conjunctivitis is known.
Fluorescence shows epithelial stainings. The slit-lamp features
are as shown above. The following are true:
a. anterior uveitis is likely to be present
b. vision is likely to be impaired due to corneal scarrings
c. the corneal sensation is reduced
d. topical acyclovir is useful
e. topical steroid is useful
a.F b.F c.F d.F e.T
5. This patient complains of foreign body sensation and photophobia.
There are no signs of cells in the anterior chamber or foreign body.
The following is true:
231
a. the patient usually have bilateral red eyes in primary position
b. the patient is likely to be female than male
c. follicular conjunctivitis of the upper lid is a feature
d. the cornea is not affected
e. RGP (rigid gas permeable) contact lens wear is a cause
a.F b.T c.F d.F e.T
MCQs on Uveitis
1. The matching of the following diseases and their HLA association are true:
a. sympathetic ophthalmitis
b. par planitis
HLA-A11
HLA-DR2
c. cicatricial pemphigoid
HLA-B5
d. Posner-Schlosmann syndrome
e. Vogt-Koyanagi-Harada
1.
a.T
b.T
c.F
d.T
HLA-BW5
HLA-B22
e.T
2. Severe vitritis is a feature of:
a. cytomegalovirus retinitis
b. serpigionous choroidpathy
c. presumed ocular histoplasmosis
232
d. acutre retinal necrosis
e. toxoplasma retinochoroidopathy
2.
a.F b.F c.F d.T e.T
3. In ocular toxocarisis:
a. a history of visceral larva migrans is always present
b. examination of the patient's stool is useful in diagnosing
the disease
c. bilateral ocular involvement should make one doubt the
accuracy of the diagnosis
d. ultrasound of the granuloma typically shows calcification
e. oral thiabendazole is useful
3.
a.F
b.F c.T d.F e.F
4.In juvenile chronic arthritis-related uveitis:
a. the uveitis is usually granulomatous
b. the eyes are typically white
c. the uveitis usually precedes arthritis
d. it is associated with positive rheumatoid factor
e. cystoid macular oedema is an important cause of poor
vision
4.
a.F b.T c.F
d.F e.F
5. The following are true with regard to cataract in juvenile chronic arthritis:
a. cataract should not be performed before the patients is
8 year-old
b. operation should be performed only when the anterior
chamber is free of flare
233
c. removal of the cataract should be combined with
anterior vitrectomy
d. hypotony is a known post-operative complication in
these patients
e. only heparin-coated intraocular lens should be
implanted.
5.
a.F
b.F c.T d.T
e.F
6. The following are true about cytomegaloviral retinitis:
a. it is rarely seen in patients with a CD 4+ counts of
more than 50 cells/ul
b. it causes full thickness retinal necrosis
c. frosted branch retinitis is a known clinical feature
d. spontaneous resolution in about 30% of patients
without treatment
e. anti-CMV antibody is useful for diagnosis
6.
a.T b.T c.T
d.F e.F
7. In the treatment of CMV retinitis:
a. ganciclovir, foscanet and cidofovir are all virostatic
b. ganciclovir causes bone marrow suppression
c. foscanet is associated with renal impairment
d. cidofovir is associated with hepatic dysfunction
e. recurrent CMV retinitis is rare in patients on a
maintenance dose of ganciclovir, foscanet or cidofovir
7.
a.T
b.T c.T d.F e.F
234
8.True statements about HAART (highly active anti-retroviral therapy) include:
a. it reduces the recurrent rate of CMV retinitis
b. it is useful in maintaining a high level of CD 4+ counts
c. it is useful in treating acute retinal necrosis
d. it causes increased vitritis
e. there is an increased risk of cystoid macular oedema
8.
a.T b.T c.F d.T e.T
9. In AIDS patients with toxoplasma retinitis:
a. brain abscess occurs in 40% of cases
b. the retinitis is more severe than immunocompetent
patients
c. treatment is only needed in patients with impaired vision
d. retinal detachment is a common cause of visual loss
e. steroid is contraindicated
9.
a.T b.T c.F d.F e.T
10. A pregnant woman develops toxoplasmosis in her first trimester of
pregnancy. The following are true:
a. there is a 40% chance of the fetus acquiring the infection
b. subsequent offsprings are at risk of infections
c. the Ig M antibody will be raised
d. abortion is advised due to the high risk of fetal
malformation
e. sulphadiazine is the preferred treatment than clindamycin
10.
a.T b.F
c.T d.F e.F
235
11. True statements about pars planitis include:
a. it is associated with multiple sclerosis in 5% of cases
b. sheathing of the vein is caused by perivascular cuffing
of lymphocytes
c. snowbanking is the result of hard exudate
d. vitreous balls are composed of epitheloid cells and
multinucleated giant cells
e. vitreous haemorrhage is the most common cause of
visual loss
11.
a.T b.T
c.F d.T e.F
12. A 20 year-old man complains of decreased right vision. Four weeks earlier,
he had undergone an extensive bowel operation for Crohn's disease and had
received hyperalimentationa dn intravenous antibiotics since the operation. On
fundal examination, a fluffy white choroidal lession is seen near the macula.
There is minimal vitritis. The following are true about the organisms most likely
to be responsible for this condition:
a. culture of the blood is usually positive
b. although commonly seen in the respiratory tract, it
rarely causes pneumonia
c. this organism will grow on the Sabouraud's culture
within 48 hours
d. the organism rarely invade the retina
e. intravenous vancomycin is the treatment of choice
12.
a.F
b.T
c.T d.F
e.F
13. In serpiginous choroiditis:
a. there is loss of retinal pigment epithelium and the choroid
b. the lesion typically begins near the optic disc
236
c. visual loss is caused chiefly by cystoid macular oedema
d. fluorescein angiography in the acute phase reveals early
hyperfluorescence of the lesions
e. systemic cyclosporine is the treatment of choice
13.
a.T
b.T c.F
d.F e.F
14. True statements about APMPPE (acute posterior multifocal placoid pigment
epitheliopathy)
a. females are more commonly affected than males
b. recurrence does not occur
c. in the later phase of the fluorescein angiography, the
lesions appears hyperfluorescent
d. pigmentary disturbance is common in the recovery
phase
e. most patients maintain normal vision on recovery
14.
a.F b.F c.T d.T e.T
15. The following drugs are known to cause anterior uveitis:
a. topical alphagan
b. rifabutin
c. rifampicin
d. digoxin
e. alpha-chymotrypsin
15.
a.T
b.T
c.F
d.F
e.T
16. In Schwartz's syndrome:
a. there is raised intraocular pressure
237
b. the underlying cause is rhegmatogenous retinal
detachment
c. granulomatous uveitis is a feature
d. there is forward movement of the lens-iris diaphragm
e. macular oedema is a main cause of visual loss
16.
a.T
b.T
c.F
d.F
e.F
17. In Fuch's heterochromic cyclitis:
a. Koeppe's nodules are seen
b. abnormal vessels are seen bridging the angle of the
trabecular meshwork
c. cataract is seen in 50% of the cases
d. glaucoma is seen in 15% of the cases
e. there is an increased vitreous loss during
phacoemulsification
17.
a.T
b.T
c.T
d.F
e.F
18. Posner-Schlossman syndrome:
a. is painless
b. does not cause glaucomatous field loss
c. is associated with hypopyon in the majority of cases
d. is a self-limiting condition
e. should be treated with systemic steroid
18.
a.F
b.F
c.F
d.T
e.F
19.The following immunosupprassants and their side-effects are true:
a. cyclophophamide - haemorrhagic cystitis
b. cyclosporine - bone marrow dysplasia
238
c. azathioprine - nephrotoxicity
d. methotrexate - hepatic fibrosis
e. tacrolimus (FK506) - hyperglycaemia
19.
a.T
b.F
c.F
d.T
e.T
20. A 25 year-old man from the Middle East complains of blurred right vision.
On examination, he was found to have a small hypopyon in the right anterior
chamber. Posterior segment of the right eye reveals sheathing of the blood
vessels with retinal haemorrhages. He gave a past history of painful lesions in his
mouth and his genitals. The following are true about this condition:
a. histopathology of the painful lesion reveals
pathognomonic feature
b. puncturing the skin with an hypodermic needle may be
useful in diagnosis
c. the posterior segment pathology is caused by
inflammation of the retina and choroid
d. oral steroid alone is usually effective in preventing
relapse of the ocular inflammation
e. unless treated aggressively, the ocular inflammation will
become chronic
20.
a.F b.T
c.F d.F e.F
Return to the main page
Cornea MCQs
1. Conditions that can give rise to the following appearance include:
239
a. superior limbic keratoconjunctivitis
b. Sjogren's syndrome
c. Bell's palsy
d. Thygeson's keratitis
e. adenoviral keratitis
a.F
b.T
c.T
d.F
e.F
2. Conditions that may be associated with the following cornea appearance include:
a. Sjogren's syndrome
b. cerebrovascular accident
c. prolonged patching
d. iritis
e. herpes simplex keratitis
a.T
b.T
c.T
d.F
e.F
3. With regard this physical sign:
240
a. the abnormality is seen in the stroma
b. the underlying condition is inherited in an autosomal dominant fashion
c. its absence makes the diagnosis of the underlying condition unlikely
d. it usually begins at the nasal limbus
e. a brain MRI scan may of this patient may show the picture below
a.F
b.F
c.F
d.F
e.T
4. The following are true about this condition:
241
a. it is related to solar damage
b. it is seen in chronic corneal inflammation
c. the Bowman's membrane is replaced by scar tissue
d. histology shows amyloid deposits
e. penetrating keratoplasty is useful in patients with involvement of the
visual axis
a.F
b.T
c.T
d.F
e.F
5. This is the corneal appearance of a contact lens wearer. The picture on the right is the organism
responsible:
Figure 1
a. the organism originates in the tap water
b. a corneal biopsy may show the picture below (Figure 3)
c. the condition is responsive to gentamicin
d. ciprofloxacin can be used as monotherapy
e. the organism is difficult to eradicate
242
Figure 2
Figure 3
a.F
b.F
c.T
d.T
e.F
Click here for more MCQs
Cornea MCQs
1. The following specimen is taken from a patient who has bilateral abnormal
corneal
appearance. Similar corneal changes are also seen in some of his relatives.
True
statements include:
a. this is an autosomal recessive condition
b. glaucoma occurs in 50% of the cases
c. corneal decompensation is common by 40 years of age
d. the slit-lamp examination may be similar to figure1 (below)
e. the electron microscopy may show changes similar to figure 2 (see below)
243
Figure 1
a.F
b.F
c.F
d.T
Figure 2
e.T
2. The following corneal appearance belongs to a patient who had had blunt
ocular
trauma::
a. it is commoner in secondary hyphaema than primary hyphaema
b. high intraocular pressure favors this appearance
c. histology will reveals red blood cells in the stroma
d. clearing of the opacity usually begins in the periphery
e. visual disturbance is usually minimal
a. T
b.T c.F
d.T e.F
3. With regard to the following condition:
244
a. the Bowman's layer is seldom involved.
b. there are three components (ie. a head, cap and body) to this tissue.
c. the whole abnormal structure is adherent to the corneal surface.
d. it is commoner in white than black population.
e. the following corneal topography may be taken from a patient with this
condition.
a.F
b.T
c.T
d.F
e.T
4. The following corneal topography is taken from a patient with bilateral poor
vision:
245
a. there is inferior thinning of the cornea
b. against the rule astigmatism is common
c. iron deposition on the cornea is rare
d. corneal neovascularization is a common complication.
e. acute hydrop secondary to Descemet's membrane break can occur
a.T b.T c.T d.F e.T
5. True statements about the following corneal appearance include:
a. endothelial count is reduced
b. topical steroid is contraindicated in this condition
c. associated with sudden onset hearing loss
d. can occur after tick bite
e. corneal biopsy is useful for diagnosis
a.T
b.F c.T d.T e.F
246
Click here for more MCQs
Cornea MCQs
1. The tissue below is taken from a patient who had had a corneal graft. The following are
true about this condition:
a. the tissue has been stained with PAS
b. the picture above shows multiple abnormal endothelial cells
c. the patient is more likely to be a female than male
d. vision is typically worse towards the end of the day
e. the condition is painless
a.T
b.F c.T
d.F
e.F
2. True statements about ferritin lines include:
247
a. ferritin line around keratoconus (A) is called the Fleischer's ring
b. ferritin line at the edge of a trabeculectomy bleb (B) is called the Stocker's line
c. ferritin line seen in the elderly cornea (C) is called the Henle's line
d. ferritin line at the edge of pterygium (D) is called the Ferry's line
e. ferritin lines result from iron deposition from the tear
a.T
b.F
c.F d.F
e.T
3. True statements about this corneal condition include:
Slit-lamp appearance
a. it is an autosomal dominant condition
b. the corneal appearance usually appears in the third decade of life
248
Histology
c. ferritin lines are commonly seen on the epithelium
d. the corneal abnormality involves the Bowman's membrane
e. irregular astigmatism is a common cause of poor vision
a.T
b.F
c.T
d.T
e.T
4. Anterior segment signs associated with the appearance of these hands include:
a.
b.
249
c.
d.
a.T
b.T
c.T
d.F
e.
e.F
5. This is the corneal appearance of a patient with poor vision. The eye is white.
a. the patient is more likely to be a male than female
b. the poor vision is due to corneal astigmatism
c. lipid deposit in the cornea is a common feature
d. fluorescence staining shows epithelial defect in the periphery
e. descemetocele does not develop
a.T b.T c.T d.F e.F
Click here for more MCQs
250
Cornea MCQs
1. The following are true with regard to this condition:
a. this is an example of hamartoma
b. astigmatism is a common cause of poor vision in this condition
c. there may be problem with abduction
d. deafness is a recognized association
e. histology shows mainly adipose tissue only
a.F
b.T
c.T
d.F
e.F
2. True statements about the following corneal condition include:
251
a. it occurs at the junction between the Descemet's membrane and the endothelium
b. it has an autosomal dominant inheritance
c. glaucoma occurs in 90% of cases
d. it is a feature in Chandler's syndrome
e. it occurs in Peter's anomaly
a.F
b.T
c.F
d.F
e.T
3. This patient has this corneal conditions in both eyes. The following are true:
a. the lesions first appears in the deep corneal stroma
b. corneal graft is usually needed by the age of 30
c. the chance of the offsprings with this condition is 50%
d. enlarged corneal nerves are commonly seen
e. the following cornea specimen is taken from a patient with similar condition
a.F
b.F
c.T
d.F
e.F
llllllll
252
4. Association with the following appearance include:
a. past history of breast carcinoma
b. high serum creatinine
c. history of osteoarthritis
d. history of schizophrenia
e. history of coronary artery bypass
a. F
b.T
c.T
d.T
e.T
5. True statements about this corneal specimen include:
253
a. the specimen is stained with alcian blue
b. the peripheral cornea is typically involved
c. the condition is inherited in an autosomal dominant fashion
d. recurrent erosion may be a presenting symptom
e. it may be seen in the corneal button of this patient (picture below) who has
had a corneal graft
a.T
b.T
c.F
d.T
e.T
Click here for more MCQs
Paper I (answers)
1.
a.T
b.T
c.F
d.F e.F
Gorlin's syndrome (basal cell naevus syndrome)



2.
a.T
rare, autosomal dominant and multisystem disorder
characterised by multiple basal cell carcinoma, jaw cysts, skeletal anomalies,
ectopic calcification of the falx cerebri and pitting of the hand and feet
ocular features also include hypertelorism, lateral displacement of the medial
canthi and prominent supraorbital ridges
b.T
c.F
d.F
e.F
Posterior polymorphous dystrophy:



a bilateral dominantly inherited dystrophy
vesicular polymorphous deposits with clear halos in Descemet's membrane
usually asymptomatic, rarely endothelium decompensation requiring penetrating
corneal graft
254


3.
the abnormal endothelium may extend into the trabecular meshwork and iris
glaucoma can occur as a result of trabecular meshwork involvement iris
involvement can lead to corectopia and ectopia simulating iridocorneal endothelial
syndrome except that the later is unilateral
a.T
b.T
c.T
d.F
e.T
Crystalline deposits occurs with:




4.
myloidosis as in multiple myeloma
cystinosis
treatment with gold
uric acid from uraemia
a.T
b.F
c.F
d.F
e.T
Pterygium:




5.
a.T
fibrovascular overgrowth of the bulbar conjunctiva
the stroma shows basophilic degeneration (elastotic) of collagen
the epithelium is often thin but may show hyperplasia or dysplasia
it is not regarded as precancerous
b.T
c.F
d.T
e.T
Keratoconjunctivitis sicca (dry eyes):



6.
a.F
causes dysplasia of the epithelium which may eventually become keratinise
the goblet cells may increase due to chronic irritation from dry eyes
in Sjogren's syndrome, there is lymphocytic infiltration of the lacrimal and
accessory glands
b.F
c.T
d.F
e.F
Merkel cell carcinoma:




7.
a.F
an aggressive primary tumour of the skin, arising from papillary dermis
usually presents as a firm non-tender solitary skin nodule on the face and neck
early metastasis through the lymphatic channels
contains APUD (amine precursor and uptake decarboxylation) cells
b.T
c.F
d.F
e.T
255
Lens-induce uveitis:
- three types

hacoanalphylactic endophthalmitis
- severe granulomatous inflammation when the lens protein is exposed
- zonal granulomatous reaction surrounding the exposed lens material. The
inflammation include neutrophils, epihtelioid cells, macrophages and giant cells
phacotoxic uveitis
- nongranulomatous inflammation when the lens protein is exposed.
- non-specific inflammation with lymphocytes, macrophages and occasional giant
cells
phacolytic glaucoma
- occurs when the hypermature cataract leaks out protein
- macrophages laden with eosinophilic lens materials are seen in the anterior
chamber and the trabecular meshwork


8
a.F
b.F
c.T
d.T
e.T
Xeroderma pigmentosa:



inheritance is autosomal recessive
marked sensitive of skin to sunlight
malignant skin tumours including squamous cell tumour, basal cell carcinoma and
fibrosacorma
main defect is due to the inability of the cells to repair damaged DNA following
ultraviolet exposure

9.
a.T
b.F
c.T
d.T
e.T
Pathology of thyroid ophthalmopathy:





10.
there is enlargement and inflammation of orbital tissue especially the extraocular
muscles
histologically there is interstitial oedema and inflammatory cell infiltration (mainly
lymphocytes, plasma cells and sometimes mast cells)
thecondition tend to involve the nontendinous part of extraocular muscle
the endomysial fibroblasts produces mucopolysaccharide especially hyaluronic
acid
the muscles are initially inflammed and at later stage undergoes fibrosis and shows
fatty infiltrate
a.T
b.T
c.T
d.F
e.F
Histological staining:



Giemsa stain can demonstrate inclusion body such as chlamydia
osmium tetroxide is used to fix and stain myelin for electron microscopy
yeast and fungi can be stained with Fuelgin's stain, PAS, Luxol blue fast and
256

11.
Gomori methenamine silver
eosin stains the cytoplasm pink whereas haematoxylin stains the nuclei blue
a.T
b.F
c.F
d.T
e.F
Sympathetic ophthalmitis







12.
a uncommon bilateral granulomatous panuveitis which occurs after penetrating
ocular injury or intraocular surgery
the inflammation usually begins 4 to 8 weeks after the injury
Dellen-Fuch's nodules are seen at the level of Bruch's membrane and represent
aggregates of epitheloid
cells, it is not diagnostic as the nodules also occur in VKH syndrome
may have systemic manifestation identical to Vogt-Koyanagi-Harada syndrome
with cerebrospinal fluid
pleocytosis, miningismus, alopecia, vitiligo and poliosis
prevention is only useful if the injured eye were removed within the first 2 weeks
after injury
a.T
b.F
c.T
d.T
e.T
Abnormal material in the vitreous include:




13.
pseudoexfoliation in pseudoexfoliation syndrome
haemosiderin from vitreous haemorrhage
amyloidosis in familial amyloidosis
calcium in asteroid hyalosis
a.T
b .T
c.F
d.F
e.T
Giant cell arteritis:






14.
disease of the elderly
sudden, painless and profound visual loss
female more susceptible than male
head ache, low grade fever, anorexia, weight loss, tenderness upon brushing hair
and jaw claudication
ESR and C-reactive proteins are always raised but not diagnostic of the condition
diagnosis is base on biopsy which reveal fragmentation of the internal elastic
lamina and giant cell infiltration of the tunica media of the artery. However, giant
cells are not essential for diagnosis.
a.F
b.T
c.T
d.F
e.T
The following conditions are pre-malignant:

actinic keratosis is the result of metaplasia due to ultraviolet light
257

Bowen's disease
Oncocytoma of the caruncle results from metaplasia of the accessory gland and is not
thought to be pre-malignant
Squamous papilloma is benign hyperplasia of the skin
Syringoma is benign tumour of the sweat glands
15.
a.T
b.T
c.T
d.T
e.F
Hypoxic damage in diabetic mellitus is suggested by:





16.
cotton-wool spots
extensive IRMA
vascular beading
extensive retinal haemorrhages
iris neovascularization
a.T
b .F
c.T
d.F
e.T
Biopsy report in rheumatoid arthritis include



posterior scleritis
episcleral necrotic tissue
vasculitis
The dry eye in rheumatoid arthritis is caused by aqueous deficiency and not goblet cell
dysfunction
17.
a.T
b.F
c.F
d.F
e.F
The prognosis of retinoblastma is detemined by:



evidence of external spread such as optic nerve involvement
size of the tumours
cell differentiation
Calcification and necrosis are common in retinoblastoma.
18.
a.F
b.F
c.F
d.T
e.F
Microangiopathy in diabetes mellitus is characterised by:





microvascular obstruction and non-perfusion of capilaries
retinal capillary microaneurysms
absent membrane thickening
loss of pericytes
intraretinal microvascular abnormality
258
19.
a.T
b.F
c.T
d.T
e.F
Optic nerve glioma:






20
associated with type I neurofibromatosis
most common type is pilocytic (hair-like) astrocytoma
the age of onset is usually before the age of 10
low-grade and rarely infiltrate the perineural tissue
reactive meningeal hyperplasia
good long-term survival
a.F
b.T
c.F
d.T
e.T
Mutton fat keratic precipitates:
- occurs in chronic granulomatous uveitis and seen in










21.
tuberculosis
fungal
leprosy
syphilis
sarcoidosis
juvenile xanthogranuloma
histiocytosis X
sympathetic ophthalmia
Vogt-Koyanagi-Harada syndrome
toxoplasmosis
a.F
b.T
c.F
d.F
e.T
Albinism:





22.
can be of oculocutaneous or ocular types
oculocutaneous types can be divided into tyrosinase-positive and tyrosinasenegative types. Hair bulb
test is useful to differentiate the two within the first three years of life
is associated with abnormal platelet aggregation in Hermansky-Pudlak syndrome
poor vision is usually due to macular hypoplasia
a.T
b.T
c.F
d.T
e.F
Congenital glaucoma:
- may be primary
- secondary to:


aniridia
Sturge-Weber's syndrome
259

23.
anterior chamber angle cleavage syndrome such as Peter's anomaly or Axenfeld's
syndrome
a.F
bT
c.T
d.F
e.T
Nystagmus:





24.
see-saw nystagmus is seen with bitemporal hemianopia
down-beat nystagmus occurs with lesion in the cervico-medullary junction at the
foramen magnum
manifest latent nystagmus is a type of horizontal jerk nystagmus which increases
in amplitude when one eye is covered. It occurs in strabismus, amblyopia or
uniocular pathology such as cataract
gaze-evoked potential is not specific for cerebellar lesion, it occurs when the eyes
are unable to maintain an eccentric gaze position through weakness of muscle tone
in the agonist muscle
convergence retraction nystagmus occurs with mid-brain tumours such as
pinealoma
a.T
b.F
c.F
d.F
e.F
Melanocytoma:








25.
benign, highly pigmented tumour arising from melanocytes
most commonly found in optic nerve head
histologically shows plump polyhedral cells
equally sex incidence
commoner in black population
usually asymptomatic but may show enlargement of blind spots
50% have associated choroidal naevus
rarely the tumour shows necrosis, vascular obstruction and optic nerve
compression
a.F
b.F
c.T
d.T
e.F
Retinoblastoma:





most cases are due to new gene mutation only 10% is inherited
caused by deletion of 13q14
rosettese suggests cell differentiation and therefore better prognosis; however other
factors such as size and optic nerve involvement may be more important
has increased risk of osteosarcom (the oncogene of which is also located on
chromosome 13)
prognosis is generally good at around 90%
260
26.
a.T
b. T
c.T
d.F
e.F
Corneal dystrophy:
- inherited and bilateral conditions
- the following abnormal substance are documented:





27.
lycoaminoglycan (macular dystrophy, fleck dystrophy)
yaline degeneration (granular dystrophy)
amyloid (lattice dystrophy)
lipid (fleck dystrophy)
cholesterol (central crystalline dystrophy)
a.T
b.F
c.T
d.T
e.T
Adenocystic carcinoma of the lacrimal gland:






28.
the most common malignant tumour of the lacrimal gland
has no well-defined capsule
invade perineural tissue causing pain and metastasise early to the brain
5 histologic patterns: cribriform (Swiss cheese and of lower grade), solid
(basaloid), sclerosing,comedocarcinomatous and tubular (ductal)
treatment is with orbital exenternation and removal of involved bone
the prognosis is very poor
a.F
b.T
c.T
d.F
e.T
Dissociated vertical deviation:





29.
refers to the phenomenon in which the eye elevates when the amount of light
entering it is reduced
usually begins between 2 to 3 years of age
associated with infantile esotropia
the binocular vision is usually poor
surgery of choice is either superior rectus recession or Faden's procedure
a.T
b.T
c.F
d.F
e.T
Duane type A:




Brown's classification
Type A: limited abduction and less marked adduction limitation
Type B: limited abduction and normal adduction
Type C: limitation of adduction exceeds the limitation of abduction
widening of the palpebral fissure on abduction
globe retraction and palpebral narrowing on adduction
face turns to the affected side is common
261


30.
caused by innervation of the lateral rectus by a branch of third nerve in place of
absent sixth nerve
sixth nerve nucleus aplasia has been documented
a.F
b.T
c.F
d.T
e.F
Retinoschisis:







31.
presents in 5% of the population and is caused by the splitting of the neurosensory
retinal in the outer plexiform layer
often bilateral
arises from coalescence of peripheral cystoid degeneration
associated with hypermetropia
asymptomatic but can cause absolute scotoma (cf with retinal detachment which
causes relative scotoma)
does not cause water mark which is a feature of retinal detachment and is due to
transformation of escaped retinal pigment epithelial cells
renal breaks on the internal layer is common but if associated with breaks in the
outer layer, there is a risk of retinal detachment
a.F
b.F
c.T
d.T
e.T
Anti-glaucoma medications:




32.
beta-blocker causes slowing of heart rate but is not contra-indicated in primary
heart block ( prolonged PR interval)
beta-blocker should be avoided in patients taking centrally acting calcium channel
blocker such as verapamil. Nifedipine acts peripherally
acetazolide contains structure similar to sulphonamide and should be avoided in
those who is allergic to it
acetazolamide causes diuresis and loss of potassium, supplement may be needed
latanoprost is contraindicated in patients with intraocular inflammation
a.F
b.T
c. T
d.T
e.F
Dendritic ulcer of the cornea can be treated with:



33.
iodination
debridement
topical or systemic acyclovir
a.T
b.F
c.F
d.F
Choroidaemia:
262
e.F





34.
X-linked recessive disorder characterised by progressive degeneration of the
choriocapillaris and RPE
typically affects male
onset is in the first decade with night blindness
central vision is affected late
ERG shows reduced a and b waves female carriers may shows peripheral
pigmentary changes
a.T
b.T
c.T
d.T
e.F
Deafness occurs in:




35.
Cockayne's syndrome: premature ageing, dwarfism, bird like facies and retinal
degeneration
Altroem's syndrome: retinitis pigmentosa, deafness, obesity and diabetes mellitus
Alport's syndrome: haematuria, sensorineural deafness, anterioir lenticonus
Usher's syndrome: neurosensory deafness, retinitis pigmentosa
a.T
b.T
c.T
d.F
e.T
Enlarged corneal nerves occur in:













36.
multiple endocrine adenomatosis
keratoconus
ichthyosis
Refsum's disease
Fuchs' corneal dystrophy
neurofibromatosis
leprosy
trauma
congenital glaucoma
failed corneal graft
keratoconjucntivitis sicca
advanced age
acanthoamoeba keratitis
a.T
b.T
c.T
d.F
e.T
The following association are true:




abetalipoproteinaemia with acanthocytosis
Refsum's disease with cerebellar ataxia
Friedrich'a ataxia with spinocerebellar degeneration
Paget's disease and deafness
Homocystinuria causes recurrent aterial thrombosis
263
37.
a.T
b.T
c.T
d.F
e.T
Keratoconus occurs in:



38.
connective tissue disorders such as Ehlers-Danlos's syndrome and Marfan's
syndrome
atopic eye conditions such as atopic keratoconjunctivitis
Down's syndrome
a.F
b.F
c.T
d.T
e.F
Acquired syphilis:




causes painless ulcer in the primary stage which is highly infectious
in the secondary stage, the patient may develop a rash which is non-infectious
uveitis can occur in the secondary stage
treatment is with penicillin
Interstitial keratitis is a feature of congenital syphilis
39.
a.T
b.T
c.T
d.F
e.F
Reduced blinking occurs in:





40.
Parkinson's disease
progressive supranuclear palsy
alcohol intoxication
neurotrophic keratitis
contact lens wear
a.F
b.F
c.F
d.T
e.T
Heterochromic cyclitis:





41
chronic uveitis associated with cataract and glaucoma
glaucoma develops in 20% of patients
iris atrophy causes transillumination
Amsler's sign occurs during cataract extraction and is due to wispy iris vessels
which extends from the iris to the trabecular meshwork and do not cause anterior
synechiae
is resistant to steroid treatment
a. F
b.F
c.F
d.T
264
e.F
Retinal dialysis:




42.
caused by full hickness separation of the retina at the ora serrata
traumatic dialysis is most often found in the superonasal quadrant
idiopathic dialysis is most often in the inferotemporal quadrant
can be closed with cryotherapy to the base of the dialysis followed by local scleral
buckle
a.T
b.F
c.T
d.T
e.F
Ocular bobbing:





43.
conjugate involuntary recurrent downward movement of the eyes
rapid downward movement with a slower return to the neutral position
absent horizontal movements
occurs in patients with acute pontine lesion who are either comatose or locked-in
state
can occur with tumour in the pontine
a.T
b.T
c.T
d.T
e.T
Investigation of sarcoidosis:





44.
chest X-ray typically shows bilateral hilar lymphadenopathy, there may also be
interstitial infiltrate
ACE concentration is increased
conjunctival biopsy may show non-caseating granuloma
serum calcium may be raised in sarcoidosis
gallium scan shows increase in parts of body affected by sarcoidosis
a.T
b.F
c.F
d.T
e.T
Holme-Adie's pupil:





45.
there is light-near dissociation however there may be delayed or absent pupil
reaction to both
the condition is usually unilateral
women are affected more often than men
due to paralysis of the ciliary muscle, the accommodation is impaired
denervation hypersensitivity is demonstrated by constriction to 2.5% methacholine
and 0.1% pilocarpine
a.F
b.T
c.T
d.T
e.T
Absent of pupil response to direct and consensual light response:
265
- occur when the iris of the affected eye is paralysed and this occur in




46.
blunt trauma
third nerve palsy
atropine
siderosis bulbi
a.F
b.T
c.T
d.F
e.F
Leber's optic neuropathy:




47.
a mitochondrial inherited disease
bilateral loss of central vision which is severe and painless
typically in the second decade of life
classic early picture shows a triad of circumpapillary telangiectatic
microangiopathy, pseudoedema of the disc and absent fluorescein staining
a.T
b.T
c.T
d.F
e.F
AMPPE:






48.
typically affects healthy young adults who presents with sudden onset central or
paracentral visual loss after a flu like illness
scattered, patchy creamy lesions at the level of the retinal pigment epithelium
layers
the lesions fade after one to two weeks leaving behind granular pigmentary
changes
fluorescein angiography shows early blockage of choroidal circulation by these
lesions but in the late phase the lesions show late staining
other findings: uveitis, serous retinal detachment, cerebral vasculitis, cerebrospinal
fluid pleocytosis, headache, hearing loss and tinnitus
spontaneous resolution is common and systemic steroid has not been shown to be
useful
a.F
b.T
c.F
d.F
e.F
Central serous retinopathy:



49.
the patient usually presents with distorted vision without significant central visual
loss
Amsler's chart testing usually shows distortion of lines or scotoma
the image appears smaller than the unaffected eye
a.T
b.T
c.T
d.T
266
e.F
Kayser-Fleishcer's ring:




50.
occurs in Wilson's disease and intraocular copper foreign body
the ring is caused by copper deposited in Descemet's membrane which may be
orange, brown, green- brown or grey in colour
it begins superiorly then inferiorly and finally circumferentially
regresses with D-penicillamine treatment
a.T
b.F
c.T
d.F
e.T
In corneal graft:



51.
the epithlium, keratocytes, nerves and epithelium will eventually be replaced by
the host tissue
the endothelium does not regenerate and therefore will remains that of donor.
the descemet's membrane is produced by the endothelium and will therefore
remains that of donor
a.F
b.T
c.T
d.F
e.F
Vernal keratoconjunctivitis:





52.
affects mainly young people
an allergic inflammatory condition characterised by giant papillae in the tarsal
conjunctiva
the limbal type is found commonly in black with typical Horner-Trantas' dots
which are giant papillae containing eosinophils
causes keratitis and corneal ulcer / scarring
treatment is avoidance of allergens, mast cell stabilisers and short course of steroid
in acute phase
a.F
b.F
c.F
d.F
e.T
The following conditions are inherited as:




53.
X-linked in protanopia (red-green colour blindness) and juvenile retinoschisis
autosomal dominant in neurofibromatosis
autosomal recessive in blue cone achromatopsia
non-inherited in Sturge-Weber's syndrome
a.F
b.F
c.F
d.F
e.T
Coat's disease:

non-hereditary, unilateral congenital disorder characterised by abnormal
267



54.
telangiectatic and aneurysmal retinal vessels in the peripheral retina
male outnumber female by 3 to 1
presents with leukocoria and strabismus. Untreated can lead to exudative retinal
detachment, glaucoma and pthisical eye
laser is useful in destroying the abnormal blood vessels
a.F
b.T
c.F
d.F
e.F
Laser:




55.
green light is not absorbed by xanthophyll and therefore it is theoretically more
advantageous to use green laser in the macular area
diode laser penetrate vitreous haemorrhage well and is therefore ideal in vitreous
haemorrhage
YAG laser is colourless and therefore require neodymium to give it red colour for
targeting
used in ophthalmic practice is classified as type IV laser
a.T
b.T
c.F
d.T
e.F
Roth's spots:




56.
many causes including subacute bacterial endocarditis, leukaemia, anaemia etc
contain white areas in the centre of retinal haemorrhage
the white areas may contain immunocomplex, lymphoblasts or purulent exudates
in subacute bacterial endocarditis, the white centres may contain organisms,
although the majority are sterile and consists mainly of white blood cells and
fibrin thrombus at the site of extravasation of blood
a.T
b.F
c.F
d.F
e.T
Breaks in Descemet's membrane occurs in:



57.
trauma as in forcep delivery
keratoconus
congenital glaucoma
a.T
b.T
c.F
d.F
e.F
Stigma of acute angle closure glaucoma:




iris atrophy
sphincter paralysis due to ischaemia
pigment dispersion
glaukomflecken which is white opacities on the anterior surface of the lens caused
268
by necrosis of the anterior lens capsule
58.
a.T
b.T
c.T
d.F
e.F
In blow-out fracture:






59.
the orbital rim may be fracture or intact. In the former a step can be felt
occur only when the air sinuses have develop
surgical emphysema is a feature
although typically cause problem with upgaze, the horizontal recti have connective
tissue that extends to
the orbital floor and therefore horizontal movement may be impaired to some
extent
the infraorbital nerve is involved but the nerve does not supply the tip of the nose
which is by nasociliary nerve
a.T
b.T
c.T
d.T
e.T
Persistent hyperplastic primary vitreous:





60.
caused by abnormal regression of primitive hyaloid vascular system
typically there is a fibrovascular stalk extending from optic disc and form a
retrolental membrane
the membrane extend to the ciliary process and if contract can lead to elongation
of the ciliary processes
causes shallow anterior chamber and acute glaucoma in untreated cases
the prognosis is good if treated early especially if the retina were normal
a.F
b.T
c.F
d.T
e.F
Pars planitis:








commonly affect young adult and children
presents with folater or decreased visual acuity due to cystoid macular oedema
80% bilateral
clinical examination shows inflammatory cells and snowball opacity in vitreous
main complications are cataract and cystoid macular oedema. However, the visual
prognosis is usually good
rare complication include band keratopathy, glaucoma, vitreous organisation,
tractional retinal detachment and vitreous haemorrhage.
treatment is indicated only with decreased visual acuity from CMO and severe
inflammation
treatment include: periocular steroid, cryotherapy to vitreous base, par plana
vitrectomy and use of immunosuppresive agents.
Click here to return to Questions
Answers on Medical Retina
269
(click on the number to return to the questions)
1.
a.F b.T
c.T
d.T
e.T
Familial exudative vitreoretinopathy







FEVR is normally an autosomal dominantly inherited disease but X-linked inheritance has
been described.
although both eyes are affected, the degree of involvement
may be very asymmetrical.
the fundus appearances may be confused with retinopathy of prematurity (ROP)
it is characterized by peripheral areas of avascularity in the peripheral retina, almost
indistinguishable from ROP. The lack of history of premature birth, low birth weight, or
oxygen therapy differentiates this condition from retinopathy of prematurity.
dragging of the retina temporally with vessel straightening, subretinal exudation,
cicatrization and retinal detachment are all features of this condition. Complications
include neovascularisation in the peripheral retina.
treatment with cryotherapy to neovascular areas, and scleral buckling and vitrectomy
procedures for tractional detachments have all been used in the treatment.
relentless progression is uncommon. Visual impairment tends to occur early and it is rare
to lose vision after the age of 30 unless the patient develops tractional retinal
detachment.
2. a.T b.F c.F d.F e.F
Acute zonal occult outer retinopathy (AZOOR):
270

an idiopathic conditions which may be precipitated by punctate inner choroiodapthy (PIC),
multifocal choroiditis (MIC) or multiple evasnescent white dot syndrome (MEWDS)
 it is characterized by a rapid loss of visual field which
cannot be explained by the ophthalmoscopic changes





3.
a.T
majority of the sufferers are healthy young white myopic females.
initial presentation is with photopsia and increased blind spot in
the presence of normal visual acuity. Later, the visual field is
decreased with decreased visual acuity.
fluorescein angiography is not helpful as it is normal or might only
show the precipitation condition initially.
In electrophysiology, the electro-oculogram (EOG) light rise is
often reduced and the ERG is usually very abnormal.
there is no effective treatment. In some patients, the vision
returns spontaneously.
b.F c.F d.F e.F
Punctate idiopathic choriodopathy (PIC)

is commoner in myopic women in their 30s and 40s.
patients presents with blurred vision, paracentral scotoma and
photopsia.

the condition is bilateral in the majority of the patients

the acute lesions are small yellow lesions with slightly fuzzy
borders. there are no cells or other signs of inflammation in the
vitreous or anterior chamber.
the lesions gradually evolve into well defined scars and
slowly become more pigmented.


viral prodrome is not associated with PIC
the visual prognosis of eyes with that do not develop subfoveal
CNV is very good.
4. a.F b.T c.T d.T e.T
271
Idiopathic polypoidal choroidal vasculopathy (IPCV)





5.
a.T
is also known as posterior uveal bleeding syndrome and multiple
recurrent serosanguineous REP detachment syndrome.
although originally described in black hypertensive females in
middle age. It is now been recognized in other races.
the characteristic lesion appears to be an inner choroidal vascular
network of vessels ending in an aneurysmal bulge or outward
projection.
recurrent and multiple REP detachments with or without the
associated subretinal bleeding (posterior uveal bleeding
syndrome) may then occur.
the absence of drusen, retinal vascular disease and intraocular
inflammation is characteristic of the condition. Vitreous
haemorrhage may also occur.

the lesions were originally described to be peripapillary in
location, but pure macular lesions have also been reported.
indocyanine green angiography is most helpful in identifying
polyps.

it has a better prognosis than other causes of haemorrhagic
detachments of the retina. Direct laser therapy to the
lesion appears to carry a better prognosis in IPCV compared to
laser therapy of CNV in AMD patients.
b.F c.T
d.T
e.T
Stickler's syndrome:

autosomal disorders

associated with abnormal production of type II collagen
272


ocular features include myopia, cataracts, strabismus, and
optically-empty vitreous vitreous traction and lattice
degeneration
multiple retinal breaks occurs in more than 75% of the patients.
systemic manifestation include maxillary and mandibular
hypoplasia, cleft palate, abnormal uvula, neurosensory hearing
loss and skeletal abnormalities with joint hyperextensibility, and
marfanoid habitus
6. a.F b.F c.F d.T e.T
Difference between ARN and PORN:




both conditions are associated with herpes virus chiefly herpes
simplex and zoster
both conditions can lead to retinal detachment
ARN is seen both in healthy and immunocompromised patients
whereas PORN is seen exclusively in AIDS or
immunocompromised patients
vitritis is often severe in ARN but is usually minimal or absent in
PORN
ARN responds to intravenous acyclovir but PORN responds poorly
and PORN patients usually have poorer eventual visual outcome
7. a.F b.T c.F d.T e.F
Vigabatrin:






indicated only when all other appropriate antiepileptic drug
combinations have proved ineffective or poorly tolerated
is an GABA transaminase inhibitor
indicated as first line therapy only in infantile spasm
about 1/3 of epilepsy patients using it have characteristic visual
field defects which can vary from asymptomatic to severe and
disabling
the defect is not reversible even with cessation of the therapy
the cause of the visual field loss is unknown
not recommended in patients with pre-existing visual field defects
273
8.
a.F b.T
c.F d.T
e.F
Oguchi disease:






is a form of congenital stationary night blindness
is characterized by a golden/grey-white discoloration of the retina
giving a metallic sheen to the back of the eye. This disappeared
when the fundus was viewed after some time in the dark and has
become known as the Mizuo-Nakamura phenomenon.
visual acuity, colour vision and visual fields are usually normal in
Oguchi disease.
two genes involved in Oguchi disease have been identified
and include: arrestin, a gene located in the region of the distal
arm of chromosome 2q, and rhodopsin kinase. The arrestin
mutations are more common in Japanese Oguchi disease, and
rhodopsin kinase in European Oguchi disease.
patients are asymptomatic in light, but are night blind.
dark adaptation shows extremely retarded rod function.
9. a.T b.F c.T d.F e.F
Retinal crystals are seen in:

drug-induced:
tamoxifen
canthaxanthin
talc
methoxyflurane

metabolic disorders
cysintonosis
primary oxalosis type 1
274

others
Bietti retinal dystrophy
Sjorgren-Larsson syndrome
10. a.F b.T c.T d.F e.F
Shaken baby syndrome:




typically occurs in children less than 3 years of age and results
from violent shaking
there is no external eye injury but the posterior segment shows
retinal haemorrhages (both intra and subretinal) and vitreous
haemorrhage
skull fracture is uncommon but CT scan reveals subarachnoid or
intracerebral haemorrhages
vomiting, lethargy and focal neurologic findings are common
the visual prognosis is poor due to macular scarring, vitreous
haemorrhage and retinal detachment
11.
a.F b.F c.F d.T
e.T
In central retinal artery occlusion and ophthalmic artery occlusion:





both give cherry-red spot in the acute phase
both causes relative afferent pupillary defect
in ophthalmic artery occlusion, the a and b waves on the ERG are
abnormal due to insult to the outer and inner retina
in ophthalmic artery occlusion, both the choroidal and retinal circulation
are delayed
in ophthalmic artery occlusion, the REP is disturbed resulting in
pigmentary changes at a later date
12. a.F b.F c.T d.T e.T
Cystoid macular oedema without fluorescein leakage occurs in:
275



some cases of retinitis pigmentosa
X-linked juvenile retinoschisis
Goldman-Favre syndrome

nicotinic acid maculopathy
13.
a.F b.T
c.F d.F e.T
Rhegmatogenous retinal detachment is associated with:





posterior vitreous detachment
lattice degeneration
cystic retinal tuft
meridonal folds
white without pressure
14. a.T b.F c.T d.F e.F
Acute macular neuroretinopathy:





15.
an uncommon condition which causes paracentral scotoma in young adult
fundoscopy reveals dark lesion in the macula which is typically triangular
in shape
otherwise the fundus is normal
the lesion is confined to the macula and therefore the ERG is normal
the visual loss is mild and spontaneous recovery is common
a.T
b.T
c.T
d.T
e.T
von Hippel-Lindau's syndrome:

is an autosomal dominant condition with variable penetrance
276



50% of the patients has angiomatosis retinae which may be found in the optic disc
or the periphery
those in the optic disc or juxtapapillary may mimic choroidal neovascularization
the angioma enlarges slowly leading to visual loss mainly through exudative
maculopathy and also vitreous haemorrhage, tractional retinal detachment or
epiretinal membrane
16. a.F b.T c.T d.F e.F
Stargardt's disease:





is usually inherited in an autosomal recessive inheritance
it is regarded as a type of fundus flavimaculatus
lipofuscin substance is found within the REP cells which give rise to dark choroid
during fluorescein angiography
colour vision defect along the red-green axis is common
in late stage the macula can assume bull's eye maculopathy
most patients become symptomatic within the first or second decade of life
17. a.F b.T c.F d.F e.T
Wyburn-Mason's syndrome:







a type of phakomatoses
sporadic condition
characterized by arteriovenous communications of the retina and brain
beginning in adolescence
the arteriovenous communications of the retina gives rise to racemose
aneurysm of the retina
unlike von Hippel Lindau syndrome, exudative maculopathy is uncommon
instead the visual loss is due to the loss of capillary bed or overlying of the
vessels over the macula
tram-line calcification seen in skull X-ray is characteristic of SturgeWeber's syndrome
visual field defect is seen in one-third of the patient due to the vascular
abnormalities
18. a.F b.F c.T d.T e.T
Electrophysiology:


EOG is useful in detecting Best's disease but is normal in adult onset
foveomacular dystrophy
electroretinogram is abnormal in carrier of X-linked retinitis pigmentosa
and is diagnostic in Leber's congenital amaurosis
277


in Stargardt's disease, the ERG is variable and is therefore not useful for
diagnosis
carrier of choroideremia has normal ERG despite changes in the
peripheral retina
19. a.F b.T c.T d.T e.T
Bietti's crystalline dystrophy:






it is an autosomal recessive disorders
is characterized by crystalline deposits in all layers of the retina, cornea
and also in the lymphocytes
the crystals are made up of cholesterol and lipid
REP and choroidal atrophy are common features
patients present with night blindness and progressive visual loss
ERG and EOG are abnormal
20. a.T b.T c.T d.F e.F
Mucopolysaccharidosis associated with pigmentary retinopathy include:





Type I-H (Hurler)
Type I-S (Scheie)
Type II (Hunter)
Type III (Sanfilippo's)
Type IV (Morquio's)
Type VI (Maroteaux-Lamy's syndrome) and type VII (Sly's syndrome)
are not associated with pigmentary retinopathy
21.
a.T
b.T
c.F d.T
e.F
Optically empty vitreous (due to vitreous liquefaction) and peripheral retinal
pigmentary changes occur in:




Stickler's syndrome
Jansen's disease
Wagner's disease
Goldmann-Favre disease
278
22.
a.T
b.F c.T
d.T
e.T
Jansen's disease and Wagner's disease:





both conditions are autosomal dominant
high myopia, cataract and glaucoma are common in both conditions
the ERG are abnormal in both conditions
the EOG are normal in both conditions
the main distinguishing feature is the increased risk of retinal detachment
in Jansen's disease which is not seen in Wagner's disease
23. a.T b.T c.T d.T e.F
Macroaneurysms:





are more common in women than men
found mainly along the temporal arcade
causes retinal artery occlusion, retinal haemorrhages and vitreous
haemorrhages
symptomatic only if it bleeds or causes exudative maculopathy,
spontaneous closure is common due to thrombosis
IRVAN syndrome is made up of Idiopathic Retinal Vasculitis, Aneurysms
and Neuroretinitis. It is characterized by bilateral multiple bilateral
macroaneurysms
24. a.T b.T c.F d.T e.F
Carcinoma associated retinopathy (CAR):





is a paraneoplastic syndrome
colour vision defect, night blindness and rapid visual loss are typical
presentation
the condition may present before the diagnosis of the primary tumour
which is typically oat cell carcinoma of the lung
autoantibodies are commonly seen in the blood
both rod and cone ERG are abnormal
there is no effective treatment
25. a.T c.T c.T d.T e.T
Deafness and pigmentary retinopathy are seen in the following conditions:
279









26.
Congenital syphilis
Congenital rubella
Usher's syndrome
Cockaryne's syndrome
Alstroms syndrome
Leber's amaurosis
Alport's syndrome
Hunter's syndrome (MP II)
San Fillipo's disease (MP III)
a.T
b.T
c.T
d.T
e.T
Norrie's disease:





27.
a rare X-linked disorders
bilateral blindness with abnormal retina development
rosette formation is common in retina histology
1/3 has hearing abnormality
retrolental masses are common which may be mistaken for
retinoblastoma
a.F b.T
c.F d.T
e.F
X-linked retinoschisis:






splitting of the retina occurs at the nerve fibre layer
electrophysiology reveals negative ERG with normal a wave but reduced b
wave
the main cause of visual loss is central macular abnormalities
other causes of visual loss include retinal detachment and vitreous
haemorrhage
hypermetropia is an association
DNA analysis is useful in detecting the carrier, the abnormal gene is
located at XLRS1gene on the short arm of the X-chromosome
28. a.F b.T c.F d.T e.F
Significant macular oedema as defined by ETDRS are:
280



29.
retinal thickening within 500 microns of the centre of the fovea
hard exudate within 500 microns of the centre of the fovea with adjacent
retinal thickening
one disc area of retinal thickening, any part of which is within 1 disc
diameter of the centre of the fovea
a.F b.F c.F d.F e.F
Branch Retinal Vein Occlusion Study:




no treatment for macular oedema for at least 3 months
fluorescein angiography is not useful at the time of presentation due to
retinal haemorrhages, it is only useful when the haemorrhages clear
laser treatment is useful in patients with vision of 6/12 or less and without
evidence of macular ischaemia on fluorescein angiography
although sectorial panphotocoagulation is useful in reducing the risk of
vitreous haemorrhage in those patients who have more than 5 disc
diameter of ischaemia, the recommendation is for the PRP to be
performed only in the presence of neovascularization and not based on
ischaemia of more than 5 disc diameter as the majority of the eyes do not
develop neovascularization
30. a.F b.T c.T d.T e.F
Findings of the Diabetes Control and Complications Trial are:



intensive insulin treatment resulted in a statistically significant reduction
in incidence and progression of DR and the need for macular laser
panphotocoagulation or PRP
these only apply to insulin dependent diabetic
initial worsening during the first year was noted in some eyes, but after 3
years there was a beneficial effect on progression of DR
severe hypoglycaemia was more common in the tight control group
Click here to return to the questions
Click here to return to the main page
Answers on Ocular Motility
281
(click on the answer number to return to the questions)
1.
a.T
b.T


c.T
d.T
e.F
This patient has intermittent exotropia of the divergence excess type because there is a
difference of greater than 10 prism dioptre between near and distance and the patching
did not eliminate this difference.
Binocular single vision is often suppressed on distant fixation when the exotropia is
manifested, but it is usually normal with near fixation
 Patching is useful to suspend the tonic fusional convergence and
to
reveal the full latent deviation at near.
 Spontaneous resolution is uncommon for intermittent exotropia.
2. a.T b.T c.T d.F e.T
In intermittent exotropia:






3.
tinted glasses are useful if bright light constitutes a major
dissociative factor and allow the patient better control the
deviation
concave glasses are useful in stimulating accommodation and
convergence
orthoptic exercise is useful but is rarely successful if the deviation
is more than 15 prism dioptre
base-in prism is used preoperative to control
the deviation and is usually in combination with exercise
miotic decreases accommodation and therefore make the
exotropia worse
a.F b.F c.F d.T
e.T
Post-operative esotropia:


a small degree of consecutive esotropia is desirable and usually
decreases as the effect of the surgery is lessen.
postoperative diplopia is common and usually resolves within 2
282
weeks.
patching is not needed as the patient is now 8 year-old and
amblyopia is unlikely to develop.
4.
a.T
b.F c.F d.T




e.F
The history and the orthoptic findings suggest that the patient has
consecutive exotropia with dissociated vertical deviation and
latent nystagmus. The underlying condition prior to surgery is
likely to be infantile esotropia.
The majority of infantile esotropia has a deviation of greater than
30 prism dioptres.
The binocular single vision is usually subnormal even with early
surgery.
Asymmetrical optokinetic nystagmus is common.
A small exotropia is cosmetically acceptable and does not require
surgery.
5.
a.F b.T




6.
a.F b.T
c.F d.T
e.T
The diagnosis is dissociated vertical deviation. It is seen in
60 to 90% of infantile esotropia.
Bielchowsky phenomenon refers to elevation of the eye when the
light entering that eye is reduced.
The majority of the cases are bilateral but may be asymmetrical.
In inferior oblique muscle overaction, the eye does not elevate
when it is covered but shows elevation on adduction.
c.T
d.F e.F
283



The patient has latent nystagmus. Manifest latent nystagmus is
present when the nystagmus remains when both eyes are
uncovered.
The fast phase is toward the side of the uncovered eye and
increases on abduction.
Unlike congenital nystagmus, the wave form has a decreasing
velocity slow phase.
The cause is unknown but is not associated with cerebellar
dysfunction.
7.
a.F b.T




8.
a.T
b.T
c.F d.F e.F
Duane's retraction syndrome is more common in female than
male and more likely to affect the left than right eye.
The type of Duane's retraction syndrome is type II
Narrowing of the lid is caused by globe retraction.
Amblyopia is uncommon and found in only 10% of the patient.
c.F d.T
e.T
Associated signs are:







cataract
heterochromia iridies
microphthalmos
crocodile tears
Marcus Gunn Jaw winking
Goldenhar's syndrome
Klippel-Feil's syndrome
9.
a.F b.F c.T
d.T
284
e.T





10.
a.T
The strabismus is incomitant.
Surgery is indicated to reduce or eliminate the abnormal head
posture, or place the field of binocular single vision more central
and enlarge the field.
However, the ocular movement does not improve with surgery.
Resection can worsen the globe retraction and narrowing of the
lid fissure.
Faden procedure is useful in reducing the upshoot.
b.T
c.T
d.F e.T
In fourth nerve palsy:
11.
a.T

The affected side is hyperdeviated and therefore the image is
seen as lower and in addition the eye is extorted and therefore
the image will also appear intorted giving the diagram as shown
with the point of the V pointing towards the abnormal side

deviation of the eye in fourth nerve palsy depends on which eye is
the fixating eye. If the fixating eye is the right eye, then the left
eye will become hypodeviated and vice versa

in the three step test, the right eye is hyperdeviated in the
primary position, on left gaze and right head tilt
b.F c.F d.T
e.F
Feature of congenital fourth nerve palsy include:


absence of cyclotorsion
higher vertical fusional amplitude
Abnormal head posture occurs in any long-standing fourth nerve
285
palsy as a compensatory device and is not specific to congenital
palsy.
12. a.T b.T c.T d.F e.T
Features in favour of bilateral fourth nerve palsy include:

slight hyperdeviation in primary position



reversal of hyperdeviation and diplopia on lateral versions
large V pattern
chin depression being the main abnormal head posture with little
or no head tilt
extorsion of more than 10 degrees
positive Bielchowsky head tilt test with head tilt to either shoulder


13. a.T b.F c.T d.F e.F
Surgical treatment may involve:


14.
a.T
right inferior oblique recession or left inferior rectus recession to
control the vertical deviation
right Harada-Ito's procedure to control the cyclotorsion
b.F c.T
d.T
e.F
Sixth nerve palsy:





this patient has a right sixth nerve palsy
the esotropia is worse for distant than near
face turn to the affected side and therefore the right
duction is better than version and therefore the ocular movement
is better when the unaffected side is closed
V pattern is common on upgaze
286
15. a.F b.T c.T d.T e.F
Muscle sequelae are:



16.
overaction of the left medial rectus
contraction of the right medial rectus
secondary inhibitional palsy of the left lateral rectus
a.F b.T
c.F
d.T
e.T
Additional signs and location:





17.
a.T
both sixth nerve palsy and swollen discs can result from raised
intracranial pressure and are of not localizing value
presence of Horner's syndrome point to a lesion in the cavernous
sinus
presence of fourth nerve palsy can result from a lesion in the
cavernous sinus
Folville's syndrome result from a lesion in the dorsal pons with
ipsilateral abduction weakness, ipsilateral facial weakness and
analgesia, ipsilateral peripheral deafness, loss of taste from the
anterior two-third of the tongue and ipsilateral Horner's
syndrome.
Millard-Gubler's syndrome result from a lesion in the ventral
pons. The signs in additional to those seen in Folville's syndrome
has an additional contralateral hemiplegia.
b.T
c.T
d.T
e.T
Treatment of right sixth nerve palsy:


the use of botulinum toxin into the right medial rectus will reduce
the contracture of the right medial rectus
use of base out Fresnel prism over the paralysed can help to fuse
287

the double images
in patient without inhibitional palsy of the left lateral rectus, the
following operation may be used:
a. recession of the left medial rectus combined with
resection of the paresed lateral rectus
b. Faden operation on the left medial rectus

18.
a. T
in patient with fully developed muscle sequelae, the choice of
surgery is recession of the right medial rectus combined with
resection of the right lateral rectus.
b.F c.F d.T
e.T
Microtropia:






19.
a common form of anomalous binocular single vision
the strabismus is 10 prism dioptres or less
the squinting eye has reduced vision
anisometropia is found in most cases
stereopsis is reduced but rarely absent
foveal suppression scotoma is present in the affected eye and can
be tested by placing a base out 4 dioptre prism over the affected
eye; there is absent of eye movement.
a.F b.T
c.F d.T
e.F
Spasmus nutans:





consists of nystagmus, involuntary head movement and abnormal
head posture
usually begins between 3 to 18 months of age
nystagmus is jerky but of small amplitude and high frequency
involuntary head movement can be either head nodding or
shaking or both
most resolves by 3 years of age
288

20.
a.T
rarely associated with neurologic dysfunction but in some cases
may be associated with glioma of the chiasm
b.T
c.T
d.T
e.F
Accommodative esotropia:



usually occurs around 2 years of age
patients with accommodative esotropia usually suppress the
deviating eye and therefore diplopia is uncommon; however due
to the suppression amblyopia is common unless the patient has
alternating esotropia
there is usually hypermetropia of greater than +3.00D or a high
AC/A ratio
289